Permutation and Combination

You might also like

Download as pdf or txt
Download as pdf or txt
You are on page 1of 67

WWW.SARKARIPOST.

IN

Foundation Level
1. The sum of all the four digit even numbers which can be (a) 72 (b) 144
formed by using the digits 0, 1, 2, 3, 4 and 5 if repetition of (c) 84 (d) 192
digits is allowed is 10. Every body in a room shakes hands with every else. If total
(a) 1765980 (b) 1756980 number of hand-shaken is 66, then number of persons in
(c) 1769580 (d) 1759680 the room is
2. How many words beginning with vowels can be formed (a) 11 (b) 12

WWW.SARKARIPOST.IN
with the letters of the word EQUATION? (c) 13 (d) 14
(a) 25200 (b) 15200 11. The number of words from the letters of the words BHARAT
(c) 25300 (d) 35200 in which B and H will never come together, is
3. The number of words that can be formed out of the letters (a) 360 (b) 240
of the word COMMITTEE is (c) 120 (d) None of these
9! 9! 12. A bag contains 3 black, 4 white and 2 red balls, all the balls
(a) 3 (b) 2 being different. The number of at most 6 balls containing
(2!) (2!)
balls of all the colours is
9! (a) 42(4!) (b) 26 × 4!
(c) (d) 9!
2! (c) (26 – 1)(4!) (d) None of these
4. If 10Pr = 720, then r is equal to 13. How many different ways are possible to arrange the letters
(a) 4 (b) 2 of the word “MACHINE” so that the vowels may occupy
(c) 3 (d) 1 only the odd positions?
5. The number of ways of selecting exactly 4 fruits out of 4 (a) 800 (b) 125
apples, 5 mangoes, 6 oranges is (c) 348 (d) 576
(a) 10 (b) 15 14. If nPr = nPr + 1 and nCr = nCr – 1, then the values of n and r
(c) 20 (d) 25 are
6. Number of ways in which 12 different balls can be divided (a) 4, 3 (b) 3, 2
into groups of 5, 4 and 3 balls are (c) 4, 2 (d) None of these
12! 12! 15. If nPr = 720 nCr, then r is equal to
(a) (b)
5!4! 5!4!3! (a) 3 (b) 7
12! (c) 6 (d) 4
(c) (d) None of these
5!4!3!3! 16. In how many ways a hockey team of eleven can be elected
7. How many different letter arrangements can be made from from 16 players?
the letter of the word EXTRA in such a way that the vowels (a) 4368 (b) 4267
are always together? (c) 5368 (d) 4166
(a) 48 (b) 60 17. The number of values of r satisfying the equation
(c) 40 (d) 30 39 39 39 39
C3 r 1 Cr 2 Cr 2 1 C3r is
8. In how many ways can a committee of 5 made out 6 men
and 4 women containing atleast one woman? (a) 1 (b) 2
(c) 3 (d) 4
(a) 246 (b) 222
18. The total number of all proper factors of 75600 is
(c) 186 (d) None of these
(a) 120 (b) 119
9. How many integers greater than 5000 can be formed with
the digit 7, 6, 5, 4 and 3, using each digit at most once? (c) 118 (d) None of these

PDF Download FROM >> WWW.SARKARIPOST.IN Join >> https://www.facebook.com/Sarkaripost.in/


WWW.SARKARIPOST.IN
582 Quantitative Aptitude

19. In how many ways can six different rings be worn on four BAT & PAT but not through SAT. Further there are 3 ways
fingers of one hand? to become a CA(viz., Foundation, Inter & Final). Find the
(a) 10 (b) 12 ratio of number of ways in which an Engineer can make it
(c) 15 (d) 16 to Oxford University to the number of ways a CA can make
20. Find the number of ways in which 8064 can be resolved as it to Oxford University.
the product of two factors? (a) 3 : 2 (b) 2 : 3
(a) 20 (b) 21 (c) 2 : 9 (d) 9 : 2
(c) 22 (d) 24 30. How many straight lines can be formed from 8 non-collinear
21. In how many ways can twelve girls be arranged in a row if points on the X-Y plane?
two particular girls must occupy the end places? (a) 28 (b) 56
(c) 18 (d) 19860
10!
(a) (b) 12! 31. A man has 3 shirts, 4 trousers and 6 ties. What are the number
2!
of ways in which he can dress himself with a combination
12! of all the three?
(c) 10! × 2! (d)
2! (a) 13 (b) 72

WWW.SARKARIPOST.IN
22. To fill a number of vacancies, an employer must hire 3 (c) 13!/3! 4! 6! (d) 3! 4! 6!
programmers from among 6 applicants, and 2 managers 32. If (28C2r : 24C2r–4) = 225 : 11. Find the value of r.
from among 4 applicants. What is the total number of ways (a) 10 (b) 11
in which she can make her selection? (c) 7 (d) 9
(a) 1,490 (b) 132 33. There is a question paper consisting of 15 questions. Each
(c) 120 (d) 60 question has an internal choice of 2 options. In how many
23. A father has 2 apples and 3 pears. Each weekday (Monday ways can a student attempt one or more questions of the
through Friday) he gives one of the fruits to his daughter. given fifteen questions in the paper?
In how many ways can this be done? (a) 37 (b) 38
(a) 120 (b) 10 (c) 3 15 (d) 315 – 1
(c) 24 (d) 12 34. How many numbers can be formed with the digits 1, 6, 7, 8,
24. If a secretary and a joint secretary are to be selected from a 6, 1 so that the odd digits always occupy the odd places.
committee of 11 members, then in how many ways can they (a) 15 (b) 12
be selected? (c) 18 (d) 20
(a) 110 (b) 55 35. There are 6 boxes numbered 1, 2, ... 6. Each box is to be
(c) 22 (d) 11 filled up either with a red or a green ball in such a way that
25. On a railway route there are 20 stations. What is the number at least 1 box contains a green ball and the boxes containing
of different tickets required in order that it may be possible green balls are consecutively numbered. The total number
to travel from every station to every other station? of ways in which this can be done is
(a) 40 (b) 380 (a) 5 (b) 21
(c) 400 (d) 420 (c) 33 (d) 60
26. If P(32, 6) = kC (32, 6), then what is the value of k? 36. In how many ways five chocolates can be chosen from an
(a) 6 (b) 32 unlimited number of Cadbury, Five-star, and Perk
(c) 120 (d) 720 chocolates?
27. How many times does the digit 3 appear while writing the (a) 81 (b) 243
integers from 1 to 1000? (c) 21 (d) 31
(a) 269 (b) 308 37. There are 20 people among whom two are sisters. Find the
(c) 300 (d) None of these number of ways in which we can arrange them around a
28. A person X has four notes of rupee 1, 2, 5 and 10 circle so that there is exactly one person between the two
denomination. The number of different sums of money she sisters.
can form from them is (a) 18! (b) 2!19!
(a) 16 (b) 15 (c) 19! (d) None of these
(c) 12 (d) 8 38. In a company, each employee gives a gift to every other
29. There are 4 qualifying examinations to enter into Oxford employee. If the number of gifts is 61, then the number of
University: RAT, BAT, SAT, and PAT. An Engineer cannot employees in the company is :
go to Oxford University through BAT or SAT. A CA on the (a) 11 (b) 13
other hand can go to the Oxford University through the RAT, (c) 12 (d) 8

PDF Download FROM >> WWW.SARKARIPOST.IN Join >> https://www.facebook.com/Sarkaripost.in/


WWW.SARKARIPOST.IN
Permutations and Combinations 583

39. In how many ways can Ram choose a vowel and a constant 50. How many 6 digit number can be formed from the digits 1,
from the letters of the word ALLAHABAD? 2, 3, 4, 5, 6 which are divisible by 4 and digits are not
(a) 4 (b) 6 repeated?
(c) 9 (d) 5 (a) 192 (b) 122
40. There are three rooms in a hotel: one single, one double (c) 140 (d) 242
and one for four persons. How many ways are there to house 51. There are 5 candidates in an election and 3 of them are to
seven persons in these rooms? be elected. A voter can cast any number of votes but not
(a) 7!/1!2!4! (b) 7! more than three. The number of ways in which he can cast
(c) 7!/3 (d) 7!/3! his vote is
41. The digits, from 0 to 9 are written on 10 slips of paper (one (a) 5 (b) 15
digit on each slip) and placed in a box. If three of the slips (c) 20 (d) 25
are drawn and arranged, then the number of possible 52. If 2n+1Pn–l : 2n–1Pn = 3 : 5, the possible value of n will be :
different arrangements is (a) 3 (b) 5
(a) 1000 (b) 720 (c) 4 (d) 2
(c) 810 (d) None of these

WWW.SARKARIPOST.IN
53. All possible two factors products are formed from the
42. The number of ways in which 7 different books can be given numbers 1, 2, 3, 4, ....., 200. The number of factors out of
to 5 students if each can receive none, one or more books is total obtained which are multiples of 5 is
(a) 57 (b) 75 (a) 5040 (b) 7180
(c) 11C5 (d) 12! (c) 8150 (d) None of these
43. In how many ways can 13 different alphabets (a, b, c, ... m) 54. A set of 15 different words are given. In how many ways is
be arranged so that the alphabets f and g never come together? it possible to choose a subset of not more than 5 words?
(a) 13 ! – 12 ! (b) 13 ! – 12! / 2! (a) 4944 (b) 415
(c) 13 ! – 2 × 12 ! (d) None of these (c) 15 4 (d) 4943
44. Number of ways in which the letters of word GARDEN 55. In an examination, there are 3 multi-choice questions and
can be arranged with vowels in alphabetical order, is each question has 4 alternatives. If a student is declared
(a) 360 (b) 240 pass only when he attempts all question correctly, then
(c) 120 (d) 480 number of ways in which he can fail is
45. The number of ways in which a mixed double tennis game (a) 1 (b) 12
can be arranged from amongst 9 married couple if no (c) 27 (d) 63
husband and wife plays in the same game is
56. Seven nouns, five verbs, and two adjectives are written on
(a) 756 (b) 3024
a blackboard. We can form a sentence by choosing one
(c) 1512 (d) 6048 word of each type, and we do not care about how much
46. In how many ways can 21 identical white balls and 19 sense the sentence makes. How many ways are there to do
identical black balls be arranged in a row so that no 2 black this?
balls are together? (a) 72 × 52 × 22 (b) 71 × 51 × 21 × 3!
(a) 1540 (b) 1640
(c) 7! × 5! × 2! (d) 27 × 25 × 22
(c) 1240 (d) 1440
57. In how many ways can the eight directors, the vice-
47. If 5 parallel straight lines are intersected by 4 parallel chairman and the chairman of a firm be seated at a round-
straight, then the number of parallelograms thus formed is table, if the chairman has to sit between the vice-chairman
(a) 20 (b) 60 and the director?
(c) 101 (d) 126 (a) 9! × 2 (b) 2 × 8!
48. The number of ways in which a couple can sit around a (c) 2 × 7! (d) None of these
table with 6 guests if the couple take consecutive seat is
58. How many 4 digit numbers divisible by 5 can be formed
(a) 1440 (b) 720
with the digits 0, 1, 2, 3, 4, 5, 6 and 6?
(c) 5040 (d) None of these
(a) 220 (b) 249
49. How many different words beginning with O and ending
(c) 432 (d) 288
with E can be formed with the letters of the word
59. The number of circles that can be drawn out of 10 points of
ORDINATE, so that the words are beginning with O and
which 7 are collinear is
ending with E?
(a) 8! (b) 6! (a) 130 (b) 85
(c) 7! (d) 7!/2! (c) 45 (d) Cannot be determined

PDF Download FROM >> WWW.SARKARIPOST.IN Join >> https://www.facebook.com/Sarkaripost.in/


WWW.SARKARIPOST.IN
584 Quantitative Aptitude

Standard Level
1. The number of ways of choosing a committee of 2 women (a) 36 (b) 24
and 3 men from 5 women and 6 men, if Mr. A refuses to (c) 42 (d) 6
serve on the committee if Mr. B is a member and Mr. B can 12. The number of ways of distributing 8 identical balls in 3
only serve, if Miss C is the member of the committee, is distinct boxes so that none of the boxes is empty is
(a) 60 (b) 84 (a) 8C3 (b) 21
(c) 124 (d) None of these (c) 38 (d) 5
2. 5 men and 6 women have to be seated in a straight row so 13. How many different nine digit numbers can be formed from
that no two women are together. Find the number of ways the number 223355888 by rearranging its digits so that the
this can be done. odd digits occupy even positions?
(a) 48400 (b) 39600 (a) 16 (b) 36
(c) 9900 (d) 86400 (c) 60 (d) 180

WWW.SARKARIPOST.IN
3. The total number of ways in which 8 men and 6 women can 14. If two dices are tossed simultaneously, the number of
be arranged in a line so that no 2 women are together is elements in the resulting sample space is
(a) 48 (b) 8P8.9P6 (a) 6 (b) 8
(c) 8! (84) (d) 8C8.9C8 (c) 36 (d) 24
4. How many words can be formed with the letters of the word 15. A boy has 3 library cards and 8 books of his interest in the
INTERNATIONAL? library. Of these 8, he does not want to borrow Chemistry
(a) 129729600 (b) 129729500 part II unless Chemistry part I is also borrowed. In how
(c) 29729600 (d) 127829600 many ways can he choose the three books to be borrowed?
5. Numbers of ways in which atleast three fruits be selected (a) 56 (b) 27
out of 20 fruits in which 10-mangoes, 5-apples, 2-oranges (c) 26 (d) 41
and rest are different, are 16. By stringing together 9 different coloured beads, how many
(a) 1583 (b) 1577 different bracelets can be made?
(c) 1559 (d) None of these (a) 20160 (b) 40320
6. The number of different ways in which 8 persons can stand (c) 80640 (d) 10080
in a row so that between two particular person A and B there 17. How many ways are there to place a set of chess pieces on
are always two person, is the first row of chessboard. The set consists of a king, a
(a) 60 (5!) (b) 15(4!) × (5!) queen, two identical rooks, knights & bishops?
(c) 4! × 5! (d) None of these (a) 8! (b) 88
7. The total number of eight digit numbers in which all digits (c) 5040 (d) 4280
are different, is 18. In how many ways can 7 persons stand in the form of a
(a) 9!9 (b) 9!9/2 ring?
(c) 9! (d) None of these (a) P (7, 2) (b) 7 !
8. In how many ways can the letters of the word 7!
“VALEDICTORY” be arranged so that the vowels are never (c) 6 ! (d)
2
separated?
(n 2)! (n 1) (n 1)!
(a) 883490 (b) 967680 19. What is equal to?
(n 1) (n 1)!
(c) 563680 (d) 483840 (a) 1
9. From 6 boys and 7 girls a committee of 5 is to be formed so (b) Always an odd integer
as to include atleast one girl. The number of ways this can (c) A perfect square
be done is (d) None of the above
(a) 13C4 (b) 6C4 . 7C1 20. In a football championship 153 matches were played. Every
6
(c) 7 . C4 (d) 13C5 – 6C1 team played one match with each other team. How many
10. The number of all possible selections of one or more teams participated in the championship?
questions from 10 given questions, each question having (a) 21 (b) 18
one alternative is
(c) 17 (d) 15
(a) 310 (b) 210 – 1
10
(c) 3 – 1 (d) 210 21. If P(77, 31) = x and C (77, 31) = y, then which one of the
11. The number of ways in which 13 gold coins can be following is correct?
distributed among three persons such that each one gets at (a) x = y (b) 2x = y
least two gold coins is (c) 77x = 31 y (d) x > y

PDF Download FROM >> WWW.SARKARIPOST.IN Join >> https://www.facebook.com/Sarkaripost.in/


WWW.SARKARIPOST.IN
Permutations and Combinations 585

22. 6 equidistant vertical lines are drawn on a board. 6 32. A shopkeeper has 10 copies of each of nine different books,
equidistant horizontal lines are also drawn on the board then number of ways in which atleast one book can be
cutting the 6 vertical lines, and the distance between any selected is
two consecutive horizontal lines is equal to that between (a) 911 – 1 (b) 1010 – 1
any two consecutive vertical lines. What is the maximum 9
(c) 11 – 1 (d) 109
number of squares thus formed? 33. How many numbers greater than one million can be formed
(a) 37 (b) 55 with 2, 3, 0, 3, 4, 2, 3? (repetitions not allowed)
(c) 91 (d) 225 (a) 720 (b) 360
23. In how many ways can 12 papers be arranged if the best (c) 120 (d) 240
and the worst paper never come together? 34. 5 Indian and 5 American couples meet at a party & shake
(a) 12!/2! (b) 12! – 11! hands . If no wife shakes hands with her husband and no
(c) (12! – 11!)/2 (d) 12! – 2.11! Indian wife shakes hands with a male, then the number of
24. In the Suniti building in Mumbai there are 12 floors plus hand shakes that takes place in the party is
the ground floor. 9 people get into the lift of the building on (a) 95 (b) 110
the ground floor. The lift does not stop on the first floor. If (c) 135 (d) 150

WWW.SARKARIPOST.IN
2, 3 and 4 people alight from the lift on its upward journey, 35. In a college examination, a candidate is required to answer
then in how many ways can they do so? 6 out of 10 question which are divided into two section
(Assume they alight on different floors.) each containing 5 questions. Further the candidates is not
(a) 11C3 × 3P3 (b) 11P3 × 9C4 × 5C3 permitted to attempt more than 4 questions from either of
(c) 10 9 5
P3 × C4 × C3 (d) 12C3 the section. The number of ways in which he can make up a
25. In how many ways can a selection be made of 5 letters out choice of 6 question is
of 5As, 4Bs, 3Cs, 2Ds and 1E? (a) 200 (b) 150
(a) 70 (b) 71 (c) 100 (d) 50
(c) 15 C5 (d) None of these 36. The total number of ways in which letters of the word
ACCOST can be arranged so that the two C's never come
26. If a team of four persons is to be selected from 8 males and
together will be
8 females, then in how many ways can the selections be
(a) 120 (b) 360
made to include at least one male.
(c) 240 (d) 6 ! – 2 !
(a) 1550 (b) 1675
37. In how many ways can a term of 11 cricketers be chosen
(c) 1725 (d) 1750
from 6 bowlers. 4 wicket keepers and 11 batsmen to give a
27. Letters of the word DIRECTOR are arranged in such a way
majority of bastemen if at least 4 bowlers are to be included
that all the vowels come together. Find out the total number
and there is one wicket keeper?
of ways for making such arrangement.
(a) 27730 (b) 27720
(a) 4320 (b) 2720
(c) 17720 (d) 26720
(c) 2160 (d) 1120
38. There are 4 boys and 4 girls. In how many ways can they be
28. 4 boys and 2 girls are to be seated in a row in such a way
seated ina row so that all the girls do not sit together?
that the two girls are always together. In how many different
(a) 17440 (b) 37440
ways can they be seated?
(c) 37340 (d) 37450
(a) 1200 (b) 7200
39. Rajdhani express going from Bombay to Delhi stops at 5
(c) 148 (d) 240
intermediate stations. 10 passengers enter the train during
29. In how many ways can 7 Englishmen and 7 Americans sit
the journey with ten different ticket of two classes .The
down at a round table, no 2 Americans being in consecutive number of different sets of tickets they may have is
positions?
(a) 15C10 (b) 20C10
(a) 3628800 (b) 2628800
(c) 30C10 (d) None of these
(c) 3628000 (d) 3328800
40. Find the minimum possible number of boxes that Rahul
30. In a jet there are 3 seats in front and 3 in the back. Number
must have, given that the total number of coins in all the
of different ways can six persons of different heights be
boxes put together is between 3235 and 3256 (both values
seated in the jeep, so that every one in front is shorter than
inclusive).
the person directly behind is
(a) 110 (b) 111
(a) 90 (b) 60
(c) 54 (d) 15 (c) 117 (d) 118
31. The total number of integral solutions for (x, y, z) such that 41. Three dice are rolled. The number of possible outcomes in
xyz = 24 is which at least one die shows 5 is
(a) 36 (b) 90 (a) 215 (b) 36
(c) 120 (d) None of these (c) 125 (d) 91

PDF Download FROM >> WWW.SARKARIPOST.IN Join >> https://www.facebook.com/Sarkaripost.in/


WWW.SARKARIPOST.IN
586 Quantitative Aptitude

42. The sides AB, BC, CA of a traingle ABC have 3, 4 and 5 of selecting 10 balls if twice as many red balls as green
interior points respectively on them. The total number of balls are to be selected, is
triangles that can be constructed by using these points as (a) 3 (b) 4
vertices is (c) 6 (d) 8
(a) 220 (b) 204 52. Out of 10 consonants and four vowels, the number of words
(c) 205 (d) 195 that can be formed using six consonants and three vowels
43. If all permutations of the letters of the word AGAIN are is
arranged as in dictionary, then fiftieth word is (a) 10P × 6P (b) 10C × 6C
6 3 6 3
(a) NAAGI (b) NAGAI (c) 10C × 4C × 9! (d) 10P × 4P
6 3 6 3
(c) NAAIG (d) NAIAG 53. The number of 5 digit numbers that can be made using the
44. In a chess tournament, where the participants were to play digits 1 and 2 and in which at least one digit is different, is
one game with another, two chess players fell ill, having (a) 30 (b) 31
played 3 games each. If the total number of games played is (c) 32 (d) None of these

WWW.SARKARIPOST.IN
84, the number of participants at the beginning was
54. In how many ways can 10 books on English and 8 books on
(a) 15 (b) 16 physics be placed in a row on a shelf so that two books on
(c) 20 (d) 21 physics may not be together?
45. All the words that can be formed using alphabets A, H, L, U (a) 160 (b) 165
and R are written as in a dictionary (no alphabet is repeated). (c) 170 (d) 180
Rank of the word RAHUL is
55. In how many ways can 2310 be expressed as a product of 3
(a) 71 (b) 72 factors?
(c) 73 (d) 74 (a) 40 (b) 41
46. In how many different ways can a cube be painted if each (c) 42 (d) 43
face has to be painted either red or blue?
Directions for Qs. 56–58:
(a) 20 (b) 16
Different words are formed with the help of letters of the word
(c) 12 (d) 10 SIGNATURE. Find the number of words in which
47. How many new words are possible from the letters of the 56. vowels always occupy even places.
word PERMUTATION? 4C 5C
(a) 4 × 4! × 5! (b) 4 . (5!)2
(a) 11!/2! (b) (11!/2!) – 1 6C
(c) 4 . (4!)2 (d) None of these
(c) 11! – 1 (d) None of these
57. letters S, G and N are always together
48. There are five boys and three girls who are sitting together
(a) 8! × 2! (b) 7! × 3!
to discuss a management problem at a round table. In how
(c) 6! × 4! (d) None of these
many ways can they sit around the table so that no two girls
are together? 58. no two letters from N, T and R come together
(a) 7C × 3! × 4! (b) 7C × 2! × 5!
(a) 1220 (b) 1400 4 2
(c) 7C × 3! × 6! (d) None of these
(c) 1420 (d) 1440 6

49. Seven different objects must be divided among three people. 59. The number of ways in which ten candidates A1, A2, ...., A10
In how many ways can this be done if at least one of them can be ranked so that A1 is always above A2 is
gets exactly 1 object? 10 !
(a) (b) 10 !
(a) 2484 (b) 1218 2
(c) 729 (d) None of these 8!
(c) 9 ! (d)
50. How many 6-digit numbers have all three digits either all 2
60. A class photograph has to be taken. The front row consists
odd or all even?
of 6 girls who are sitting. 20 boys are standing behind. The
(a) 31,250 (b) 28,125 two corner positions are reserved for the 2 tallest boys. In
(c) 15,625 (d) None of these how many ways can the students be arranged?
51. There are three piles of identical red, blue and green balls (a) 18! × 1440 (b) 6! × 1440
and each pile contains at least 10 balls. The number of ways (c) 18! × 2! × 1440 (d) None of these

PDF Download FROM >> WWW.SARKARIPOST.IN Join >> https://www.facebook.com/Sarkaripost.in/


WWW.SARKARIPOST.IN
Permutations and Combinations 587

61. A,B,C and D are four towns any three of which are non- (a) 5 (b) 7
colinear. Then the number of ways to construct three roads (c) 9 (d) None of these
each joining a pair of towns so that the roads do not form a 69. In the given figure, what is the maximum number of different
triangle is ways in which 8 identical balls can be placed in the small
(a) 7 (b) 8 triangles 1, 2, 3 and 4 such that each triangle contains at
(c) 9 (d) More than 9 least one ball?
62. There are 10 points on a line and 11 points on another line,
which are parallel to each other. How many triangles can
be drawn taking the vertices on any of the line? 1

(a) 1,050 (b) 2,550 2


3 4
(c) 150 (d) 1,045
63. How many motor vehicle registration number plates can be
(a) 32 (b) 35
formed with the digits 1, 2, 3, 4, 5 (No digits being repeated)

WWW.SARKARIPOST.IN
if it is given that registration number can have 1 to 5 digits? (c) 44 (d) 56

(a) 100 (b) 120 70. There are 5 different Jeffrey Archer books, 3 different Sidney
Sheldon books and 6 different John Grisham books. The
(c) 325 (d) 205
number of ways in which at least one book can be given
64. Find the number of 6-digit numbers that can be found using away is
the digits 1, 2, 3, 4, 5, 6 once such that the 6-digit number is
(a) 210 – 1 (b) 211 –1
divisible by its unit digit. (The unit digit is not 1).
(c) 212 – 1 (d) 214 – 1
(a) 620 (b) 456
71. The number of natural numbers of two or more than two
(c) 520 (d) 528
digits in which digits from left to right are in increasing
65. How many different 9-digit numbers can be formed from order is
the number 223355888 by rearranging its digits so that the
(a) 127 (b) 128
odd digits occupy even positions?
(c) 502 (d) 512
(a) 120 (b) 9!(2!)3.3!
72. How many natural numbers not more than 4300 can be
(c) (4!)(2!)3.(3!) (d) None of these
formed with the digits 0, 1, 2, 3, 4 (if repetitions are
66. Boxes numbered 1, 2, 3, 4 and 5 are kept in a row and they allowed)?
are to be filled with either a red or a blue ball, such that no
(a) 574 (b) 570
two adjacent boxes can be filled with blue balls. Then how
(c) 575 (d) 569
many different arrangements are possible, given that
all balls of a given colour are exactly identical in all 73. The sides of a triangle have 4, 5 and 6 interior points marked
respects? on them respectively. The total number of triangles that can
be formed using any of these points
(a) 8 (b) 10
(c) 15 (d) 22 (a) 371 (b) 415
(c) 286 (d) 421
67. Five persons A, B, C, D and E along with their wives are
seated around a round table such that no two men are 74. Total number of ways in which six '+' and four '–' sings can
adjacent to each other. The wives are three places away from be arranged in a line such that no two '–' sings occur together,
their husbands. Mrs. C is on the left of Mr. A, Mrs. E is two is
places to the right of Mrs. B. Then, who is on the right hand (a) 35 (b) 18
side of Mr. A? (c) 15 (d) 42
(a) Mrs.B (b) Mrs.D 75. Between two junction stations A and B, there are 12
(c) Mrs. E (d) Either Mrs B or Mrs D intermediate stations. The number of ways in which a train
can be made to stop at 4 of these stations so that no two of
68. N persons stand on the circumference of a circle at distinct
these halting stations are consecutive, is
points. Each possible pair of persons, not standing next to
(a) 8C (b) 9C
each other, sings a two-minute song one pair after the other. 4 4
(c) 12C –4 (d) None of these
If the total time taken for singing is 28 minutes, what is N? 4

PDF Download FROM >> WWW.SARKARIPOST.IN Join >> https://www.facebook.com/Sarkaripost.in/


WWW.SARKARIPOST.IN
588 Quantitative Aptitude

76. In a unique hockey series between India & Pakistan, they 79. In how many ways can 5 prizes be distributed among 4
decide to play till a team wins 5 matches . The number of boys when every boy can take one or more prizes?
ways in which the series can be won by India, if no match (a) 1024 (b) 625
ends in a draw is (c) 120 (d) 600
(a) 126 (b) 252 80. A man invites 4 men and 4 women to a party. In how many
ways can they sit at a round table so that no two men are
(c) 225 (d) None of those together?
77. The different letters of the alphabet are given, Out of which (a) 24 (b) 6
five letter words are formed. Then the numbers of words in
(c) 144 (d) 120
which at least one letter is repeated is
81. Three dice are rolled. The number of possible outcomes in
(a) 50400 (b) 840
which at least one die shows 5 is
(c) 30240 (d) 69760
(a) 215 (b) 36
78. With 17 consonants and 5 vowels the number of words of
(c) 125 (d) 91
four letters that can be formed having two different vowels
in the middle and one consonant, repeated or different at 82. There are 10 points in a plane out of which 5 are collinear.

WWW.SARKARIPOST.IN
each end is The number of triangles that can be drawn will be
(a) 5780 (b) 2890 (a) 120 (b) 110
(c) 5440 (d) 2720 (c) 100 (d) 78

PDF Download FROM >> WWW.SARKARIPOST.IN Join >> https://www.facebook.com/Sarkaripost.in/


WWW.SARKARIPOST.IN
Permutations and Combinations 589

Expert Level
1. How many possible values of n will make 13Cn < 13Cn+2? 11. The number of 5 digit numbers of the form xyzyz in which
(a) 4 (b) 3 x < y is
(c) 6 (d) 2 (a) 350 (b) 360
2. The sum of 5 digit numbers in which only odd digits occur (c) 380 (d) 390
without any repetition is 12. Three are n points in a plane, no three being collinear except
(a) 277775 (b) 555550 m of them which are collinear. The number of triangles that
(c) 1111100 (d) None of these can be drawn with their vertices at three of the given points
Directions for Qs. 3–5 : Refer to the following information to is
answer the questions that follow. (a) n – mC3 (b) nC3 – mC3
A number of 4 different digits is formed by using the digits 1, 2, n
(c) C3 – m (d) None of these

WWW.SARKARIPOST.IN
3, 4, 5, 6, 7 in all possible ways without repetition. 13. The number of arrangements of the letters of the word
3. How many of them are greater than 3400? BANANA is which the two ‘N’s do not appear adjacently
(a) 840 (b) 560 is
(c) 480 (d) 120 (a) 40 (b) 60
4. How many of them are exactly divisible by 25? (c) 80 (d) 100
(a) 20 (b) 35 14. Number of integers greater than 7000 and divisible by 5
(c) 40 (d) 50 that can be formed using only the digits 3, 6, 7, 8 and 9, no
digit being repeated, is
5. How many of them are exactly divisible by 4?
(a) 46 (b) 48
(a) 150 (b) 160
(c) 72 (d) 42
(c) 120 (d) 200
15. There are 10 points in a plane out of which 5 are collinear.
6. A person has 12 friends out of which 7 are relatives. In how
The number of straight lines than can be drawn by joining
many ways can he invite 6 friends such that at least 4 of
these points will be
them are relatives?
(a) 35 (b) 36
(a) 462 (b) 562
(c) 45 (d) 24
(c) 450 (d) 400
16. The streets of a city are arranged like the lines of a chess
7. Messages are conveyed by arranging 4 white , 1 blue and 3
board . There are m streets running North to South and 'n'
red flags on a pole . Flags of the same colour are alike . If a
streets running East to West . The number of ways in which
message is transmitted by the order in which the colours
a man can travel from NW to SE corner going the shortest
are arranged then the total number of messages that can be
possible distance is
transmitted if exactly 6 flags are used is
(a) 45 (b) 65 (a) m2 n2 (b) (m 1) 2 . ( n 1) 2
(c) 125 (d) 185
8. If the letters of the word ‘PARKAR’ are written down in all ( m n) ! (m n 2) !
possible manner as they are in a dictionary, then the rank of (c) (d)
m! . n! (m 1) ! . ( n 1) !
the word ‘PARKAR’ is 17. In a conference 10 speakers are present . If S1 wants to speak
(a) 98 (b) 99 before S2 and S2 wants to speak after S3 , then the number
(c) 100 (d) 101 of ways all the 10 speakers can give their speeches with the
9. The number of integers satisfying the inequality above restriction if the remaining seven speakers have no
objection to speak at any number is
n 1 n 1
C3 C2 100 is (a) 10C3 (b) 10P8
(a) nine (b) eight (c) 10P3 (d) 10!/3
(c) five (d) None of these 18. Six persons A, B, C, D, E and F are to be seated at a circular
10. The number of triangles whose vertices are at the vertices table . The number of ways this can be done if A must have
of an octagon but none of whose sides happen to come from either B or C on his right and B must have either C or D on
the sides of the octagon is his right is
(a) 24 (b) 52 (a) 36 (b) 12
(c) 48 (d) 16 (c) 24 (d) 18

PDF Download FROM >> WWW.SARKARIPOST.IN Join >> https://www.facebook.com/Sarkaripost.in/


WWW.SARKARIPOST.IN
590 Quantitative Aptitude

19. To fill up 12 vacancies, there are 25 candidates of which 5 should be no identical series side by side and that the students
are from SC. If 3 of these vacancies are reserved for SC sitting one behind the other should have the same series?
candidates while the remaining are open to all then the (a) 2 × 12C6 × (6!)2 (b) 6!× 6!
number of ways in which the selection can be made is (c) 7! × 7! (d) None of these
(a) 5C3 × 15C9 (b) 5C3 × 22C9 29. A, B, C D, . ................X, Y, Z are the players who participated
(c) 5C3 × 20C9 (d) None of these in a tournament. Everyone played with every other player
20. The number of non negative integral solution of the equation, exactly once. A win scores 2 points, a draw scores 1 point
x + y + 3z = 33 is and a loss scores 0 points. None of the matches ended in a
draw. No two players scored the same score. At the end of
(a) 120 (b) 135
the tournament, the ranking list is published which is in
(c) 210 (d) 520 accordance with the alphabetical order. Then
21. On a plane there are 37 straight lines, of which 13 pass (a) M wins over N
through the point A and 11 pass through the point B. Besides,
(b) N wins over M
no three lines pass through one point, no lines passes through
(c) M does not play with N
both points A and B, and no two are parallel, then the number
(d) None of these

WWW.SARKARIPOST.IN
of intersection points the lines have is equal to
(a) 535 (b) 601 30. Out of 2n+1 students, n students have to be given the
scholarships. The number of ways in which at least one
(c) 728 (d) 963
student can be given the scholarship is 63. What is the
22. How many numbers lying between 3000 and 4000 and number of students receiveing the scholarship?
which are divisible by 5 can be made with the digits 3, 4, 5, (a) 5 (b) 7
6, 7 and 8? (Digits are not to be repeated in any number.)
(c) 3 (d) 9
(a) 11 (b) 12
31. A graph may be defined as a set of points connected by
(c) 13 (d) 14 lines called edges. Every edge connects a pair of points.
23. In how many ways can n women be seated in a row so that Thus, a triangle is a graph with 3 edges and 3 points. The
a particular women will not be next to each other? degree of a point is the number of edges connected to it.
(a) (n – 2) × (n – 1)! (b) (n – 2) × (n – 2)! For example, a triangle is a graph with three points of degree
(c) (n – 1) × (n – 1)! (d) None of these 2 each. Consider a graph with 12 points. It is possible to
reach any point from any other point through a sequence of
24. The number of ways in which n distinct objects can be put
edges. The number of edges, e, in the graph must satisfy the
into two different boxes so that no box remains empty is
condition
(a) 2n – 1 (b) n2 – 1
(a) 11 e 66 (b) 10 e 66
n
(c) 2 – 2 (d) n2 – 2
(c) 11 e 65 (d) 0 e 11
25. The number of words of four letters containing equal number
of vowels and consonants, repetition being allowed, is 32. There are three coplanar parallel lines. If any p points are
taken on each of the lines, then find the maximum number
(a) 1052 (b) 210 × 243
of triangles with the vertices of these points.
(c) 105 × 243 (d) None of these
(a) p2 (4p – 3) (b) p3 (4p – 3)
26. Let S be the set of five-digit numbers formed by the digits (c) p (4p – 3) (d) p3
1, 2, 3, 4 and 5, using each digit exactly once such that
33. There are three books on table A which has to be moved to
exactly two odd positions are occupied by odd digits. What
table B. The order of the book on Table A was 1, 2, 3, with
is the sum of the digits in the rightmost position of the
book 1 at the bottom. The order of the book on table B
numbers in S?
should be with book 2 on top and book 1 on bottom. Note
(a) 228 (b) 216 that you can pick up the books in the order they have been
(c) 294 (d) 192 arranged. You can’t remove the books from the middle of
27. m distinct animals of a circus have to be placed in m cages, the stack. In how many minimum steps can we place the
one in cach cage. If n (< m) cages are too small to books on table B in the required order?
accommodate p (n < p < m) animals, then the number of (a) 1 (b) 2
ways of putting the animals into cages are (c) 3 (d) 4
(a) (m – nPp) m – pPm – p (b) m – nCp 34. In how many ways is it possible to choose a white square
(c) ( m – n Cp) ( m – p Cm – p ) (d) None of these and a black square on a chess board so that the squares
28. Two series of a question booklets for an aptitude test are to must not lie in the same row or column?
be given to twelve students. In how many ways can the (a) 56 (b) 896
students be placed in two rows of six each so that there (c) 60 (d) 768

PDF Download FROM >> WWW.SARKARIPOST.IN Join >> https://www.facebook.com/Sarkaripost.in/


WWW.SARKARIPOST.IN
Permutations and Combinations 591

35. There are 12 towns grouped into four zones with three towns
per zone. It is intended to connect the towns with telephone
lines such that every two towns are connected with three
direct lines if they belong to the same zone, and with only
one direct line otherwise. How many direct telephone lines
are required?
(a) 72 (b) 90
(c) 96 (d) 144
(a) 38 (b) 28
36. There are 6 tasks and 6 persons. Task I cannot be assigned (c) 16 (d) 14
either to person 1 or to person 2; task 2 must be assigned to
43. Two variants of the CAT paper are to be given to twelve
either person 3 or person 4. Every person is to be assigned
students. In how many ways can the students be placed in
one task. In how many ways can the assignment be done?
two rows of six each so that there should be no identical
(a) 144 (b) 180 variants side by side and that the students sitting one behind
(c) 192 (d) 360 the other should have the same variant?

WWW.SARKARIPOST.IN
37. The letters of the word ALLAHABAD are rearranged to (a) 2 × 12C6 × (6!)2 (b) 6! × 6!
form new words and put in a dictionary. If the dictionary (c) 7! × 7! (d) None of these
has only these words and one word on every page in
44. The straight lines S1, S2, S3 are in a parallel and lie in the
alphabetical order then what is the page number on which
same plane. A total number of A points on S1; B points on S2
the word LABADALAH comes? and C points on S3 are used to produce triangles. What is
(a) 6089 (b) 6088 the maximum number of triangles formed?
(c) 6087 (d) 6086 (a) A + B + CC3 – AC3 – BC3 – CC3 + 1
38. How many natural numbers smaller than 10,000 are there (b) A + B + CC3
in the decimal notation of which all the digits are different? (c) A + B + CC3 + 1
(a) 2682 (b) 4474 (d) (A + B + CC3 – AC3 – BC3 – CC3)
(c) 5274 (d) 1448 45. If x, y and z are whole numbers such that x.y, then how
39. Sameer has to make a telephone call to his friend Harish many solutions are possible for the equation x + y + z
Unfortunately he does not remember the 7- digit phone = 36?
number. But he remembers that the first 3 digits are 635 or (a) 361 (b) 323
674, the number is odd and there is exactly one 9 in the (c) 382 (d) 342
number. The minimum number of trials that Sameer has to
46. The HCF of three natural numbers x, y and z is 13. If the
make to be successful is
sum of x, y and z is 117, then how many ordered triplets
(a) 10, 000 (b) 3,402 (x, y, z) exist?
(c) 3,200 (d) 5,000 (a) 28 (b) 27
40. 10 straight lines, no two of which are parallel and no three (c) 54 (d) 55
of which pass through any common point, are drawn on a
47. The crew of an 8-member rowing team is to be chosen from
plane. The total number of regions (including finite and
12 men (M1, M2, ...., M12) and 8 women (W1, W2, ...., W8).
infinite regions) into which the plane would be divided by
There have to be 4 people on each side with at least one
the lines is woman on each side. Further it is also known that on the
(a) 56 (b) 255 right side of the boat (while going forward) W1 and M1
(c) 1024 (d) not unique must be selected while on the left side of the boat M2, M3
41. Suppose you have a currency, named Miso, in three and M10 must be selected. What is the number of ways in
denominations: 1 Miso, 10 Misos and 50 Misos. In how which the rowing team can be arranged?
many ways can you pay a bill of 107 Misos? (a) 1368 × 4! × 4! (b) 1200 × 4! × 4!
(a) 19 (b) 17 (c) 1120 × 4! × 4! (d) 728 × 4! × 4!
(c) 16 (d) 18 48. A boy plays a mathematical game where he tries to write
42. Each of 8 identical balls is to be placed in the squares shown the number 1998 into the sum of 2 or more consecutive
in the figure given in a horizontal direction such that one positive even numbers (e.g., 1998 = 998 + 1000). In how
horizontal row contains 6 balls and the other horizontal row many different ways can he do so?
contains 2 balls. In how many maximum different ways can (a) 5 (b) 6
this be done? (c) 7 (d) 8

PDF Download FROM >> WWW.SARKARIPOST.IN Join >> https://www.facebook.com/Sarkaripost.in/


WWW.SARKARIPOST.IN
592 Quantitative Aptitude

Test Yourself

1. How many three-digit number can be generated from 1, 2, 10. There are 10 points on a straight line AB and 8 on another
3, 4, 5, 6, 7, 8, 9, such that the digits are in ascending order? straight line, AC none of them being A. How many triangles
(a) 80 (b) 81 can be formed with these points as vertices?
(c) 83 (d) 84 (a) 720 (b) 640
2. How many ways are there to arrange the letters in the word (c) 816 (d) None of these
GARDEN with vowels in alphabetical order 11. How many numbers can be formed from 1,2,3,4 and 5
(a) 480 (b) 240 (without repetition), when the digit at the units place must
(c) 360 (d) 120 be greater than that in the tenth place?
(a) 54 (b) 60
3. There are 5 historical moments, 6 gardens and 7 shopping

WWW.SARKARIPOST.IN
malls in the city. In how many ways a tourist can visit the 5!
city, if he visits at least one shopping mall? (c) (d) 2 × 4!
3
5 6 7
(a) 2 .2 . (2 – 1) 4 6
(b) 2 .2 (2 – 1)7
12. The figure below shows the network connecting cities A, B,
(c) 25.26(26 –1) (d) None of these
C, D, E and F. The arrows indicate permissible direction of
4. In how many ways 7 men and 7 women can sit on a round
travel. What is the number of distinct paths from A to F?
table such that no two women sit together ?
(a) (7 !)2 (b) 7!×6! B C
(c) (6 !)2 (d) 7 !
5. On a triangle ABC, on the side AB, 5 points are marked, 6
points are marked on the side BC and 3 points are marked A F
on the side AC (none of the points being the vertex of the
triangle). How many triangles can be made by using these
points? D E
(a) 90 (b) 333
(a) 9 (b) 10
(c) 328 (d) None of these
(c) 11 (d) None of these
6. How many 6-digit numbers have at least 1 even digit?
13. If there are 10 positive real numbers n1 < n2 < n3 ...... < n10.
(a) 884375 (b) 3600
How many triplets of these numbers (n1, n2, n3), (n2, n3,
(c) 880775 (d) 15624
n4), ..... can be generated such that in each triplet the first
7. There is a 7-digit telephone number with all different digits.
number is always less than the second number and the
If the digit at extreme right and extreme left are 5 and 6
second number is always less than the third number?
respectively, find how many such telephone numbers are
(a) 45 (b) 90
possible. (c) 120 (d) 180
(a) 120 (c) 1,00,000 14. In how many ways can the letters of the English alphabet
(c) 6720 (d) None of these be arranged so that there are seven letters between the letters
8. The numbers of ways in which the letters of the word A and B ?
'VOWEL' can be arranged so that the letters O, E occupy (a) 31!.2! (b) 24P7.18!.2
only even places is (c) 36.24! (d) None of these
(a) 12 (b) 24 15. In a chess competition involving some boys and girls of a
school, every student had to play exactly one game with
(c) 18 (d) 36
every other student. It was found that in 45 games both the
9. A bouquet has to be formed from 18 different flowers so players were girls and in 190 games both were boys. The
that it should contain not less than three flowers. How many number of games in which one player was a boy and the
ways are there of doing this in? other was a girl is
(a) 5,24,288 (b) 2,62,144 (a) 200 (b) 216
(c) 2,61,972 (d) None of these (c) 235 (d) 256

PDF Download FROM >> WWW.SARKARIPOST.IN Join >> https://www.facebook.com/Sarkaripost.in/


WWW.SARKARIPOST.IN
Permutations and Combinations 593

Hints & Solutions


10. (b) If number of persons be n, then total number of hand-
Foundation Level shaken = nC2 = 66
1. (c) n (n–1) =132 (n + 11) ( n = 12) = 0
n =12 ( n – 11 1)
Last place can be filled by 0, 2, 4
11. (b) There are 6 letters in the word BHARAT, 2 of them are
So total sum = 5 × 6 × 6 (0 + 2 + 4) + 5 × 6 × 3 × 10 (0 identical.
+ 1 + 2 + 3 + 4 + 5) + 5 × 6 × 3 × 100 (0 + 1 + 2 + 3 + Hence total number of words with these letter = 360
4 + 5) + 6 × 6 × 3 × 1000 (0 + 1 + 2 + 3 + 4 + 5)
Also the number of words in which B and H come
= 180 × 6 + 900 × 15 + 9000 × 15 + 10800 × 15 together = 120
= 1080 + 13500 + 135000 + 1620000 = 1769580 The required number of words = 360 – 120 = 240
2. (a) There are 8 letters in the word EQUATION.

WWW.SARKARIPOST.IN
12. (a) The required number of selections
= 3C1 × 4C1 × 2C1 (6C3 + 6C2 + 6C0) = 42 × 4!
A/E/I/O/U
13. (d) MACHINE has 4 consonants and 3 vowels.
5 in ways in7P7 = 7! = 5040 The vowels can be placed in position no. 1, 3, 5, 7
Reqd. no. = 5 × 5040 = 25200 Total number of ways possible = 4P3 = 24.
3. (a) There are 9 letters in the given word in which two T's, For each of these 24 ways the 4 consonants can occupy
two M's and two E's are identical. Hence the required the other 4 places in 4P4 ways
9! 9! Total = 24 × 24 = 576
number of words = 2!2!2!
(2!)3 14. (b) We have, nPr = nPr + 1
4. (c) Given, 10Pr = 720 n! n! 1
1
(n r )! (n r 1)! (n r )
10!
or n – r = 1 ...(1)
10 r ! = 720
Also, nCr = nCr – 1 r + r – 1 = n
10 × 9 × 8 × . . . to r factors = 720 = 10 × 9 × 8 2r – n = 1 ...(2)
r=3 Solving (1) and (2), we get r = 2 and n = 3
5. (b) 4 apples, 5 mangoes and 6 oranges 15. (c) nPr = 720nCr
coeff. of x4 in (1 + x + x2 + x3 + x4) 3
n! 720( n !)
= coeff. of x4 in (1 – x)–3 = 6C2 = 15 or
(n r )! (n r )!r !
12! r! = 720 = 1 × 2 × 3× 4 × 5 × 6!
6. (b) or r = 6
5!4!3!
16!
7. (a) Considering the two vowels E and A as one letter, the 16. (a) Total number of ways = 16 C11 = = 4368.
total no. of letters in the word ‘EXTRA’ is 4 which can 11! × 5!
16 ×15 ×14 ×13×12
be arranged in 4P4, i.e. 4! ways and the two vowels = = 4368.
5 × 4 × 3 × 2 ×1
can be arranged among themselves in 2! ways.
39 39 39 39
reqd. no. = 4! × 2! = 4 × 3 × 2 × 1 × 2 × 1 = 48 17. (b) C3 r 1 Cr 2 Cr 2 1 C3r
8. (a) A committee of 5 out of 6 + 4= 10 can be made in 39 39 39 39
10C = 252 ways. C3r 1 C3r Cr 2 1 Cr 2
5
If no woman is to be included, then number of ways 40 40
C3r Cr 2
= 5C5 = 6
the required number = 252 – 6 = 246 r2 3r or r 2 40 3r

9. (d) 4 digit number r 0, 3 or – 8, 5


3 4 3 2 = 72,
3 and 5 are the values as the given equation is not
5 digit number = 120
defined by r = 0 and r = –8. Hence, the number of
Total = 192
values of r is 2.

PDF Download FROM >> WWW.SARKARIPOST.IN Join >> https://www.facebook.com/Sarkaripost.in/


WWW.SARKARIPOST.IN
594 Quantitative Aptitude

18. (c) We have, 75600 = 24 . 33 . 52 . 7 33. (d) For each question we have 3 choices of answering the
The total number of ways of selecting some or all out question (2 internal choices + 1 non-attempt).
of four 2's, three 3's, two 5's and one 7's is Thus, there are a total of 315 ways of answering the
(4 + 1) (3 + 1) (2 + 1) (1 + 1) – 1 = 119 question paper. Out of this there is exactly one way in
which the student does not answer any question.
But this includes the given number itself. Therefore,
Thus there are a total of 315 – 1 ways in which at least
the required number of proper factors is 118.
one question is answered.
19. (c) Required number of ways = ways of selecting 4 34. (c) The digits are 1, 6, 7, 8, 7, 6, 1. In this seven-digit no.
objects out of 6 given objects there are four odd places and three even places
6 5 OEOEOEO. The four odd digits 1, 7, 7, 1 can be
= 6C4 = 15 arranged in four odd places in [4!/2!×2] = 6 ways [as 1
2
and 7 are both occurring twice].
20. (d) First of all we will prime factorize 8064.
The even digits 6, 8, 6 can be arranged in three even
8064 = 2 × 4032 = 22 × 2016 = 23 × 1008 = 24 × 504 places in 3!/2! = 3 ways.
= 25 × 252 = 26 × 126 = 27 × 63 Total no. of ways = 6 × 3 = 18

WWW.SARKARIPOST.IN
= 27 × 32 × 71 35. (b) With one green ball there would be six ways of doing
Required no. of ways = (7 + 1) (2 + 1) .1 this. With 2 green balls 5 ways, with 3 green balls 4
= 8 × 3 = 24 ways, with 4 green balls 3 ways, with 5 green balls 2
ways and with 6 green balls 1 way. So a total of 1 + 2
21. (c) Two particular girls can be arranged in 2! ways and + 3 + 4 + 5 + 6 = 21 way.
remaining 10 girls can be arranged in 10! ways.
36. (b) For each selection there are 3 ways of doing it. Thus,
Required no. of ways = 2! × 10! there are a total of 3 × 3 × 3 × 3 × 3 = 243.
22. (c) Required no. of the ways = 6C3 × 4C2 = 20 × 6 = 120 37. (d) First arrange the two sisters around a circle in such a
5! way that there will be one seat vacant between them.
23. (b) Required number of ways = 10 . [This can be done in 2! ways since the arrangement of
2!3!
the sisters is not circular.]
24. (b) Selection of 2 members out of 11 has 11C2 number of
ways Then, the other 18 people can be arranged on 18 seats
11C = 55 in 18! ways.
2 38. (c) Let the total number of employees in the company be n.
25. (b) From each railway station, there are 19 different tickets
to be issued. There are 20 railway station n n(n 1)
Total number of gifts = C2 61
So, total number of tickets = 20 × 19 = 380. 2
26. (d) Since 32P6 = k 32C6 n 2 n 132 0 or (n 11)(n 12) = 0
32! 32! or n = 12 [– 11 is rejected]
k.
(32 6)! 6!(32 6)! 39. (a) In the letters of the word ALLAHABAD there is only
k = 6! = 720 1 vowel available for selection (A). Note that the fact
27. (c) Before 1000 there are one digit, two digits and three that A is available 4 times has no impact on this fact.
digits numbers. Also, there are 4 consonants available – viz. L, H, B
and D. Thus, the number of ways of selecting a vowel
Numbers of times 3 appear in one digit number = 20×9
and a consonant would be 1 × 4C1 = 4.
Number of times 3 appear in two digit numbers = 11×9 40. (a) Choose 1 person for the single room & from the
Number of times 3 appear in three digit numbers = 21 remaining choose 2 people for the double room & from
Hence total number of times the digit 3 appear while the remaining choose 4 people for the 4 persons room
writing the integers from 1 to 1000 7C × 6C × 4C .
1 2 4
= 180 + 99 + 21 = 300 41. (b) 10P = 720
3
28. (b) 24 – 1 = 15 sums of money can be formed. 42. (a) Ist book can be given to any of the five students.
29. (b) An IITian can make it to IIMs in 2 ways, while a CA Similarly other six books also have 5 choices. Hence
can make it through in 3 ways. Required ratio is 2 : 3. the total number of ways is 57.
30. (a) For a straight line we just need to select 2 points out of 43. (c) Total possible arrangements = 13P13 = 13!
the 8 points available. 8C2 would be the number of Total number in which f and g are together
ways of doing this. = 2 × 12P12 = 2 × 12!
31. (b) 3C × 4C × 6C = 72 44. (a) Order of vowels of fixed
1 1 1
32. (c) At r = 7, the value becomes 6!
required number of ways are
(28!/14! × 14!) /(24!/10! × 14!) 225 : 11 2!

PDF Download FROM >> WWW.SARKARIPOST.IN Join >> https://www.facebook.com/Sarkaripost.in/


WWW.SARKARIPOST.IN
Permutations and Combinations 595

45. (c) Selection of two husbands = 9C2 56. (b) Total number of ways
Selection of two wifes whose husbands are not chosen = 71 51 21 3!
yet = 7C2 57. (b) Let the vice-chairman and the chairman from 1 unit
Total number of ways to form two teams along with the eight directors, we now have to arrange
= 9C2 . 7C2 . 2 ! = 1512 9 different units in a circle.
46. (a) First arrange 21 white balls in a row. This can be done This can be done in 8! ways.
in 1 way (Since they are identical). Now there are 22 At the same time, the vice-chairman & the chairman
place for the 19 black balls and so the place can be can be arranged in two different ways. Therefore, the
filled in total number of ways = 2 × 8!.
58. (b) We need to think of this as: Number with two sixes or
22C
22!
19 ways = ways numbers with one six or number with no six.
3!.9!
0, 1, 2, 3, 4, 5, 6
22C
22 21 20 Number with 2 sixes:
or 3 = = 1540 5C × 3!/2! = 15
2 3 Number ending n zero 1

WWW.SARKARIPOST.IN
Numbers ending in 5 and
47. (b) Number of parallelograms = 5C2 × 4C2 = 60. 5C × 2! = 10
(a) Starting with 6 1
48. (a) A couple and 6 guests can be arranged in (7 – 1) ways. 4
But in two people forming the couple can be arranged (b) Not starting with 6 C1 (as zero is not allowed) = 4
among themselves in 2! ways. Number with 1 six or no sixes.
Number ending in 0 6C × 3! = 120
the required number of ways = 6! × 2! = 1440 3
Number ending in 5 5 C1 × 5C2 × 2! = 100
49. (b) 6! ways, O fixed 1st and E fixed in last.
50. (a) For the number to be divisible by 4, the last two digits Thus a total of 249 numbers.
must be any of 12, 24, 16, 64, 32, 36, 56 and 52. The 59. (b) For drawing a circle we need 3 non collinear points.
last two digit places can be filled in 8 ways. Remaining This can be done in:
3 places in 4P3 ways. Hence no. of 5 digit nos. which 3C + 3C × 7C + 3C × 7C = 1 + 21 + 63 = 85.
3 2 1 1 2
are divisible by 4 are 24 × 8 = 192.
51. (d) The volter can cast one or two or three votes. So total Standard Level
number of ways in which he can cast his vote 1. (c) (i) Miss C is taken
= 5C1 + 5C2 + 5C3 = 5 + 10 + 10 = 25 (1) B included A excluded 4C . 4C = 24
1 2
52. (c) Given : 2n+1Pn–l : 2n–1Pn = 3/5 4 5
(2) B excluded C1 . C3 = 40
(2n 1)! (2n 1)! 3 (ii) Miss C is not taken
(2n 1 n 1)! (n 2) ( n 1)! 5 B does not comes ; 4C2 . 5C3 = 60 Total = 124
2. (d) Total seats = 5 + 6 = 11.
(2n 1) 2n (2n 1)! (n 1)! 3
Arrangement will be : W M W M W M W M W M W
(n 2) (n 1) n (n 1)! (2 n 1)! 5
Total possible arrangements will be :
6P × 5P = 86400.
2 (2n 1) 3 6 5
5 (4n + 2) = 3 (n2 + 3n + 2) 3. (b) 8 men can sit in a row in 8P8 ways. Then for the 6
(n 2) (n 1) 5
20n + 10 = 3n2 + 9n + 6 3n2 – 11n – 4 = 0 women, there are 9 seats to sit
(3n + 1) (n – 4) = 0 n=4 the women can sit in 9P6 ways
53. (b) The total number of two factor products = 200C2. The total number of ways = 8P8 . 9P6
number of numbers from 1 to 200 which are not 4. (a) There are 13 letters in the word INTERNATIONAL ,
multiples of 5 is 160. Therefore the total number of of which N occurs thrice, each of I, T and A occurs
two factor products which are not multiple of 5 is 160C2. twice and the rest are different.
Hence, the required number of factors which are
multiples of 5 = 200C2 – 160C2 = 7180. 13!
Reqd. no. = 3! 2! 2! 2!
54. (a) 15C + 15C + 15C + 15C3 + 15C4 + 15C5
0 1 2
= 1 + 15 + 105 + 455 + 1365 + 3003 = 4944 13 12 11 10 9 8 7 6 5 4 3 2
55. (d) Since every question can be attempted in 4 ways and =
6 2 2 2
each question has only one correct answer, hence
number of required ways = ( 4 × 4 × 4) = 1 = 63 = 13 × 11 × 10 × 9 × 8 × 7 × 6 × 5 × 3 × 2 = 129729600

PDF Download FROM >> WWW.SARKARIPOST.IN Join >> https://www.facebook.com/Sarkaripost.in/


WWW.SARKARIPOST.IN
596 Quantitative Aptitude

5. (c) Total number of ways of selecting any number of fruits


= coeff of x8 in x 2 (1 x x 2 .... x5 )3
= 11 × 6 × 3 × 2 × 2 × 2 = 1584
Number of ways in which no fruit is selected = 1 3
Number of ways in which only one fruit is selected 1 x6
coeff of x5 in
=6 1 x
Number of ways in which two fruit are selected
= 6C2 + 3 = 18 = coeff of x5 in (1 x) 3

Number of ways in which at least three fruits are


selected = 1584 – (1 + 6 + 18) = 1559. = coeff of x5 in (1 3
C1 x 4
C2 x 2 ...) = 7C5 = 21
6. (a) The number of 4 persons including A, B = 6C2 13. (c) X - X - X - X - X. The four digits 3, 3, 5,5 can be arranged
Considering these four as a group, number of
4!
arrangements with the other four = 5! at (–) places in = 6 ways.
2!2!
But in each group the number of arrangements
= 2! × 2! The five digits 2, 2, 8, 8, 8 can be arranged at (X) places

WWW.SARKARIPOST.IN
The required number of ways = 6C2 × 5! × 2! × 2! 5!
in ways = 10 ways
7. (b) There are ten digits 0, 1, 2, . ........., 9. Permutations of 2!3!
these digits taken eight at a time = 10P8 which include Total no. of arrangements = 6 10 60 ways
permutations having 0 at the first. When 0 is fixed at
14. (c) Number of elements in the sample space
the first place, then number of such permutations
= 6 × 6 = 36
10! 9! 9!9
= 9P7 – 9P7 = The sample space is given by
2 2 2
(1, 1), (1, 2), (1, 3), (1, 4), (1, 5), (1, 6)
8. (b) Consider the vowels to be one entity
(2, 1), (2, 2), (2, 3), (2, 4), (2, 5), (2, 6)
(a, e, i, o), v, l, d, c, t, r, y have to be permuted and
·······························
the 4 vowels can also permute in the set
Total number of arrangements possible ·······························
= P8 × 4P4 = 967680
8 ·······························
9. (d) From total 13 members 5 can be select as 13C5 (6, 1), (6, 2), (6, 3), (6, 4), (6, 5), (6, 6)
For at least one girl in the committee number of ways 15. (d) Two possibilities are there :
are 13C5 – 6C1 (i) Chemistry part I is available in 8 books with
10. (c) Since each question can be selected in 3 ways, by Chemistry part II.
selecting it or by selecting its alternative or by rejecting or
it. Thus, the total number of ways of dealing with 10
(ii) Chemistry part II is available in 8 books but
given questions is 310 including a way in which we Chemistry part I is not available.
reject all the questions.
Total No. of ways
Hence, the number of all possible selections is 310 – 1.
=1 6 7
11. (a) No. of ways = coeff. of x13 in (x2 + x3 + x4 +.....+ x9)3 C1 C3
[Max coin one can get is 9] 7 6 5
= 6 6 35 41
= coeff. of x13 in x6 (1 + x + x2 +.....+ x 7 )3 3 2
16. (a) There are 8! arrangements of the beads on the bracelet,
8 3 but half of these can be obtained from the other half
1 x
= coeff. of x7 in = coeff. of x7 in (1 – x)–3 simply by turning the bracelet over.
1 x
1
Hence there are (8!) = 20160 different bracelets.
= coeff. of x7 in (1 + 3C1 x + 4C2 x2 + ......) = 9C7 = 36 2
12. (b) Required number of ways 17. (c) We have 8 pieces and 8 squares on the chessboard so
we can distribute them in 8! ways but 2 pieces are
= coefficient of x 2 in ( x x 2 ...x 6 )3 identical in three cases so total ways
[ each box can receive minimum 1 and maximum 6 8!
= = 5040.
balls] 2! 2! 2!

PDF Download FROM >> WWW.SARKARIPOST.IN Join >> https://www.facebook.com/Sarkaripost.in/


WWW.SARKARIPOST.IN
Permutations and Combinations 597

18. (c) Number of ways in which 7 persons can stand in the 25. (b) Number of ways of selecting 5 different letters
form of a ring = (7 – 1) ! = 6! = 5C5 =1
19. (c) Given expression is : Number of ways of selecting 2 similar and 3 different
letters = 4C1 × 4C3 = 16
(n 2)! (n 1)!(n 1)! Number of ways of selecting 2 similar letters and 2
x (let)
(n 1)!( n 1)! more similar letters and 1 different letter
= 4C2 × 3C1 = 18
(n 2)(n 1)n(n 1)! (n 1)(n 1)! Number of ways of selecting 3 similar letters and 2
x
(n 1)(n 1)! different letters = 3C1 × 4C1 = 18
Number of ways of selecting 3 similar letters and
= (n + 2)n + 1= n2 + 2n + 1 = (n + 1)2
another 2 other similar letters = 3C1 × 3C1 = 9
Which is a perfect square. Number of ways of selecting 4 similar letters and 1
20. (b) Let total no. of team participated in a championship be different letter = 2C1 × 4C1 = 8
n. Number of ways of selecting 5 similar letters

WWW.SARKARIPOST.IN
Since, every team played one match with each other = 1C1 = 1
team. Total number of ways = 1 + 16 + 18 + 18 + 9 + 8 + 1
n! = 71.
nC = 153 153 26. (d) 1m + 3f = 8C1 × 8C3 = 8 × 56 = 448
2 2!(n – 2)!
2m + 2f = 8C2 × 8C2 = 28 × 28 = 784
n(n – 1)(n – 2)! n(n –1) 3m + 1f = 8C3 × 8C1 = 56 × 8 = 448
153 153
2!(n – 2)! 2 4m + 0f = 8C4 × 8C0 = 70 × 1 = 70
Total = 1750
n(n – 1) = 306
27. (c) Taking all vowels (IEO) as a single letter (since they
n2 – n – 306 = 0
come together) there are six letters among which there
n2 – 18n + 17n – 306 = 0 are two R.
n (n – 18) + 17 (n – 18) = 0 6!
Hence no. of arrangements = × 3! = 2160
n = 18, – 17 2!
n cannot be negative There vowels can be arranged in 3! ways among
themselves, hence multiplied with 3!.
n – 17
28. (d) Assume the 2 given students to be together (i.e. one).
n = 18
Now these are five students.
21. (d) As we know Possible ways of arranging them are = 5! = 120
P(n, r) = r! C (n, r) Now they (two girls) can arrange themselves in 2!
From the question, we have ways.
x = r ! (y) Hence total ways = 120 × 2 = 240
Here r = 31 29. (a) Putting l Englishman in a fixed position, the remaining
6 can be arranged in 6! 720 ways, For each such
x = (31)!. y. arrangement, there are 7 positions for the 7 Americans
22. (c) The number of squares would be and they can be arranged in 7! ways.
12 + 22 + 32 + 42 + 52 + 62 = 91. Total number of arrangements = 7! × 6! = 3628800
23. (d) All arrangements – Arrangements with best and worst 30. (a) Group 6 persons can be divided into 3 equal groups
paper together = 12! – 2! × 11!.
6! P1 P2 P3
24. (b) We just need to select the floors and the people who in ways
2!2!2!3! P4 P6 P6
get down at each floor.
The floors selection can be done in 11C3 ways. say P1P4 ; P2P5 ; P3P6
The people selection is 9C4 × 5C3. Now each elements of a group can be arranged in 3!
ways.
Also, the floors need to be arranged using 3!
6!3! 720
Thus, 11C3 × 9C4 × 5C3 × 3! or 11P3 × 9C4 × 5C3 Total ways = 90
2!2!2!3! 8

PDF Download FROM >> WWW.SARKARIPOST.IN Join >> https://www.facebook.com/Sarkaripost.in/


WWW.SARKARIPOST.IN
598 Quantitative Aptitude

31. (c) 24 = 2.3.4, 2.2.6.4, 1.3.8, 1.2.12, 1.1.24 (as product of If we choose 5 bowlers then we have to choose 5
three positive integers) batsmen
the total number of positive integral solution of there is no majority.
3! 3! Total number of ways = 4 × 6930 = 27720.
xyz = 24 is equal to 3! + + 3! + 3! + , i.e., 30 38. (b) Total no. of persons = 4 + 4 = 8
2! 2!
When there is no restriction they can be seated in a
Any two of the factors in each factorization may be
row in 8! ways. But when all the 4 girls sit together,
negative
we can consider the group of 4 girls as one person.
The total number of integral solution
Therefore, we have only 4 (no. of boys) + 1 = 5 persons,
= 30 + 3 × 30 = 120
who can be arranged in a row in 5! ways.
B1 B2 B3 ........ B9 But the 4 girls can be arranged among themselves in
32. (c)
10 10 10 10 4P = 4! ways.
4
Selection of atleast one book No. of ways when all the 4 girls are together
9 = 5! × 4!

WWW.SARKARIPOST.IN
(10 1) (10 1)...(10 1) 1 11 1
Reqd. no. of ways in which all the 4 girls do not sit
9 times
together = 8! – 5! × 4! = 8 × 7 × 6 × 5! – 5! × 24 = 5!
33. (b) Required number is greater than 1 million (7 digits).
(336 – 24) = 120 × 312 = 37440
From given digits, total numbers which can be formed
39. (c) For a particular class total number of different tickets
= 7!
from first intermediate station = 5
Number starting from zero = 6!
Similarly number of different tickets from second
Required number = 7! – 6!
intermediate station = 4
Repetition not allowed, so required answer
So total number of different tickets
7! 6! = 5 + 4 + 3 + 2 + 1 = 15
= 360
2!3! And same number of tickets for another class
34. (c) Total number of hand shakes = 20C2 of those no Indian total number of different tickets = 30 and number
female shakes hand with male of selection = 30C10
5 × 10 = 50 hand shakes 40. (d) To minimise the number of boxes we have to fill the
No American wife shakes hand with her husband maximum number of boxes possible with the maximum
= 5 × 1 = 5 hand shakes number of coins (in the descending order) till the
total number of hand shakes occurred desired number of coins is reached
= 20C2 – (50 + 5) = 190 – 55 = 135
(i.e., at least 3235)
35. (a) The required number of ways
10 boxes of 33 coins each + 10 boxes of 32 coins
= 5C4. 5C2 + 5C3 .5C3 + 5C2. 5C4= 50 + 100 + 50 = 200
each . .......... till 10 boxes of 21 coins each
36. (c) Total number of ways to permute 6 alphabets 2 of which
are common = 6! / 2! = 360.
(1) Treat the two C’s as one = 10 × [33 + 32 +.....21] = 10 × 13 × 32 21
Number of possible ways = 5P5 = 120 2
(b) Number of ways = Total arrangements – Number = 10 × 13 × 27 × 10 = 3510 130 boxes
of arrangements in which they always come together Which is more than 3235
= 360 – 120 = 240. 3510 – 21 × 10 = 3300 120 boxes
37. (b) 1 wicket keeper from 4 can be selected in 3300 – 22 × 3 = 3234 117 boxes.
4! 117 + 1 more box = 118 boxes
4C = = 4 ways
1
3!.1! 41. (d) Required number of possible outcomes
If 4 bowlers are chosen then remaining 6 batsmen - = Total number of possible outcomes –
can be chosen in 11C6. Number of possible outcomes in which 5 does
6! 11! 5 6 11 10 9 8 7 not appear on any dice. (hence 5 possibilities in each
6C . 11C = × = ×
4 6 throw)
4!.2! 3!.1! 2 5 4 3 2
= 15 × 14 × 33 = 6930 = 63 – 53 = 216 – 125 = 91

PDF Download FROM >> WWW.SARKARIPOST.IN Join >> https://www.facebook.com/Sarkaripost.in/


WWW.SARKARIPOST.IN
Permutations and Combinations 599

42. (c) We have in all 12 points. Since, 3 points are used to IV. “3 R, 3 B”:
form a traingle, therefore the total number of traingles One case is when the three blue faces are adjacent to
including the triangles formed by collinear points on one another. The other case is when one blue face is
AB, BC and CA is 12C3 = 220. But this includes the adjacent to the other two but the other two are not
following : adjacent to each other. Hence, total 2 possibilities.
The number of traingles formed by 3 points on Total possibilities = 2 + 2 + 4 + 2 = 10.
AB =3C3 = 1 47. (b) Number of 11 letter words formed from the letter P, E,
The number of triangles formed by 4 points on R, M, U, T, A, I, O, N = 11!/2!.
BC = 4C3 = 4. Number of new words formed = total words
The number of triangles formed by 5 points on – 1 = 11!/2! – 1.
CA = 5C3 = 10. 48. (d) We have no girls together, let us first arrange the 5
Hence, required number of traingles boys and after that we can arrange the girls in the space
between the boys.
= 220 – (10 + 4 + 1) = 205.

WWW.SARKARIPOST.IN
Number of ways of arranging the boys around a circle
43. (c) Starting with the letter A, and arranging the other four
letters, there are 4! = 24 words. These are the first 24 = [5 – 1]! = 24.
words. Then starting with G, and arranging A, A, I, and Number of ways of arranging the girls would be by
placing them in the 5 spaces that are formed between
4! 24
N in different ways, there are 12 words. the boys. This can be done in 5P3 ways = 60 ways.
2!1!1! 2
Hence, total 36 words. Total arrangements = 24 × 60 = 1440.
Next, the 37th word starts with I. There are 12 words 49. (b) If only one get 1 object
starting with I. This accounts up to the 48th word. The The remaining can be distributed as:
49th word is NAAGI. The 50th word is NAAIG. (6, 0), (4, 2), (3, 3).
44. (a) Let there be n participants in the beginning. Then the (7C1 × 6C6 × 3! + 7C1 × 6C4 × 3! + 7C1 × 6C3 × 3!/2!)
number of games played by (n – 2) players = n – 2C2 = 42 + 630 + 420 = 1092.
n 2 If 2 people get 1 object each:
C2 6 84
7C × 6C × 5C × 3!/2! = 126.
1 1 5
(Two players played three games each)
Thus, a total of 1218.
n 2 (n 2)(n 3) 156 50. (b) When all digits are odd
C2 78
5 × 5 × 5 × 5 × 5 × 5 = 56
n 2 5n 150 0 n 15 . When all digits are even
45. (d) No. of words starting with A are 4 ! = 24 4 × 5 × 5 × 5 × 5 × 5 × 5 = 4 × 55
No. of words starting with H are 4 ! = 24 56 + 4 × 55 = 28125
No. of words starting with L are 4 ! = 24 51. (b) Let the number of green balls be x. Then the number
These account for 72 words of red balls is 2x. Let the number of blue balls be y.
Next word is RAHLU and the 74th word RAHUL. Then,
46. (d) Let “x R, y B” denote x Red and y Blue faces such that x + 2x + y = 10 3x + y = 10 y = 10 – 3x
x + y = 6. Clearly, x can take values 0, 1, 2, 3. The corresponding
I. “6 R, 0 B” and “0 R, 6 B”: values of y are 10, 7, 4 and 1. Thus, the possibilities
Only 1 such case is possible for each. are (0, 10, 0), (2, 7, 1), (4, 4, 2) and (6, 1, 3), where (r,
Hence, total 2 possibilities. b, g) denotes the number of red, blue and green balls.
II. “5 R, 1 B” and “1 R, 5 B”: 52. (c) Six consonants and three vowels can be selected from
Only 1 such case is possible for each. 10 consonants and 4 vowels in 10C6 × 4C3 ways. Now,
these 9 letters can be arranged in 9! ways. So, required
Hence, total 2 possibilities.
number of words = 10C6 × 4C3 × 9!.
III. “4 R, 2 B” and “2 R, 4 B”:
53. (a) Total number of numbers without restriction = 25
In “4 R, 2 B”, the two blue faces are either adjacent or
not. So 2 such cases are possible for each. Hence, Two numbers have all the digits equal. So, the required
total 4 possibilities. numbers = 25 – 2 = 30.

PDF Download FROM >> WWW.SARKARIPOST.IN Join >> https://www.facebook.com/Sarkaripost.in/


WWW.SARKARIPOST.IN
600 Quantitative Aptitude

54. (b) In order that two books on physics are never together 58. (d) By gap method :
we place the books as follows
× 1 × 2 × 3 × 4 × 5 × 6 ×
XEXEXEXEXEXEXEXEXEXEXE
Required no. of words
Where E denotes the position of English books 7C × 3! × 6!
3
X denotes the vacant places in between English books,
59. (a) Ten candidates can be ranked in 10! ways. In half of
where the books on physics is to be placed
these ways A1 is above A2 and in another half A2 is
So, there are 11 x marked placed for 8 books on physics.
10 !
11 above A1. So, required number of ways is .
Hence, the required no. of ways = C8 2
60. (a) Two tallest boys can be arranged in 2! ways. Rest 18
11 10 9 can be arranged in 18! ways.
= = 165
3 2 Girls can be arranged in 6! ways.
55. (b) 2310 = 2 3 5 7 11 = say a b c Total number of ways of arrangement = 2! × 18! × 6!
if a = 1 then b c can be chosen in = 18! × 2 × 720 = 18! × 1440

WWW.SARKARIPOST.IN
1 1 1 1 1 1 61. (d) To construct 2 roads, three towns can be selected out
= 16 ways of 4 in 4 × 3 × 2 = 24 ways.
2
If a = 2 then b c can be chosen in Now if the third road goes from the third town to the
first town, a triangle is formed, and if it goes to the
1 1 1 1 1 1 fourth town, a triangle is not formed. So, there are 24
= 8 ways of which one way has
2 ways to form a triangle and 24 ways of avoiding a
already been included in the case a = 1 triangle.
totally 7 ways. 62. (d) For a triangle, two points on one line and one on the
Similarly if a = 3, 5, 7 and 11 then number of ways other has to be chosen.
will be ( 8– 2), ( 8 – 3), (8 – 4), and (8 – 5) 10
No. of ways = C2 × 11C1 + 11C2 × 10 C1 =1,045 .
Respectively and all possible combinations would have
been covered. 63. (c) Single digit numbers = 5
16 + 7 + 6 + 5 + 4 + 3 = 41 ways. Two digit numbers = 5 × 4 = 20
Alternate method : Three digit numbers = 5 × 4 × 3 = 60
2340 = 2 3 5 7 11 Four digit numbers = 5 × 4 × 3 × 2 = 120
When a number can be expressed as a product of n Five digit numbers = 5 × 4 × 3 × 2 × 1 = 120
distinct prime numbers, then it can be expressed as a Total = 5 + 20 + 60 + 120 + 120 = 325
product of 3 numbers in 64. (d) The unit digit can either be 2, 3, 4, 5 or 6.
3n 1
1 34 1 82 When the unit digit is 2, the number would be even
= = = 41 ways. and hence will be divisible by 2. Hence all numbers
2 2 2
with unit digit 2 will be included which is equal to 5!
(56–58) or 120.
SIGNATURE When the unit digit is 3, then in every case the sum of
A, E, G, I, N, R, S, T, U, i.e. all the letters are different. the digits of the number would be 21 which is a multiple
56. (a) 1 2 3 4 5 6 7 8 9 of 3. Hence all numbers with unit digit 3 will be
divisible by 3 and hence will be included.
Total number of such numbers is 5! or 120.
Vowel are A, E, I, U
Similarly for unit digit 5 and 6, the number of required
There are 4 vowels and 4 even places.
numbers is 120 each.
Required number of words in which vowels always
occupy the even places = 4C4 × 4! × 5! When the unit digit is 4, the number would be divisible
by 4 only if the ten’s digit is 2 or 6. Total number of
57. (b) By string method :
such numbers is 2! × 4! or 48.
SGN + 6 =7 Hence, Total numbers = 120 + 120 + 120 + 120 + 48
Required number of works = 7! × 3! = 528

PDF Download FROM >> WWW.SARKARIPOST.IN Join >> https://www.facebook.com/Sarkaripost.in/


WWW.SARKARIPOST.IN
Permutations and Combinations 601

65. (d) The odd digits have to occupy even positions. This 3, 3, 1, 1 (4!/2! × 2! = 6 ways);
4! 3, 2, 2, 1 (4!/2! = 12 ways) and 2, 2, 2, 2 (1 way).
can be done in 6ways Total number of ways = 4 + 12 + 6 + 12 + 1 = 35 ways.
2!2!
70. (d) For each book we have two options, give or not give.
The other digits have to occupy the other positions.
Thus, we have a total of 214 ways in which the 14 books
5! can be decided upon. Out of this, there would be 1
This can be done in 10 ways
3!2! way in which no book would be given. Thus, the
number of ways is 214 – 1.
Hence total number of rearrangements possible
71. (c) We cannot take ‘0’ since the smallest digit must be
= 6 × 10 = 60.
placed at the left most place. We have only 9 digits
66. (d) Each box can be filled in 2 ways. from which to select the numbers. First select any
Hence, total no. of ways = 25 = 32 number of digits. Then for any selection there is only
Blue balls cannot be filled in adjacent boxes one possible arrangement where the required condition
Total no. of such cases in which blue ball is filled in 2 is met. This can be done in 9C1 + 9C2 + 9C3 +

WWW.SARKARIPOST.IN
adjacent boxes is ........ + 9C9 ways 29 – 1 = 511 ways.
2 blue + 3 blue + 4 blue + 5 blue But we can’t take numbers which have only one digit,
= 4 ways ( 12, 23, 34, 45) + 3 ways ( 123, 234, 345) hence the required number is 511 – 9 = 502
+ 2 ways (1234, 2345) + 1 way 72. (c) The condition is that we have to count the number of
= 10 ways natural numbers not more than 4300.
Hence, total cases in which blue balls can not be filled The total possible numbers with the given digits
in adjacent boxes = 32 – 10 = 22
= 5 × 5 × 5 × 5 = 625 – 1 = 624.
Subtract from this the number of natural number greater
67. (d) Mr. than 4300 which can be formed from the given digits
A = 1 × 2 × 5 × 5 – 1 = 49.
C Mrs.
Mrs. B Hence, the required number of numbers = 624 – 49.
or 73. (d) You can form triangles by taking 1 point from each
Mrs. D
side, or by taking 2 points from any 1 side and the
third point from either of the other two sides.
This can be done in: 4 × 5 × 6 = 4C2 × 11C1 + 5C2
× 10C1 + 6C2 × 9C1 = 120 + 66 + 100 + 135 = 421
74. (a) First we write six '+' sings at alternate places i.e., by
Since no two men are adjacent to each other, therefore leaving one place vacant between two successive '+'
no male is on the right of Mr. A. sings. Now there are 5 places vacant between these
sings and these are two places vacant at the ends. If we
Since wives are three places away from their husbands,
write 4 '–' sings these 7 places then no two '–' will come
therefore Mrs. A cannot be on the right of Mr. A.
together. Hence total number of ways 7C4 = 35
Mrs. E cannot be on the right of Mr. A, since Mrs. B
cannot be on left of Mr. A. Hence, either Mrs. B or 75. (b) Let x1 be the number of stations before the first halting
Mrs. D can be on the right of Mr. A. station, x2 between first and second, x3 between second and third,
x4 between third and fourth and x5 on the
68. (b) There are 28 minutes, hence total no. of songs are 14.
Since each pair sings one song. Hence, total number right of 4th stations. Then x1 0, x5 0, x2 , x3, x4 1
of pairs is 14. satisfying x1 + x2 + x3 + x4 + x5 = 8 …(1)
Since, in each possible pair persons are not standing The total number of ways is the number of solution of
next to each other. the above equation
n
C2 n 14 n 7 Let y2 = x2 –1, y3 = x3 – 1, y4 = x4 – 1.
Then (1) reduces to x1 + y2 + y3 + y4 + x5 = 5,
Hence, total number of people = 7.
where y2, y3, y4 0.
69. (b) The ways of placing the balls would be 5, 1, 1, 1(4!/3!
= 4 ways); The number of solution of this equation is
5 + 5 – 1C 9
4, 2, 1, 1(4!/2! = 12 ways); 5 – 1 = C4.

PDF Download FROM >> WWW.SARKARIPOST.IN Join >> https://www.facebook.com/Sarkaripost.in/


WWW.SARKARIPOST.IN
602 Quantitative Aptitude

76. (a) India wins exactly in 5 matches looses in none 1 1


5C ways
0 (13 n)(12 n) (n 2)(n 1)
India wins exactly in 6 matches wins the 6th and
st
looses anyone in the 1 five 5 C1 ways and so on. (n + 2) (n + 1) < (13 – n) (12 – n)
Total number of ways = C0 + C1 + 6C2 + 7C3 + 8C4
5 5 n 2 + 3n + 2 < n 2 – 25n + 156
= 126 3n + 2 < – 25n + 156
77. (d) Number of words in which no letter is repeated 28n < 154 n < 154 / 28 or n < 5.5
= 10 × 9 × 8 × 7 × 6 = 30240 Since we need integral values, so n can take any
Hence, the number of words in which at least one letter value from 0 to 5.
is repeated = 105 – 30240 = 69760 2. (d) The digits that make the numbers are 1, 3, 5, 7 and 9.
78. (a) The two letters, the first and the last of the four lettered The number of numbers with one of these in the first
word can be chosen in (17)2 ways, as repetition is place = 4!
allowed for consonants. The two vowels in the middle Every one of the digits appear in all the 5 places –
are distinct so that the number of ways of filling up the digital, 10th, 100th, etc. in one or the other 5-digit

WWW.SARKARIPOST.IN
two places is 5P2 = 20. The number of different words number. The required sum of all the numbers is,
= (17)2 . 20 = 5780. therefore,
79. (a) First prize may be given to any one of the 4 boys, hence 105 1
first prize can be distributed in 4 ways. = 25 (104 + 103 + 102 + 10 + 1) × 4! = 600 ×
10 1
Similarly every one of second, third, fourth and fifth = 600 × 11111 = 6666600.
prizes can also be given in 4 ways. 3. (b) Numbers greater than 3400 will have, 4 or 5 or 6 or 7
the number of ways of their distribution in the first place. Having filled the first place say by 4,
= 4 × 4 × 4 × 4 × 4 = 45 = 1024. we have to choose 3 digits out of the remaining 6 and
80. (c) (W) – – fixed the number will be 6P3 = 6!/3! = 6 × 5 × 4 = 120.
M M Fix the position of one woman Therefore total of such numbers will be
Remaining women can sit in 3P3 ways. 4 × 120 = 480 ........ (1)
W W Numbers greater than 3400 can be those which have
M M 34, 35, 36, 37 in the first two places. Having filled up
W 34 in the first two places we will have to choose 2
Remaining men can sit in 4P4 ways more out of remaining 5 and the number will be
5P = 5!/3! = 5 × 4 = 20
Total 3P3 × 4P4 = 144 ways 2
Therefore, total as above will be 20 × 4 = 80 ..... (2)
81. (d) Required number of possible outcomes
Hence all the numbers greater than 3400 will be
= Total number of possible outcomes – Number of
possible outcomes in which 5 does not appear on any 480 + 80 = 560
dice Alternate method : Numbers less than 3400 will have
1 or 2 in 1st place or 31, 32 in the first two position.
= 63 – 53 = 91.
6P + 6P = 120 + 120 = 240
82. (b) A triangle requires 3 non collinear points, 10 C 3 3 3
5P + 5P = 20 + 20 = 40.
combinations. But 5 points give us straight line. Hence 2 2
number of triangles = 10C3 – 5C3 = 120 – 10 = 110. Total numbers which are less than 3400
= 240 + 40 = 280.
Expert Level Also total number of numbers formed is 7P4 = 840.
Hence numbers greater than 3400 is 840 – 280 = 560
13 C 13 C 13! 13! 4. (c) A number will be divisible by 24 if the last two digits
1. (c) n < n+2
(13 n)!n! (13 n 2)!(n 2)! are divisible by 25 and this can be done in two ways
for either 25 or 75 can be three and remaining two
13! 13! places out of 5 digits can be filled in 5P2 ways.
(13 n)!n ! (11 n)!(n 2)! Hence the required number = 2 × 5P2 = 2 × 20 = 40
5. (d) A number is divisible by 4 if the last two digits are
divisible by 4 which can be done in 10 ways
1! 1! (12, 16, 24, 32, 36, 52, 56, 64, 72, 76).
(13 n)(12 n)(11 n)!n! (11 n)! ( n 2)( n 1) n ! Hence number = 10 × 5P2 = 10 × 20 = 200

PDF Download FROM >> WWW.SARKARIPOST.IN Join >> https://www.facebook.com/Sarkaripost.in/


WWW.SARKARIPOST.IN
Permutations and Combinations 603

6. (a) No. of non-relative friends = 12 – 7 = 5. Number of triangle with one side common with octagon
He may invite 6 friends in follows ways : = 8 × 4 = 32
I : 4 relatives + 2 non-relatives 7C4 × 5C2 (Consider side A1 A2 . Since two points A3 , A8 are
II : 5 relatives + 1 non-relatives 7C5 × 5C1 adjacent, 3rd point should be chosen from remaining
4 points.)
III : 6 relatives + 0 non-relatives 7C6
Reqd. no. of ways = 7C4 × 5C2 × 7C5 × 5C1 + 7C6 Number of triangles having two sides common with
octagon : All such triangles have three consecutive
= 35 × 10 + 21 × 5 + 7 = 462
vertices, viz., A1A2A3, A2A3A4, ..... A8A1A2.
7. (d) We will consider the following cases,
Number of such triangles = 8
I case : 4 alike and 2 others alike
Number of triangles with no side common
II case : 4 alike and 2 different
III case : 3 alike and 3 others alike = 56 – 32 – 8 = 16.
IV case : 3 alike and 2 other alike and 1 different 11. (b) The first digit x can be any one of 1 to 8 whereas z can
Total ways be any one of 0 to 9.
When x is 1, y can assume the values 2 to 9 ;
6! 6! 6! 6! when x is 2, y can assume the values 3 to 9 and so on.

WWW.SARKARIPOST.IN
2
= 1 1 1 C1
4! 2! 4! 3! 3! 3! 2! Thus the total number = (8 + 7 +. ...........+ 1) × 10
= 15 + 30 + 20 + 120 = 185.
8. (b) Letters of the word PARKAR written in alphabetical = 8.9 . 10 = 360.
order are A A K P R R 2
Number of words starting with A is = 60 12. (b) Given n points the number of triangles that can be
Number of words starting with K is = 30 drawn by joining any three non-collinear points nC3
out of this mC3 is to be subtracted as m points are
Number of words starting with PAA is =3
collinear and no triangle is possible within the m points.
Number of words starting with PAK is =3
Number of words starting with PARA is = 20 6! 5!
13. (a) Required number = 2!3! 3! 40
Number of words starting with PARKAR is = 1
––––
14. (d) Four digit numbers = 3 . 3. 2. 1 = 18
Rank of word PARKAR is 99
Five digit numbers = 4. 3. 2. 1. 1 = 24
9. (b) The inequality is n 1 n 1 Total number of numbers = 42
C3 C2 100
15. (b) To draw a straight line, we need two points. Hence
We must have n 1 3 and n 1 2 10C lines are possible. But 5 points are collinear, hence
2
n 2 and n 1 we subtract 5C2 but these 5 points give 1 straight line
Number of straight lines possible
(n 1) n ( n 1) ( n 1) n = 10C2 – 5C2 + 1 = 45 – 10 + 1 = 36
n 2 and also 100
6 2 16. (d) Number of ways = Arrangement of (m – 1) things of
(n 1) n (n 4) 600 one kind and (n – 1) things of the other kind
By trial the values of n satisfying this are 2, 3, 4, 5, 6, (m n 2)!
7, 8, 9 which are eight in number. =
(m 1)!(n 1)!
10. (d) Number of all possible triangles
17. (d) S1 S3 S2 or S3 S1 S2
= Number of selections of 3 points from 8 vertices
Three places can be chosen in 10C3 ways and there are
= 8C3 = 56 only 2 possible arrangements.
Now remaining seven can be arranges in 7! ways
A6 A5
Total ways = 10
C3 2 7!
18. (d) When A has B or C to his right we have AB or AC
A7 A4
when B has C or D to his right we have BC or BD
Thus we must have ABC or ABD or AC and BD
A8 for ABC D, E, F on a circle number of ways = 3 ! = 6
A3 for ABD C, E, F on a circle number of ways = 3 ! = 6
for AC , BD E, F the number of ways = 3 ! = 6
A1 A2 Total = 18

PDF Download FROM >> WWW.SARKARIPOST.IN Join >> https://www.facebook.com/Sarkaripost.in/


WWW.SARKARIPOST.IN
604 Quantitative Aptitude

19. (b) 3 vacancies for SC candidates can filled up from 5 26. (b) There can be 2 possibilities – last digit is odd or even.
candidates in 5C3 ways. Case I : Last digit is odd. Fixing one out of 1, 3 & 5 in
After this for remaining 12 – 3 = 9 vacancies, there the last position. Then only one odd number can occupy
will be 25 – 3 = 32 candidates. odd position which can be chosen in 2C1 ways = 2.
These vacancies can be filled up in 22C9 ways. One of the two odd digits can be selected for this
Hence required number of ways = 5C3 × 22C9 position in again, 2C1 ways = 2.
20. (c) Consider cases when z = 0, 1, 2, ....... , 11 The other odd number can be put in either of the two
x + y = 33, 33, 27,... even places in 2 ways.
Total number of solution of x + y = 33, 30, 27,... Finally the two even numbers can be arranged in 2!
= 34 + 31 + 28 + ...... + 1 (12 times) ways.
12 Hence sum of last digit of these nos.
= (1 + 34) = 210
2 = (2 × 2 × 2 × 2) (1 + 3 + 5) = 144 ways
Case II : Last digit is even. Then 2 odd nos. out of 3
21. (a) can be arranged in 3P2 = 3! ways.

WWW.SARKARIPOST.IN
Again the even nos. can be arranged in 2! ways
Sum = (3! × 2) (2 + 4) = 72 ways.
number of intersection points
37C – 13C2 – 11C2 + 2 Total ways = 144 + 72 = 216.
2
( two points A & B) = 535. 27. (a) Number of ways of arranging p big animals into m – n
22. (b) Every number between 3000 and 4000, which is big cages = m – n pp.
divisible by five and which can be formed by the given Now remaining animals can be arranged in any cage
digits, must contains 5 in unit’s place and 3 in in m – pPm – P ways
thousand’s place. Thus we are left with four digits out Desired number of ways = m – nPp × m – pPm – p
of which we are to place two digits between 3 and 5,
28. (b) Six students can be arranged in a row is 6! ways.
which can be done in 4P2 = 12 ways. Hence 12 numbers
Another six students can be further arranged in 6! ways.
can be formed.
Hence, total number of ways = 6! × 6!
23. (a) With no restrictions, n women may be seated in a row
in nPn ways. Note: Do not get confused with the two type of
If 2 of the n women must always sit next to each other, booklets. The booklets can be distributed in 2 ways.
the number of arrangements = 2!( n–1Pn–1). Both these arrangements will be part of the permutation
Hence the number of ways n women can be seated in a of students arrangement.
row if 2 particular women may never sit together
= nPn – 2(n–1Pn–1) = n! – 2(n – 1)! 1 2 1 2 1 2 2 1 2 1 2 1
= n(n – 1)! – 2(n – 1)! 1 2 1 2 1 2 2 1 2 1 2 1
= (n – 2) × (n – 1)!.
29. (a) Each one of the 26 players played 25 matches and none
24. (c) Each object can be put either in box B1 (say) or in box
B2 (say). So, there are two choices for each of the n of the matches ended in a draw Hence all the scores
objects. Therefore the number of choices for n distinct must be even. Also each one of them scored different
from the other. The maximum score possible is 50 and
objects is 2 2 ... 2 2n . minimum score is 0. There are exactly 26 possible
n times scores, 50, 48, 46 .....0.
One of these choices correspond to either the first or The ranking is in a alphabetical order means A scored
the second box being empty. 50, B – 48, Z – 0.
Thus, there are 2n – 2 ways in which neither box is This is possible if A wins all the matches B loses only
empty. to A win against all others etc.
25. (b) The number of selections of 1 pair of vowels and 1 In final rank, every player win only with all players
pair of consonants = 5C1 × 21C2
who are below in final ranking . Since M > N hence M
The number of selections of 2 different vowels and 2 wins over N.
different consonants = 5C2 × 21C2
30. (c) The no. of ways are
the required number of wards
2n 1 2n 1 2n 1
4! C1 C2 ... Cn 63
= 5C1 × 21C1 × 2!2! + 5C2 × 21C2 × 4!
By option elimination, 2n + 1 = 7. So n = 3.

PDF Download FROM >> WWW.SARKARIPOST.IN Join >> https://www.facebook.com/Sarkaripost.in/


WWW.SARKARIPOST.IN
Permutations and Combinations 605

31. (d) There are 12 points. Since they can be reached from Total no. of lines required for connecting towns of
12
any other point, the edges will be C2 = 66. Also the different zones = 4 P2 9 6 9 54
number of edges will be maximum 11.
Total no. of lines in all = 54 + 36 = 90
32. (a) The number of triangles with vertices on different
36. (a) Task 2 can be assigned to 3 or 4
lines p C 1 × p C 1 × p C 1 = p 3
So, there are only 2 options for task 2.
The number of triangles with 2 vertices on one Now, task 1 can not be assigned to 1 or 2 i.e. there are
line and the third vertex on any one of the other 3 options.
two lines
So required no. of ways
p p 1 = (2 options for task 2) × (3 options for task 1)
= 3C1 (pC2 × 2p C
1) = 6p. = 3p 2 (p – 1) × (4 options for task 3) × (3 options for task 4)
2
× (2 options for task 5) × (1 option for task 6).
the required number of triangles
= 2 × 3 × 4 × 3 × 2 × 1 = 144.
= p 3 + 3p 2 (p – 1) = 4p 3 – 3p 2 = p 2 (4p – 3) 37. (a) No. of words starting with A = 8!/2!3! = 3360.
(The work “maximum” shows that no selection of No. of words starting with B = 8!/2!4! = 840.

WWW.SARKARIPOST.IN
points from each of the three lines are collinear). No. of words starting with C = 8!/2!4! = 840
33. (d) First step — take book 3 to the table B and, second No. of words starting with D = 8!/2!4! = 840
step — put the book 2 on top of 3. Third step — No. of words starting with H = 8!/2!4! = 840
Transfer the arrangement and keep it over book 1 on
Now of words starting with LAA = 6!2! = 180
table A. The last step is transfer the whole arrangement
to the table B which is the fourth step to take. Thus Now LAB starts and first word starts with LABA.
total 4 steps are required. No. of words starting with LABAA = 4! = 24.
34. (d) There are 32 black and 32 white squares on a chess After this the next words will be LABADAAHL,
board. Then no. of ways in choosing one white and LABADAALH, LABADAHAL, LABADAHLA and
one black square on the chess hence, Option (a) is the answer.
38. (c) No. of 1 digit nos = 9
= 32 32
C1 C1 32 32 1024 No. of 2 digit nos = 81
There are 8 rows and 8 columns on a chess board. No. of 3 digit nos = 9 × 9 × 8 = 648
In each row or column, there are 4 white and 4 black No. of 4 digit nos = 9 × 9 × 8 × 7 = 4536
squares. Total nos = 9 + 81 + 648 + 4536 = 5274
Therefore number of ways to choose a white and a 39. (b) There are two ways of selecting 635 or 674. If last
black square from the same row digit is 9 , then there are 9 ways of filling each of the
remaining 3 digits.
= 4 C1 4
C1 8 = 128
Thus total no. of this type of numbers = 2 × 9 3 = 1,458.
No. of ways to choose a white and a black square from When last digit is not 9, total no. of this type of numbers
the same column = 2 × 3 × 4 × 92 = 1944.
4
C1 4
C1 8 128 [9 can be selected at any of the 4th, 5th or 6th place in
3 ways. Also at the unit place 4 odd nos. except 9 can
Total ways in which a white and a black squares lie on be used.]
the same row or same column = 128 + 128 = 256 Thus required no. = 1,944 + 1,458 = 3,402
Hence, required no. of ways = 1024 – 256 = 768 40. (a) For 2 such lines, no. of regions formed are 4
35. (b) For 3 lines no. of regions formed are 7 (= 4 + 3)
For 4 lines, no. of regions formed are 11 (= 7 + 4)
For 5 lines no. of regions formed are 16 (= 11 + 5)
Similarly for 6, 7, 8, 9 and 10 lines, no. of regions are
16 + 6 = 22
Consider zone 1 22 + 7 = 29
No. of lines for internal connections in each zone = 9 29 + 8 = 37
Total number of lines for internal connections in 37 + 9 = 46
four zones = 9 × 4 = 36 46 + 10 = 56
No. of lines for external connections between any For 10 lines no. of regions = 10C2 + 10 + 1
two zones = 3 × 3 = 9 (as shown in figure) = 45 + 11 = 56

PDF Download FROM >> WWW.SARKARIPOST.IN Join >> https://www.facebook.com/Sarkaripost.in/


WWW.SARKARIPOST.IN
606 Quantitative Aptitude

41. (d) Case (I): When number of 50 misos is 0, The Besides, the papers can be given in the pattern of
No. of 10 misos No. of 1 misos 121212 or 212121. Hence the answer is
10 7 2 × (12C6 × 6! × 6!)
9 17 44. (d) For the maximum possibility assume that no three
8 27 points other than given in the question are in a straight
line.
45. (a) The total number of solutions for x + y + z = 36, if
x, y and z are whole numbers is given by
1 97 36+3–1
C3–1 = 38C2 = 703.
0 107 The number of solutions where x = y will be 19 {from
Number of ways to pay the bill = 11 (x, y) = (0, 0) to (18, 18)}.
Case (II): When number of 50 misos is 1, then The number of solutions where x is not equal to y
No. of 10 misos No. of 1 misos = 703 . 19 = 684
5 7 Among these 684 solutions, half will have x > y and

WWW.SARKARIPOST.IN
4 17 the rest will have y > x.
3 27 Hence, the total number of solutions where x y
2 37 684
19 361
1 47 2
0 57 46. (b) Let the three numbers be 13a, 13b and 13c, where a,
Number of ways to pay the bill = 6 b and c are coprime.
13a + 13b + 13c = 117
Case (III) : When number of 50 misos is 2, then
13(a + b + c) = 13 × 9
No. of 10 misos No. of 1 misos
a+b+c=9
0 7
The number of positive integer solutions of a + b +
Number of ways to pay the bill = 1. Hence, from all
c = 9 is 9 – 1C3–1 i.e., 8C2 = 28.
the three cases, we got total numbers of ways to pay a
bill of 107 misos = 11 + 6 + 1 = 18. However, there is a case, a = b = c = 3, where a, b
and c are not coprime.
42. (a) The 6 balls must be on either of the middle rows. This
So the answer = 28 – 1 = 27
can be done in 2 ways. Once, we put the 6 balls in their
single horizontal row — it becomes evident that for 47. (a) In this question you will first have to complete the
placing the 2 remaining balls on a straight line there selection of 4 people for either side and then arrange the
are 2 principal options: rowers on each side (which would be done by using 4!)
1. Placing the two balls in one of the four rows with The solution would depend on the following structure
two squares. In this case the numbers of ways of the structure would very based on whether you select
placing the balls in any particular row would be 1 2 more men for the right side or you select 1 man and
way (since once you were to choose one of the 4 1 woman for the right side or you select 2 women for
rows, the balls would automatically get placed as the right side.
there are only two squares in each row.) Thus the The solution would be given by:
12C × 4! × 8C × 4! + 12C × 8C × 4! × 7C × 4! + 8C
total number of ways would be 2 × 4 × 1 = 8 2 1 1 1 1 2
ways. × 4! × 6C1 × 4! = 1368 × 4! × 4!
2. Placing the two balls in the other row with six 48. (c) For answering this question we need to plan the use of
squares. In this case the number of ways of placing the factors of 1998.
the 2 balls in that row would be 6C2. This would 1998 = 2 × 33 × 37 16 factors viz. 1 × 1998,
give us 2C1 × 1 × 6C2 = 30 ways. Total is 30 + 8 = 2 × 999, 3 × 666, 9 × 222, 18 × 111, 27 × 74, 54 × 37.
38 ways. Thus we could form 7 APs as follows:
43. (a) First select six people out of 12 for the first row. The (1) An AP with 2 terms and average 999
other six automatically get selected for the second row. (2) An AP with 3 terms and average 666 and so on
Arrange the two rows of people amongst themselves. 7 ways.

PDF Download FROM >> WWW.SARKARIPOST.IN Join >> https://www.facebook.com/Sarkaripost.in/


WWW.SARKARIPOST.IN
Permutations and Combinations 607

Explanation of
Test Yourself

1. (d) Numbers starting with 12 – 7 numbers 7. (c) The number would be of the form 6 ____ 5
Numbers starting with 13 – 6 numbers; 14 – 5, 15 – 4, The 5 missing digits have to be formed using the digits
16 – 3, 17 – 2, 18 – 1. Thus total number of numbers 0, 1, 2, 3, 4, 7, 8, 9 without repetition.
starting from 1 is given by the sum of 1 to 7 = 28. Thus, 8C5 × 5! = 6720
Number of numbers starting from 2- would be given
8. (a) V O W E L
by the sum of 1 to 6 = 21
Number of numbers starting from 3- sum of 1 to X X
5 = 15 1 2 3 4 5
Number of numbers starting from 4 – sum of 1 to O and E occupying only even places [ X marked
4 = 10

WWW.SARKARIPOST.IN
places] can be arranged in 2P2 ways = 2
Number of numbers starting from 5 – sum of
Remaining letters can be arranged in (3!) ways = 6
1 to 3 = 6
Required number of words = 2 × 6 = 12
Number of numbers starting from 6 = 1 + 2 = 3
9. (d) 18C + 18C + 18C + ... + 18C + 18C
Number of numbers starting from 7 = 1 4 5 6 17 18
Thus a total of: 28 + 21 + 15 + 10 + 6 + 3 + 1 = 84 such = [ C0 + C4 + ... + C18] – [ C0 + 18C1
18 18 18 18 + 18C
2
numbers. + 18C3]
2. (c) Total number of arrangements of letters in the word
= 218 – [1 + 18 + 153 + 816]
GARDEN = 6 ! = 720 there are two vowels A and E,
in half of the arrangements A preceeds E and other = 261158
half A follows E. 10. (b) 10C × 8C1+ 10C1 × 8C2 = 360 + 280 = 640
2
1 11. (b) The numbers should be formed from 1, 2, 3, 4 and 5
So, vowels in alphabetical order in 720 360
2 (without repetition), such that the digit at the units place
3. (a) For each historical monument, there ar e two must be greater than in the tenth place. Tenth place has
possibilities either he visit or does not visit. five options.
Total number of ways = 25. 26 (27 –1) If 5 is at the tenth place then the digit at the unit’s
4. (b) n items can be arranged in a circle or circular place cannot be filled by the digit greater than that at
arrangement of n items can be made in (n – 1)! ways. the tenth place.
The 7 men can be sitted around round table in 6! ways If 4 is at the tenth place, then the unit’s place has only
such that 7 places are left empty between them. In those option of 5, while the three places can be filled up in
7 places 7 women can be arranged in 7! ways. 3! Ways.
Required number of ways = 6! 7! If 3 is at the tenth place, then the units’ place can be
5. (b) There will be two types of triangles filled up by 4 or 5, i.e. in 2 ways. While other three
The first type will have its vertices on the three sides places can filled up in 3! ways.
of the ABC. If 2 is at the tenth place, then the unit’s place can be
filled up by 3, 4 or 5 i.e. in 3 ways. While other three
The second type will have two of it’s vertices on the
places can be filled up in 3! Ways.
same side and the third vertex on any of the other two
If 1 is at the tenth place, then any other four places can
sides.
be filled up in 4! Ways.
Hence, the required number of triangles
Thus the total number of numbers satisfying the given
= 6 × 5 × 3 6C2 × 8 + 5C2 × 9 + 3C2 × 11 conditions is
= 90 + 120 + 90 + 33 0 + 3! + 2(3!) + 3(3!) + 4! = 60.
= 333 12. (b) Paths from A to F are
6. (a) All six digit numbers – Six digit numbers with only ABCF, ADEF, ABEF, ADCF, ABDEF, ABDCF,
odd digits. ABDCEF, ABCEF, ADCEF and ABF
= 900000 – 5 × 5 × 5 × 5 × 5 × 5 = 884375. Hence, 10 possible distinct roots.

PDF Download FROM >> WWW.SARKARIPOST.IN Join >> https://www.facebook.com/Sarkaripost.in/


WWW.SARKARIPOST.IN
608 Quantitative Aptitude

13. (c) Three numbers can be selected and arranged out of A and B can be arranged in 2 ways.
10 numbers in 10P3 ways All the other 24 alphabates can be arranged in 24! ways.
10! Hence the required answer
= 10 × 9 × 8. = 2 × 18 × 24! = 36 × 24!
7!
Now this arrangement is restricted to a given condition 15. (a) Let number of girls = x and the number of boys = y
that first number is always less than the second number 45 games in which both the players were girls
and second number is always less than the third number. x
C2 45
Hence three numbers can be arranged among
themselves in 3 ways.
x!
Hence, required number of arrangements x( x 1) 90 x 10
2! ( x 2)!
10 9 8
120 ways 190 games, where both the players were boys.
3 2
yC = 190 y(y – 1) = 380 y = 20
14. (c) A and B can occupy the first and the ninth places, the 2
second and the tenth places, the third and the eleventh Hence, the total number of games in which one player

WWW.SARKARIPOST.IN
place and so on... This can be done in 18 ways. was a boy and the other was a girl = 10 × 20 = 200

PDF Download FROM >> WWW.SARKARIPOST.IN Join >> https://www.facebook.com/Sarkaripost.in/


WWW.SARKARIPOST.IN

Foundation Level
1. Two dice are thrown simultaneously. The probability of 8. The probability of getting number 5 in throwing a dice is
obtaining a total score of seven is 1
(a) 1 (b)
1 1 3
(a) (b) 1 5
6 3 (c) (d)
6 6
2 5 9. The probability of getting head and tail alternately in three
(c) (d) throws of a coin (or a throw of three coins), is
7 6
1 1

WWW.SARKARIPOST.IN
2. Four balls are drawn at random from a bag containing 5 white, (a) (b)
4 green and 3 black balls. The probability that exactly two of 8 4
them are white is 1 3
(c) (d)
3 8
14 7 10. A die is thrown once. What is the probability of occurrence
(a) (b)
33 16 of an odd number on the upper face?
18 9 2 1
(c) (d) (a) (b)
33 16 3 2
1 1
3. Two dice are tossed. The probability that the total score is a (c) (d)
prime number is : 4 8
11. A die is thrown once. Find the probability that 3 or greater
1 5 than 3 turns up.
(a) (b)
6 12 1 1
(a) (b)
1 7 2 3
(c) (d) 1 2
2 9
(c) (d)
4. Anil can kill a bird once in 3 shots. On the assumption that 4 3
he fires 3 shots, find the probability that the bird is killed. 12. Find the probability of getting a multiple of 2 in the throw
of a die.
3 (a) 1/2 (b) 1/4
1 1
(a) (b) (c) 1/3 (d) 1/6
3 3
13. India and Pakistan play a 5 match test series of hockey, the
19 8
probability that India wins at least three matches is
(c) (d) 1 3
27 9 (a) (b)
5. If A and B are two independent events with P(A) = 0.6, 2 5
4
P(B) = 0.3, then P ( A ' B ') is equal to : (c) (d) None of these
5
(a) 0.18 (b) 0.28 14. The probability that a man can hit a target is 3/4. He tries 5
(c) 0.82 (d) 0.72 times. The probability that he will hit the target at least three
6. The probabilities that A and B will die with in a year are p times is
and q respectively, then the probability that only one of them 291 371
will be alive at the end of the year is - (a) (b)
364 461
(a) p + q (b) p + q – pq 471 459
(c) p + q + pq (d) p + q – 2pq (c) (d)
502 512
7. A pair of dice is thrown thrice. The probability of throwing 15. From eighty cards numbered 1 to 80, two cards are selected
doublets at least once is randomly. The probability that both the cards have the
numbers divisible by 4 is given by
1 25
(a) (b) 21 19
36 216 (a) (b)
316 316
125
(c) (d) None of these 1
216 (c) (d) None of these
4

PDF Download FROM >> WWW.SARKARIPOST.IN Join >> https://www.facebook.com/Sarkaripost.in/


WWW.SARKARIPOST.IN
616 Quantitative Aptitude

16. The probability of getting sum more than 7 when a pair of (a) 1/63 (b) 16/17
dice are thrown is (c) 5!/9! (d) None of these
7 5 24. 3 integers are chosen at random from the set of first 20
(a) (b)
36 12 natural numbers. The chance that their product is a multiple
7 of 3, is.
(c) (d) None of these 194 1
12 (a) (b)
17. Two dice are thrown simultaneously then the probability of 285 57
obtaining a total score of 5 is 13 3
(c) (d)
1 1 19 4
(a) (b)
18 12 25. The probability that Krishna will be alive 10 years hence, is
1 7 7
(c) (d) None of these and that Hari will be alive is . What is the probability
9 15 10
that both Krishna and Hari will be dead 10 years hence ?
18. The probability that the two digit number formed by digits
21 24
1, 2, 3, 4, 5 is divisible by 4 is (a) (b)

WWW.SARKARIPOST.IN
150 150
1 1 49 56
(a) (b) (c) (d)
30 20 150 150
26. The probability that in the random arrangement of the letters
1 of the word ‘UNIVERSITY’, the two I’s does not come
(c) (d) None of these
5 together is
19. Probability of throwing 16 in one throw with three dice is 4
(a) (b) 1/ 5
1 1 5
(a) (b) (c) 1/10 (d) 9/10
36 18
27. Among 15 players, 8 are batsmen and 7 are bowlers. Find
1 1 the probability that a team is chosen of 6 batsmen and 5
(c) (d)
72 9 bowlers:
20. Of a total of 600 bolts, 20% are too large and 10% are too
8 7 28
small. The remainder are considered to be suitable. If a bolt C6 C5
(a) (b)
is selected at random, the probability that it will be 15
C11 15
suitable is
15
(c) (d) None of these
1 7 28
(a) (b)
5 10 28. A four digit number is formed by the digits 1, 2, 3, 4 with
no repetition. The probability that the number is odd is
1 3
(c) (d) 1
10 10 (a) zero (b)
3
21. The probability that in the toss of two dice we obtain the
1
sum 7 or 11 is (c) (d) None of these
4
1 1 29. X speaks truth in 60% and Y in 50% of the cases. The
(a) (b)
6 18 probability that they contradict each other narrating the same
2 23 incident is
(c) (d) 1 1
9 108 (a) (b)
22. A card is drawn at random from a pack of 100 cards 4 3
numbered 1 to 100. The probability of drawing a number 1 2
(c) (d)
which is a square, is 2 3
30. An integer is chosen at random from the numbers 1, 2, .....,
1 1
(a) (b) 25. The probability that the chosen number is divisible by
10 100
3 or 4, is
9 90
(c) (d) 2 11
10 100 (a) (b)
23. The alphabets of word ALLAHABAD are arranged at 25 25
random. The probability that in the words so formed, all 12 14
identical alphabets are found together, is (c) (d)
25 25

PDF Download FROM >> WWW.SARKARIPOST.IN Join >> https://www.facebook.com/Sarkaripost.in/


WWW.SARKARIPOST.IN
Probability 617

31. The probability that a leap year will have 53 Friday or 39. In throwing of two dice, what is the number of exhaustive
53 Saturday, is events ?
2 3 (a) 6 (b) 12
(a) (b) (c) 36 (d) 18
7 7
40. In a lottery, 16 tickets are sold and 4 prizes are awarded. If
4 1 a person buys 4 tickets,what is the probability of his winning
(c) (d)
7 7 a prize?
32. An experiment yields 3 mutually exclusive and exhaustive
4 175
events A, B, C. If P (A) = 2P (B) = 3P (C), then P (A) is (a) (b)
equal to 164 256

1 2 1 81
(a) (b) (c) (d)
11 11 4 256

3 6 41. A card is drawn from a pack of 52 cards and a gambler bets


(c) (d) that it is a spade or an ace. Which one of the following are

WWW.SARKARIPOST.IN
11 11
the odds against his winning this bet?
33. If P (A B) = 0.8 and P (A B) = 0.3, then P(A ) + P(B )
(a) 13 to 4 (b) 4 to 13
equals to
(c) 9 to 4 (d) 4 to 9
(a) 0.3 (b) 0.5
42. Each of A and B tosses two coins. What is the probability
(c) 0.7 (d) 0.9
that they get equal number of heads?
34. Five coins whose faces are marked 2, 3 are thrown. What is
the probability of obtainining a total of 12 ? 3 5
(a) (b)
1 3 16 16
(a) (b)
16 16 4 6
(c) (d)
5 7 16 16
(c) (d)
16 16 43. What is the probability that in a family of 4 children there
35. An aircraft has three engines A, B and C. The aircraft crashes will be at least one boy?
if all the three engines fail. The probabilities of failure are
15 3
0.03, 0.02 and 0.05 for engines A, B and C respectively. (a) (b)
What is the probability that the aircraft will not crash? 16 8
(a) 0.00003 (b) 0.90 1 7
(c) 0.99997 (d) 0.90307 (c) (d)
16 8
36. A coin is tossed three times. What is the probability of getting
44. The chance of winning the race of the horse A is 1/5 and
head and tail (HTH) or tail and head (THT) alternatively ?
that of horse B is 1/6. What is the probability that the race
(a) 1/4 (b) 1/5
(c) 1/6 (d) 1/8 will be won by A or B?
37. The probability that a student passes in mathematics is 4/9 (a) 1/30 ( b) 1/3
and that he passes in physics is 2/5. Assuming that passing in (c) 11/30 (d) 1/15
mathematics and physics are independent of each other, what 45. What is the probability of two persons being born on the
is the probability that he passes in mathematics but fails in same day (ignoring date)?
physics? (a) 1/49 (b) 1/365
4 8 (c) 1/7 (d) 2/7
(a) (b) 46. If A and B are two mutually exclusive and exhaustive events
15 45
26 19 with P(B) = 3P(A), then what is the value of P B ?
(c) (d)
45 45
(a) 3/4 (b) 1/4
38. From a pack of 52 cards, two cards are drawn, the first being
(c) 1/3 (d) 2/3
replaced before the second is drawn. What is the probability
47. The probabilities of two events A and B are given as
that the first is a diamond and the second is a king?
P (A) = 0.8 and P (B) = 0.7. What is the minimum value of
1 4
(a) (b) P A B ?
4 13
(a) 0 (b) 0.1
1 4
(c) (d) (c) 0.5 (d) 1
52 15

PDF Download FROM >> WWW.SARKARIPOST.IN Join >> https://www.facebook.com/Sarkaripost.in/


WWW.SARKARIPOST.IN
618 Quantitative Aptitude

48. In tossing three coins at a time, what is the probability of 56. A man and his wife appear for an interview for two posts.
getting at most one head? 1
3 7 The probability of the husband's selection is and that of
7
(a) (b)
8 8 1
1 1 the wife’s selection is . The probability that only one of
5
(c) (d)
2 8 them will be selected is
49. Two balls are selected from a box containing 2 blue and 7
6 4
red balls. What is the probability that at least one ball is (a) (b)
blue? 7 35
2 7 6 2
(a) (b) (c) (d)
9 9 35 7
5 7
(c) (d) 57. The probability that a person will hit a target in shooting
12 12 practice is 0.3. If he shoots 10 times, the probability that he
x
50. The probability of guessing a correct answer is . If the hits the target is
12
(a) 1 (b) 1 – (0.7)10

WWW.SARKARIPOST.IN
2
probability of not guessing the correct answer is , then (c) (0.7)10 (d) (0.3)10
3
what is x equal to? 58. Suppose six coins are tossed simultaneously. Then the
(a) 2 (b) 3 probability of getting at least one tail is
(c) 4 (d) 6
71 53
51. If A and B are two mutually exclusive events, then what is (a) (b)
72 54
P(AB) equal to?
(a) 0 (b) P(A) + P(B) 63 1
(c) (d)
64 12
B
(c) P(A) P(B) (d) P(A) P 59. In a given race the odds in favour of three horses A, B, C
A are 1 : 3; 1 : 4; 1 : 5 respectively. Assuming that dead head
52. If P(E) denotes the probability of an event E, then E is is impossible the probability that one of them wins is
called certain event if :
7 37
(a) P(E) = 0 (b) P(E) = 1 (a) (b)
(c) P(E) is either 0 or 1 (d) P(E) = 1/2 60 60
53. A programmer noted the results of attempting to run 20 1 1
programs. The results showed that 2 programs ran correctly (c) (d)
5 8
in the first attempt, 7 ran correctly in the second attempt, 5
ran correctly in the third attempt, 4 ran correctly in the fourth 60. The probability that the 13th day of a randomly chosen
attempt and 2 ran correctly in the fifth attempt. What is the month is a Friday, is
probability that his next programme will run correctly on 1 1
the third run ? (a) (b)
12 7
1 1
(a) (b) 1 1
4 3 (c) (d)
1 1 84 13
(c) (d)
6 5 61. In a single throw with four dice, the probability of throwing
54. The digits 1, 2, 3, 4, 5, 6, 7, 8, 9 are written in random order seven is
to form a nine digit number. Find the probability that this 4 8
number is divisible by 4: (a) 4 (b)
6 64
4 2
(a) (b)
9 9 16 20
(c) 4 (d)
17 6 64
(c) (d) None of these
81 62. Six dice are thrown. The probability that different number
55. Two cards are drawn from a pack of 52 cards. The will turn up is
probability that either both are red or both are kings is
1 1 129 1
(a) (b) (a) (b)
2 321 1296 54
325 5 5
(c) (d) None of these (c) (d)
1326 324 54

PDF Download FROM >> WWW.SARKARIPOST.IN Join >> https://www.facebook.com/Sarkaripost.in/


WWW.SARKARIPOST.IN
Probability 619

63. If A and B are events such that P(A |B) = P(B |A), then 71. A and B play a game where each is asked to select a number
(a) A B but A B (b) A = B from 1 to 25. If the two numbers match, both of them win a
(c) A B = (d) P(A) = P(B) prize. The probbility that they will not win a prize in a single
64. If two dice are tossed, find the probability of throwing a trial is
total of ten or more. 1 24
1 1 (a) (b)
(a) (b) 25 25
6 3
1 2 2
(c) (d) (c) (d) None of these
4 3 25
65. From a pack of 52 cards two are drawn with replacement.
72. In a box containing 100 bulbs, 10 are defective. The
The probability, that the first is a diamond and the second is probability that out of a sample of 5 bulbs, none is defective
a king, is is
(a) 1/26 (b) 17/2704
5
(c) 1/52 (d) None of these 1
(a) 10–1 (b)

WWW.SARKARIPOST.IN
66. Two cards are selected at random from a deck of 52 playing 2
cards. The probability that both the cards are greater than 2
but less than 9 is 5
9 9
46 63 (c) (d)
(a) (b) 10 10
221 221
81 93 73. The probability of happening an event A in one trial is 0.4.
(c) (d) The probability that the event A happens at least once in
221 221
three independent trials is –
67. If A and B are two independent events such that
(a) 0.936 (b) 0.216
P(a) = 1/6 and P(b) = 1/2, then
(c) 0.904 (d) 0.784
(a) P(AB) = 1/24 (b) P (A B) = 1/12 74. Find the probability of drawing a jack or an ace from a
(c) P (AcB) = 1/6 (d) P(AcBc) = 5/12 pack of playing cards.
1 1
1 (a) (b)
68. If A and B are two independent events such that P ( A) 8 6
2 1 2
1 (c) (d)
and P ( B ) , then which is not true? 3 13
5 75. When two dice are thrown, the probability that the difference
3 A 1 of the numbers on the dice is 2 or 3 is
(a) P( A B) (b) P
5 B 4 7 3
(a) (b)
18 11
5 5 1
(c) P( A / A B) (d) P( A B/ A B) 0 (c) (d)
6 18 2
76. In shuffling a pack of cards three are accidentally dropped.
69. A card is drawn from a pack of 52 cards. A gambler bets
The probability that the missing cards are of distinct colours
that it is a spade or an ace. What are the odds against his is
winning this bet?
(a) 17 : 52 (b) 52 : 17 169 165
(a) (b)
(c) 9 : 4 (d) 4 : 9 425 429
1
70. The probability that a man will live 10 more years is and 162 164
4 (c) (d)
1 459 529
the probability that his wife will live 10 more years is . 77. A quadratic equation is chosen from the set of all quadratic
3
Then the probability that neither will be alive in 10 years is equations which are unchanged by squaring their roots. The
chance that the chosen equation has equal roots is
5 7
(a) (b) 1 1
12 12 (a) (b)
2 3
1
1 11 (c) (d) None of these
(c) (d) 4
2 12

PDF Download FROM >> WWW.SARKARIPOST.IN Join >> https://www.facebook.com/Sarkaripost.in/


WWW.SARKARIPOST.IN
620 Quantitative Aptitude

78. Four persons are selected at random out of 3 men, 2 women Directions for Questions 81 and 82:
and 4 children. The probability that there exactly 2 children An urn contains one black ball and one green ball. A second urn
in the selection is contains one white and one green ball. One ball is drawn at random
11 9 from each urn.
(a) (b) 81. What is the probability that both balls are of same colour ?
21 21
10 (a) 1/2 (b) 1/3
(c) (d) None of these
21 (c) 1/4 (d) 2/3
79. It is given that the events A and B are such that 82. What is the probability of getting at least one green ball ?
1 1 2 (a) 1/2 (b) 1/3
P ( A) , P( A | B ) and P ( B | A) . Then P(B) is (c) 2/3 (d) 3/4
4 2 3
1 1 83. The probability that a student will pass in Mathematics is
(a) (b) 3/5 and the probability that he will pass in English is 1/3. If
6 3
the probability that he will pass in both Mathematics and
2 1
(c) (d) English is 1/8, what is the probability that he will pass in at
3 2
least one subject?

WWW.SARKARIPOST.IN
80. A coin is tossed and a dice is rolled. The probability that the
coin shows the head and the dice shows 6 is 97 87
(a) (b)
1 1 120 120
(a) (b)
2 6 53 120
(c) (d)
1 1 120 297
(c) (d)
12 24

PDF Download FROM >> WWW.SARKARIPOST.IN Join >> https://www.facebook.com/Sarkaripost.in/


WWW.SARKARIPOST.IN
Probability 621

Standard Level
1. A bag contains 6 red and 4 green balls. A fair dice is rolled 2
and a number of balls equal to that appearing on the dice is 7. The probability that A can solve a problem is and B can
3
chosen from the urn at random. The probability that all the 3
balls selected are red is solve it is . If both attempt the problem, what is the
4
probability that the problem gets solved?
1 3
(a) (b) 11 7
3 10
(a) (b)
12 12
1 1
(c) (d) 5 9
8 5 (c) (d)
12 12

WWW.SARKARIPOST.IN
2. If P(A) = 0.8, P(B) = 0.9, P(AB) = p, which one of the
8. A speaks the truth in 70 percent cases and B in 80 percent.
following is correct?
The probability that they will contradict each. other when
(a) 0.72 p 0.8 (b) 0.7 p 0.8 describing a single event is
(c) 0.72 < p < 0.8 (d) 0.7 < p < 0.8 (a) 0.36 (b) 0.38
3. A, B, C are three mutually exclusive event associated with (c) 0.4 (d) 0.42
a random experiment. Find P(A) if it is given that 9. If A an d B are two independent events and
P(B) = 3/2 P(A) and P(C) = 1/2 P(B). P(C) = 0, then A, B, C are :
(a) independent
4 2 (b) dependent
(a) (b)
13 3 (c) not pairwise independent
(d) None of these
12 1 10. A dice is thrown 6 times. If ‘getting an odd number’ is a
(c) (d)
13 13 ‘success’, the probability of 5 successes is
4. Two cards are drawn at random from a well-shuffled pack
3
of 52 cards. What is the probability that either both are red (a) (b)
10 32
or both are queens?
5 25
17 55 (c) (d)
(a) (b) 6 26
112 221 11. A bag contains 5 white and 3 black balls, and 4 are
55 33 successively drawn out and not replaced. What’s the chance
(c) (d) of getting different colours alternatively?
221 221
5. Four cards are drawn at a time from a pack of 52 playing 1 1
(a) (b)
cards. Find the probability of getting all the four cards of 6 5
the same suit.
1 1
5 12 (c) (d)
(a) (b) 4 7
13 65
12. A bag contains 5 white and 7 black balls and a man draws
44 44 4 balls at random. The odds against these being all black is
(c) (d)
4165 169 (a) 7 : 92 (b) 92 : 7
6. Two persons A and B throw a die alternatively till one of (c) 92 : 99 (d) 99 : 92
them gets a three and wins the game. Find their respective 13. The letters of the word SOCIETY are placed at random in a
probabilites of winning. row. The probability that the three vowels come together is

6 5 5 8 1 1
(a) , (b) , (a) (b)
11 11 11 11 6 7

3 7 8 3 2 5
(c) , (d) , (c) (d)
11 11 11 11 7 6

PDF Download FROM >> WWW.SARKARIPOST.IN Join >> https://www.facebook.com/Sarkaripost.in/


WWW.SARKARIPOST.IN
622 Quantitative Aptitude

14. Course materials are sent to students by a distance education 21. Two dice are thrown simultaneously. What is the probability
institution. The probability that they will send a wrong of obtaining a multiple of 2 on one of them and a multiple
1 of 3 on the other
programme’s study material is . There is a probability of
5 5 11
3 (a) (b)
that the package is damaged in transit, and there is a 36 36
4
1 1 1
probability of that there is a short shipment. What is the (c) (d)
3 6 3
probability that the complete material for the course arrives 22. Two dice are thrown at a time, find the probability that the
without any damage in transit ? sums of the numbers on the upper faces of the dice are equal
4 8 to 7.
(a) (b) 1 1
5 60
(a) (b)
8 4
8 4
(c) (d) 1 1
15 20 (c) (d)
3 6
15. A coin is tossed 5 times. What is the probability that head 23. If 4 whole numbers are taken at random, and multiplied 27

WWW.SARKARIPOST.IN
appears an odd number of times?
together, the chance that the last digit in the product is 1, 3,
2 1 7 or 9 will be
(a) (b)
5 5 (a) 13/976 (b) 17/529
1 4 (c) 16/625 (d) 13/625
(c) (d)
2 25 24. One card is drawn from a well-shuffled pack of 52 cards.
16. Two dice are tossed. The probability that the total score is a What is the probability, that it is not the ace of hearts ?
prime number is 51 1
(a) (b)
1 5 52 52
(a) (b) 1 1
6 12 (c) (d)
12 2
1 7 25. A dice is thrown twice. The probability of getting 4, 5 or 6
(c) (d)
2 9
in the first throw and 1, 2, 3 or 4 in the second throw is
17. The probability that the sum of the square of the two (a) 1/3 (b) 2/3
numbers, which show up when two fair dice are thrown, is
(c) 1/2 (d) 1/4
even is
26. Ram and Shyam appear for an interview for two vacancies
3 4
(a) (b) in an organisation for the same post. The probabilities of
7 7 their selection are 1/6 and 2/5 respectively. What is the
5 probability that none of them will be selected?
(c) (d) None of these
7 (a) 5/6 (b) 1/5
18. There are 5 pairs of shoes in a cupboard from which 4 shoes (c) 1/2 (d) 3/5
are picked at random. The probability that there is at least 27. Three groups of children contain respectively 3 girls and 1
one pair is boy, 2 girls and 2 boys and 1 girl and 3 boys. One child is
8 11 selected at random from each group. The probability that
(a) (b)
21 21 the three selected consists of 1 girl and 2 boys is
13 12
13 12 (a) (b)
(c) (d) 32 32
21 31
15 11
19. A die is rolled three times, find the probability of getting a (c) (d)
32 32
larger number than the previous number each time. 28. A class consists of 80 students, 25 of them are girls and 55
(a) 5/24 (b) 1/24 are boys. If 10 of them are rich and the remaining poor and
(c) 5/54 (d) 1/8
also 20 of them are intelligent then the probability of
20. The fair dice are thrown. The probability that the number
selecting an intelligent rich girl is
appear are not all distinct is
5 4 5 25
(a) (b)
(a) (b) 128 128
9 9
1 5 5
(c) (d) (c) (d) None of these
6 6 512

PDF Download FROM >> WWW.SARKARIPOST.IN Join >> https://www.facebook.com/Sarkaripost.in/


WWW.SARKARIPOST.IN
Probability 623

29. If the probability of A to fail in an examination is 0.2 and of 100 parts may be defective. Similarly, 5 out of 100 are
that for B is 0.3, then probability that either A or B is fail, likely to be defective int he manufacture of the part. Y.
is : Calculate the probability that the assembled product will
(a) 0.5 (b) 0.44 not be defective.
(c) 0.8 (d) 0.25 (a) 0.6485 (b) 0.6565
30. The probability of choosing at random a number that is (c) 0.8645 (d) None of these
divisible by 6 or 8 from among 1 to 90 is equal to
37. If P(A) = 3/7, P(B) = 1/2 and P A B 1/14 , then are
1 1
(a) (b) A and B are mutually exclusive events?
6 30
(a) No (b) Yes
11 23 (c) Either yes or no (d) Cannot be determined
(c) (d) 38. Michael Jordan’s probability of hitting any basketball shot
80 90
in three times than mine, which never exceesds a third. To
31. In single cast with two dice the odds against drawing 7 is
beat him in a game, I need to hit a shot myself and have
1
(a) 5 (b) Jordan miss the same shot. If I pick my shot optimally, what
5 is the maximum probability of winning which I can attain?

WWW.SARKARIPOST.IN
1
(c) 6 (d) 1 1
6 (a) (b)
32. From a group of 7 men and 4 women a committee of 6 16 12
persons is formed. What is the probability that the committee
5
will consist of exactly 2 women? (c) (d) 1/14
6
5 3
(a) (b) 39. Triangles are formed by joining vertices of an octagon. Any
11 11 one of those triangle is selected at random. What is the
4 2 probability that the selected triangle has no side common
(c) (d) with the octagon?
11 11
(a) 3/7 (b) 2/7
33. A natural number is chosen at random from the first 100
(c) 5/7 (d) 1/7
natural numbers. What is the probability that the number
40. One bag contains 4 white balls and 2 black balls. Another
chosen is a multiple of 2 ro 3 or 5?
bag contains 3 white balls and 5 black balls. If one ball is
30 1 drawn from each bag, determine the probability that one
(a) (b)
100 33 ball is white and another is black.
(a) 6/24 (b) 5/24
74 7
(c) (d) (c) 7/24 (d) 13/24
100 10
41. A woman goes to visit the house of some friend whom she
34. Two numbers a and b are chosen at random from the set of
has not seen for many years. She knew that besides the two
first 30 natural numbers. The probability that a2 – b2 is
married adults in the household, there are two children of
divisible by 3 is:
different ages. But she does not knew their genders. When
37 47 she knocks on the door of the house, a boy answers. What
(a) (b)
87 87 is the probability that the younger child is a boy?
17 2 1
(c) (d) None of these (a) (b)
29 3 2
35. A problem is given to three students whose chances of
1 1
1 1 1 (c) (d)
solving it are , and respectively. What is the 3 4
2 3 4
42. The odds against P solving a problem are 8 : 6 and odds in
probability that the problem will be solved ?
favour of Q solving the same problem are 14 : 10 The
1 1 probability of the problem being solved, if both of them try
(a) (b)
4 2 it, is
3 7 5 16
(c) (d) (a) (b)
4 12 21 21
36. An article manufactured by a company consists of two parts
5 5
X and Y. In the process of manuifacture of the part X, 9 out (c) (d)
12 7

PDF Download FROM >> WWW.SARKARIPOST.IN Join >> https://www.facebook.com/Sarkaripost.in/


WWW.SARKARIPOST.IN
624 Quantitative Aptitude

2 50. A three digit number is written down by random choice of


43. The probability that A can solve a problem is and B can the digits 1 to 9 with replacements. The probability that
3
3 atleast one of the digits chosen is a perfect square is
solve it is . If both attempt the problem, what is the 8 4
4 (a) (b)
probability that the problem gets solved? 27 9
2
11 7 (c) (d) None of these
(a) (b) 9
12 12 51. 4 gentlemen and 4 ladies take seats at random round a table.
5 9 The probability that they are sitting alternately is
(c) (d) (a) 4/35 (b) 1/70
12 12
(c) 2/35 (d) 1/35
44. Atul can hit a target 3 times in 6 shots, Bhola can hit the
target 2 times in 6 shots and Chandra can hit the 4 times in 52. Two cards are drawn one by one from a pack of cards. The
4 shots. What is the probability that at least 2 shots (out of probability of getting first card an ace and second a coloured
1 shot taken by each one of them) hit the target ? one is (before drawing second card, first card is not placed
again in the pack) :

WWW.SARKARIPOST.IN
1 2
(a) (b) (a) 1/26 (b) 5/52
2 3
(c) 5/221 (d) 4/13
1 5 53. Seven people seat themselves indiscriminately at round table.
(c) (d)
3 6 The probability that two distinguished persons will be next to
45. Suppose six coins are tossed simultaneously. Then the each other is
probability of getting at least one tail is :
1 1
71 53 (a) (b)
(a) (b) 3 2
72 54

63 1 1 2
(c) (d) (c) (d)
64 12 4 3
3 1 54. The odds against A solving a certain problem are 3 to 2 and
46. A and B are events such that P(A B) = , P(A B) = , the odds in favour of B solving the same problem are 2 to 1.
4 4
2 The probability that the problem will be solved if they both
P( A ) = then P ( A B) is
3 try, is
5 3 2 11
(a) (b) (a) (b)
12 8 5 15
4 2
5 1 (c) (d)
5 3
(c) (d)
8 4 55. If two events A and B are such that P ( A) 0.3, P ( B ) 0.4
47. Seven digits from the numbers 1, 2, 3, 4, 5, 6, 7, 8, 9 are
written in a random order. The probability that this seven and P ( A
B
B) 0.5 then P
digit number is divisible by 9 is A B
2 7
(a) (b) (a) 0.9 (b) 0.5
9 36
1 7 (c) 0.6 (d) 0.25
(c) (d)
9 12 56. If A and B are two events such that P ( A) 0 and P ( B ) 1 ,
48. A committee of 5 Students is to be chosen from 6 boys and
4 girls. Find the probability that the committee contains A
exactly 2 girls. then P
B
(a) 10/21 (b) 11/21
(c) 12/21 (d) 13/21
49. There are 10 envelopes and 10 letters to go inside them. A A
(a) 1 – P (b) 1– P
Each letter is meant for a specified envelope only. What is B B
the probability that exactly 9 of them are in the right
envelopes ? 1 – P ( A B) P ( A)
(c) (d)
(a) 1/10! (b) 1 P( B ) P(B )
(c) 0 (d) None of these

PDF Download FROM >> WWW.SARKARIPOST.IN Join >> https://www.facebook.com/Sarkaripost.in/


WWW.SARKARIPOST.IN
Probability 625

57. Let 0 < P (A) < 1, 0 < P (B) < 1 and 61. 12 persons are seated around a round table. What is the
P( A B) P( A) P( B) P ( A) P ( B ) , then : probability that two particular persons sit together?

(a) P ( B / A) P( B) P ( A) 2 1
(a) (b)
11 6
(b) P ( A ' B ') P ( A ') P ( B ')
3 3
(c) P( A B) P( A ') P( B ') (c) (d)
11 15
(d) None of these 62. The probability of a bomb hitting a bridge is 1/2 and two
58. A group of investigators took a fair sample of 1972 children direct hits are needed to destroy it. The least number of
from the general population and found that there are 1000 bombs required so that the probability of the bridge being
boys and 972 girls. If the investigators claim that their destroyed is greater than 0.9 is:
research is so accurate that the sex of a new born child can
(a) 7 bombs (b) 3 bombs
be predicted based on the ratio of the sample of the
(c) 8 bombs (d) 9 bombs
population, then what is the expectation in terms of the
63. A number is choosen at random from the numbers 10 to 99.
probability that a new child born will be a girl?
By seeing the number, a man will sing if the product of the

WWW.SARKARIPOST.IN
243 250 digits is 12. If he chooses three numbers with replacement,
(a) (b) then the probability that he will sing at least once is:
250 257
3 3
9 243 43 43
(c) (d) (a) 1 (b)
10 493 45 45
59. A life insurance company insured 25,000 young boys,
14,000 young girls and 16,000 young adults. The probability 48 86
(c) 1 (d) None of these
of death within 10 years of a young boy, young girl and a 903
young adult are 0.02, 0.03 and 0.15 respectively. One of 64. Probabilities that Rajesh passes in Maths, Physics and
the insured persons dice. What is the probability that the Chemistry are m, p and c respectively. Of these subjects,
dead person is a young boy? Rajesh has a 75% chance of passing in at least one, 50%
36 25 chance of passing in at least two and 40% chance of passing
(a) (b) in exactly two. Find which of the following is true.
165 166
19 27
26 32 (a) p + m + c = (b) p+m+c=
(c) (d) 20 20
165 165
60. Eleven books, consisting of five Engineering books, four 1 1
(c) pmc = (d) pmc =
Mathematics books and two Physics books, are arranged in 20 8
a shelf at random. What is the probability that the books of 65. Two small squares on a chess board are choosen at random.
each kind are all together? Find the probability that they have a common side:
5 2 1 1
(a) (b) (a) (b)
1155 1155 12 18
3 1 2 3
(c) (d) (c) (d)
1155 1155 15 14

PDF Download FROM >> WWW.SARKARIPOST.IN Join >> https://www.facebook.com/Sarkaripost.in/


WWW.SARKARIPOST.IN
626 Quantitative Aptitude

Expert Level
1. There are 6 positive and 8 negative numbers. Four numbers 7. Two dies are thrown n times in succession. The probability
are chosen at random and multiplied. The probability that of obtaining double – six atleast once is
the product is a positive number is
n
1 35
(a) 500
(b)
503 (a) (b) 1
1001 36 36
1001
505 101 n n
(c) (d) 1 35
1001 1001 (c) (d)
36 36
2. The probability of getting 10 in a single throw of three fair
dice is 8. Two cards are drawn from a well shuffled deck of 52 cards.
The probability that one is a red card and the other is a

WWW.SARKARIPOST.IN
1 1
(a) (b) queen is
6 8
1 101 4
(c) (d) None of these (a) (b)
9 1326 51
3. In an examination, there are 500 students, 150 passed the
first paper and 350 passed the second paper. 50 students 16
passed both the papers. Find the probability that a student (c) (d) None of these
221
selected at random has failed in both the papers.
(a) 1/5 (b) 1/10 9. Each of two persons tosses three fair coins. The probability
that they obtain the same number of heads is
(c) 3/10 (d) 3/5
4. A, B and C shoot to hit a target. If A hits the target 4 times in 1 5
5 trials, B hits it 3 times in 4 trials and C hits it 2 times in 3 (a) (b)
2 16
trials. What is the probability that the target is hit by atleast
2 persons? 7
(c) (d) None of these
5 3 16
(a) (b)
6 4 10. There are three events E1, E2 and E3 one of which must,
and only one can happen. The odds are 7 to 4 against E1
4 1
(c) (d) and 5 to 3 against E2. The odds against E3 is
5 9 (a) 4 : 11 (b) 3 : 8
5. A bag contains 5 red and 4 green balls and another bag (c) 23 : 88 (d) 65 : 23
contains 3 red and 7 black balls. If a ball is drawn from 11. If A and B are two events, the probability that at most one
each bag. Find the probability that both are of different of these events occurs is :
colours. (a) P ( A ') P ( B ') P ( A ' B ')
47 5 (b) P ( A ') P ( B ') P ( A B) 1
(a) (b)
90 6
(c) P ( A B ') P( A ' B ) P( A ' B ')
7 2 (d) All above are correct.
(c) (d) 12. A and B are two independent events. The probability that
18 15
6. A MNC has two Grids – Grid I and Grid II. Out of 5 1
both A and B occur is and the probability that neither of
Directors and 4 General Managers of Grid I, one person is 6
transferred to Grid II having already 3 Directors and 7
General Managers. If, one person superannuates from Grid 1
them occurs is . The probability of occurrence of A is.
II, then the probability that this person was a director is 3
32 4 1 1
(a) (b) (a) (b)
99 45 2 3
20 3 5 1
(c) (d)
99 10 (c) (d)
6 6

PDF Download FROM >> WWW.SARKARIPOST.IN Join >> https://www.facebook.com/Sarkaripost.in/


WWW.SARKARIPOST.IN
Probability 627

13. In each of a set of games it is 2 to 1 in favour of the winner 20. The probability that when 12 balls are distributed among
of the previous game. The chance that the player who wins three boxes, the first will contain three balls is,
the first game shall win three at least of the next four is
12
8 4 29 C3 .29
(a) (b) (a) (b)
27 81 312 312
4 12
(c) (d) None C3 .212
9 (c) (d) None of these
14. Given two bags A and B as follows : Bag A contains 3 red 312
and 2 white balls and bag B contains 2 red and 5 white 21. If n integers taken at random are multiplied together, then
balls. A bag is selected at random, a ball is drawn and put the probability that the last digit of the product is 1, 3, 7 or
into the other bag, then a ball is drawn from the second bag. 9 is
The probability that both balls drawn are of the same colour 2n 4n 2n
is (a) (b)
5n 5n
187 901
4n

WWW.SARKARIPOST.IN
(a) (b)
1680 1680 (c) (d) None of these
5n
439
(c) (d) None of these 22. The probabilities of four cricketers A, B, C and D scoring
1680
15. A positive integer N is selected such that 100 < N < 200. 1 1 1 1
more than 50 runs in a match are , , and . It is
The probability that it is divisible by either 4 or 7 is : 2 3 4 10
38 24 known that exactly two of the players scored more than 50
(a) (b) runs in a particular match. The probability that these players
99 99
were A and B is
34 14
(c) (d)
99 99 27 5
16. If three vertices of a regular hexagon are chosen at random, (a) (b)
65 6
then the chance that they form an equilateral triangle is :
1 1 1
(a) (b) (c) (d) None of these
3 5 6
1 1
(c) (d) 23. From a bag containing 4 white and 5 black balls a man
10 2 draws 3 at random; what are the odds against these being
17. Two dice are thrown. The probability that the sum of the all black?
numbers coming up on them is 9, if it is known that the
(a) 37 : 5 (b) 5 : 37
number 5 always occurs on the first die, is
(c) 23 : 19 (d) 19 : 23
1 1 24. One hundred cards are numbered from 1 to 100. Find the
(a) (b)
6 3 probability that a card chosen at random has the digit 5.
2 1 19 11
(c) (d) (a) (b)
3 2 100 100
12 1
1 4p 1 p 1 2p (c) (d)
18. If , and are the probabilities of three 100 100
4 2 2 25. Two sisters A and B appeared for an audition. The probability
mutually exclusive events, then value of p is
1 1 1 2
(a) (b) of selection of A is and that of B is Find the probability
2 3 5 7
1 2 that both of them are selected.
(c) (d)
4 3 (a) 2/35 (b) 1/35
19. The probability of getting 10 in a single throw of three fair (c) 4/35 (d) 7/35
dice is 26. A 5 digit number is formed by using the digits 0, 1, 2, 3, 5
1 1 and 5 without repetition. The probability that the number is
(a) (b)
6 8 divisible by 6 is
1 1 (a) 0.08 (b) 0.17
(c) (d) (c) 0.18 (d) 0.36
9 5

PDF Download FROM >> WWW.SARKARIPOST.IN Join >> https://www.facebook.com/Sarkaripost.in/


WWW.SARKARIPOST.IN
628 Quantitative Aptitude

27. If M and N are any two events. The probability, that exactly 34. A book has 999 pages. If a page is opened at random the
one of them occurs, is probability that the sum of the digits in its number is 9, is
(a) P(M) + P(N) – P ( M N) 55 22
(a) (b)
(b) P(M) + P(N) + P ( M N) 999 333
(c) P(M) + P(N)
117 4
(d) P(M) + P(N) – 2 P ( M N ) (c) (d)
999 37
28. It is known that at noon at a certain place the sun is hidden
35. Two integers x and y are chosen with replacement out of the
by clouds on an average two days out of every three. The
set {0, 1, 2, 3, . .........10}. Then the probability that | x – y | >5
chance that the sun will be shinning at noon on at least four
is
out of five specified future days is
1 16 81 30
(a) (b) (a) (b)
81 243 121 121
11 7
(c) (d) 25 20
243 20 (c) (d)

WWW.SARKARIPOST.IN
29. A, B, C in order, cut a pack of cards, replacing them after 121 121
each cut, on condition that the first who cuts a spade shall 36. The coefficients a and b in the quadratic equation x2 + ax +
win a prize. Then A’s chance of winning is b = 0 are the numbers that appear when a pair of fair dice is
tossed. The probability that the roots are real is-
16 12
(a) (b) 15 17
37 37 (a) (b)
36 36
9 14
(c) (d) 19 21
37 37 (c) (d)
36 36
30. What is the probability that any two different cards of a
37. Four whole numbers taken at random are multiplied
well-shuffled deck of 52 cards will be together in the deck
together. What is the chance that the last digit in the product
if their suit is not considered?
is 1, 3, 7 or 9 ?
9 10
(a) (b) 16 1
11 3 (a) (b)
625 210
8 10 8 4
(c) (d) (c) (d)
13 11 125 25
31. Three integers are chosen at random from the first 20 38. A die is thrown 7 times. The chance that an odd number
integers. The probability that their product is even, is turns up at least 4 times, is
2 3 (a) 1/4 (b) 1/2
(a) (b) (c) 1/8 (d) None of these
19 29
39. A letter is taken from the word ASSISTANT and another
17 4 from the word STATISTICS. What is the probability that
(c) (d)
19 29 both the letters are the same ?
32. The probability that two integers chosen at random and their 1 17
(a) (b)
product will have the same last digit is : 45 90
3 1 19 13
(a) (b) (c) (d)
10 25 90 90
40. A box contains 3 white and 2 red balls. If first drawing ball
4 7
(c) (d) is not replaced then the probability that the second drawing
15 15 ball will be red is
33. The probability that the birth days of six different persons (a) 8/25 (b) 2/5
will fall in exactly two calendar months is
(c) 3/5 (d) 21/35
1 12C ×
26 41. An event X can happen with probability P, and event Y can
(a) (b) 2 happen with probability P . What is the probability that
6 12 6
exactly one of them happens?
12C × 26 – 1 341 (a) P + P – 2PP (b) 2PP – P + PP
(c) 2 (d) (c) P – P + 2PP (d) 2P P – P + P
12 6
12 5

PDF Download FROM >> WWW.SARKARIPOST.IN Join >> https://www.facebook.com/Sarkaripost.in/


WWW.SARKARIPOST.IN
Probability 629

42. In a bombing of the Nathula pass, the Indian troops have to 44. Two integers x and y are chosen with replacement out of
destroy a bridge on the pass. The bridge is such that it is the set {0, 1, 2, 3...10}. Then the probability that |x – y| > 5
destroyed when exactly 2 bombs hit it. A MIG-27 is is:
dispatched in order to do the bombs hit it. A MIG-27 is 7 40
(a) (b)
dispatched in order to do the bombing. Flt Lt. Rakesh 11 121
Sharma needs to ensure that there is at least 97% probability 35 30
(c) (d)
for the bridge to be destroyed. He knows that when he drops 121 121
a bomb on the bridge the probability of the bomb hitting 45. A consignment of 15 wristwatches contain 4 defectives.
the bridge is 90%. Weather conditions and visibility being The wristwatches are selected at random, one by one and
poor he is unable to see the bridge from his plane. How examined. The ones examined are not put back. What is the
many bombs does he need to drop to be 95% sure that the probability that ninth one examined is the last defective?
bridge will be destroyed? 11 17
(a) (b)
(a) 3 (b) 4 195 195
(c) 5 (d) 6 8 16
(c) (d)

WWW.SARKARIPOST.IN
43. 20 girls, among whom are A and B sit down at a round 195 195
table. The probability that there are 4 girls between A and B 46. A letter is takenout at random from ‘ASSISTANT’ and
is: another is taken out from ‘STATISTICS’. The probability
17 2 that they are the same letters is:
(a) (b) 35 19
19 19
(a) (b)
87 90
13 6
(c) (d) 19
19 19 (c) (d) None of these
96

PDF Download FROM >> WWW.SARKARIPOST.IN Join >> https://www.facebook.com/Sarkaripost.in/


WWW.SARKARIPOST.IN
630 Quantitative Aptitude

Test Yourself
1. In four schools B1, B2, B3, B4 the percentage of girls students 4
, while the
is 12, 20, 13, 17 respectively. From a school selected at 6. The probability that A speaks truth is
5
random, one student is picked up at random and it is found 3
that the student is a girl. The probability that the school probability for B is . The probability that they contradict
4
selected is B2, is each other when asked to speak on a fact is
4 1
6 10 (a) (b)
(a) (b) 5 5
31 31
7 3
(c) (d)
13 17 20 20
(c) (d) 7. A bag contains 5 black and 3 red balls. A ball is taken out
62 62

WWW.SARKARIPOST.IN
from the bag and is not returned to it. If this process is
2. The probability that a leap year will contain 53 Sunday is
repeated three times, then what is the probability of drawing
1 2 a black ball in the next draw of a ball?
(a) (b)
7 7 (a) 0.7 (b) 0.625
4 5 (c) 0.1 (d) None of these
(c) (d) 8. If two squares are chosen at random on a chess board, the
7 7
probability that they have a side in common is
3. A bag contains 3 red and 7 black balls, two balls are taken
out at random, without replacement, If the first ball taken 1 2
out is red, then the probability that the second taken out red (a) (b)
9 7
ball is
1 2
1 1 (c) (d)
18 9
(a) (b)
10 15 9. In a horse race the odds in favour of three horses are 1 : 2,
1 : 3 and 1 : 4. The probability that one of the horse will win
3 2
(c) (d) the race is
10 21
37 47
4. The probabilities of a problem being solved by two students (a) (b)
60 60
1 1 1 3
A and B are , respectively. Then the probability of the (c) (d)
2 3 4 4
problem being solved is : 10. The probability that a contractor will get a plumbing contract
2 4 is 2/3 and the probability that he will get an electric contract
(a) (b) is 5/9. If the probability of getting at least one contract is
3 3
4/5, what is the probability that he will get both the contracts?
1 19 13
(c) (d) 1 (a) (b)
3 45 45
5. The ratio of number of officers and ladies in the Scorpion 12 11
Squadron and in the Gunners Squadron are 3 : 1 and 2 : 5 (c) (d)
35 23
respectively. An individual is selected to be the chairperson
11. One hundred identical coins each with probability P of
of their association. The chance that this individual is
showing up Heads are tossed once. If 0 < P < 1 and the
selected from the Scorpions is 2/3. Find the probability that
probability of Heads showing on 50 coins is equal to that of
the chairperson will be an officer.
Heads showing on 51 coins, then value of P is
25 13 1 49
(a) (b) (a) (b)
42 43 21 101
11 7 50 51
(c) (d) (c) (d)
43 42 101 101

PDF Download FROM >> WWW.SARKARIPOST.IN Join >> https://www.facebook.com/Sarkaripost.in/


WWW.SARKARIPOST.IN
Probability 631

12. A bag contains 15 tickets numbered 1 to 15. A ticket is drawn 14. A pair of fair dice are rolled together till a sum of either
and replaced. Then one more ticket is drawn and replaced. 5 or 7 is obtained. The probability that 5 comes before
The probability that first number drawn is even and second 7 is
is odd is (a) 0.45 (b) 0.4
56 26
(a) (b) (c) 0.5 (d) 0.7
225 578
57 15. A book contains 1000 pages numbered consecutively. The
(c) (d) None of these probability that the sum of the digits of the number of a
289
13. A and B are two independent events. The probability that page is 9, is :
both A and B occur is 1/6 and the probability that neither of 55
them occurs is 1/3. The probability of occurrence of A is : (a) zero (b)
1000
5 1
(a) (b)
6 6 33 44
(c) (d)
1 1000 1000
(c) (d) None of these

WWW.SARKARIPOST.IN
5

PDF Download FROM >> WWW.SARKARIPOST.IN Join >> https://www.facebook.com/Sarkaripost.in/


WWW.SARKARIPOST.IN
632 Quantitative Aptitude

Hints & Solutions

Foundation Level The probability of a doublet not occurring at all in three


3
1. (a) When two are thrown then there are 6 × 6 exhaustive 5 125
cases n = 36. Let A denote the event “total score of throws = = .
6 216
7” when 2 dice are thrown then A = [(1, 6), (2, 5),
(3, 4), (4, 3), (5, 2), (6, 1)]. 125 91
Thus there are 6 favourable cases. Required probability = 1 – = .
216 216
m
m = 6 By definition P ( A) 8. (c) Required probability = 1/6.
n
6 1 9. (b) Total probable ways = 8
P ( A) .
36 6 Favourable number of ways = [HTH = THT]

WWW.SARKARIPOST.IN
2. (a) No of ways of drawing 2 white balls from 5 white
2 1
Hence required probability =
balls = 5 C2 . 8 4
Also, No of ways of drawing 2 other from remaining 7 10. (b) Any of the six numbers 1,2,3,4,5,6 may appear on the
balls = 7C2
upper face. n=6
Number of odd numbers = 3, since the odd numbers
Total number of balls = 12 are
5
C2 7
C2 14 1, 3, 5. m = 3.
Hence, required probability 12 The required probability
C4 33
3. (b) Total no. of outcomes when two dice are thrown = n number of favourable cases m 3 1
=
(S) = 36 and the possible cases for the event that the number of all cases n 6 2
sum of numbers on two dice is a prime number, are 11. (d)n = Number of all cases = 6
(1, 1), (1, 2), (1, 4), (1, 6), (2, 1), (2, 3), (2, 5), (3, 2), (3, 4), m = Number of favourable cases = 4 (since the numbers
(4, 1), (4, 3), (5, 1), (5, 6), (6, 1), (6, 5) that appear are 3, 4, 5, 6)
Number of outcomes favouring the event = n (A) = 15 m 4 2
The required probability = p
n A 15 5 n 6 3
Required probability 12. (a) S = (1, 2, 3, 4, 5, 6) n (S) = 6
n S 36 12
Let A be the event that the die shows a multiple of 2.
1 2 A = {2, 4, 6} n (A) = 3
4. (c) P(A) P (A )
3 3 n A 3 1
P (bird killed) = 1 – P (none of 3 shots hit) P(A) = n S = =
6 2
2 2 2 19 13. (a) India win atleast three matches
1 .
3 3 3 27 5 5 5 5
1 1 1 1
5. (b) Since, A and B are independent events = 5C3 + 5C4 + 5C5 =
2 2 2 2
A' and B' are also independent events
P ( A ' B ') P( A ').P ( B ') 1
(16) =
(0.4)(0.7) 0.28 2

P( A ') 1 P( A), P( B ') 1 P( B) 3 1


14. (d) P = ,q ,n 5
6. (d) 4 4
7. (d) Doublets occur when the numbers thrown are (1, 1), Required probability
(2, 2), . ............, (6, 6). Therefore the probability of a
3 2 4 5
3 1 5 3 1 5 3 459
6 1 = 5C3 C4 . C5 =
doublet occurring in single throw = = . 4 4 4 4 4 512
36 6

PDF Download FROM >> WWW.SARKARIPOST.IN Join >> https://www.facebook.com/Sarkaripost.in/


WWW.SARKARIPOST.IN
Probability 633

15. (b) Total no. of divisible by 4 between 1 to 80 21. (c) Total possible outcomes = 36
80 = 4 + (n – 1)4 E = Event of getting sum 7
80 = 4 n = {(1,6), (2,5), (3,4), (4,3), (5,2), (6,1)}
n = 20 F = Event of getting sum 11
20 = {(6,5), (5,6)}
C2 19
Required probability = = Total no. of favourable cases = 6 + 2 = 8
80 316
C2 Now required probability
16. (b) Here n (S) = 62 = 36 Total favourable cases 8 2
Let E be the event “getting sum more than 7” i.e. sum Total outcomes 36 9
of pair of dice = 8, 9, 10, 11, 12
22. (a) n(S) = 100
E = square of terms lies between 1 to 100.
(2, 6) (3,5) (4, 4) (5, 3) (6, 2)
= 1, 4, 9, 16, 25, 36, 49, 64, 81, 100
(3, 6) (4,5) (5, 4) (6,3) n(E) = 10
i.e., E =
(4, 6) (5,5) (6, 4)

WWW.SARKARIPOST.IN
n( E ) 10 1
(5, 6) (6,5) (6, 6) Required probability
n( S ) 100 10
n (E) = 15 23. (a) (AAAA), (LL), HBD
n( E ) 15 5 5! 5! 4! 2!
Required prob P
n( S ) 36 12 9! 9!
17. (c) Number of sample points on throwing two dice 4! 2!
= 6 × 6 = 36
24 2 1
The possible outcomes are
9 8 7 6 63
(1, 4), (2, 3), (3, 2), (4, 1)
The probability of obtaining a total score of 5 is 24. (a) Total number of ways of selecting 3 integers from 20
natural numbers = 20C3 = 1140
4 1
. Their product is a multiple of 3 means, at least one
6 6 9 number is divisible by 3.
18. (c) Given digits are 1, 2, 3, 4, 5 The numbers which are divisible by 3 are 3, 6, 9, 12,
Total no. of 2 digits numbers formed = (5)2 = 25 15, 18 and the number of ways of selecting atleast one
Favourable cases are 12, 24, 32, 44, 52
of them is 6 C1 14
C2 6
C2 14
C1 6
C3 776
No. of favourable cases = 5
5 1 776 194
Required Probability = Required Probability =
25 5 1140 285
19. (a) Total no. of cases = 63 = 216 25. (b) The probability that Krishna will be alive 10 years
16 can appear on three dice in following ways 7
(6, 6, 4), (6, 5, 5), (6, 4, 6), (4, 6, 6), (5, 5, 6), hence, is
15
(5, 6, 5). So, probability that Krishna will be dead 10 years
No. of favourable cases = 6 hence, the
6 1 7 8
Hence, the required probability 1–
6 3 36 15 15
Also, probability that Hari will be alive 10 years hence
20. (b) Total number of bolts = 600 7
Number of too large bolts = 20% of 600 is
10
20 600 So, the probability that Hari will be dead 10 years
120
100 7 3
hence, 1–
Number of too small bolts = 10% of 600 = 60 10 10
So, the probability that both Krishna and Hari will be
Number of suitable bolts = 600 – 120 – 60 = 420
dead 10 years hence
420 7 3 24
Thus required probability =
600 10 15 10 150

PDF Download FROM >> WWW.SARKARIPOST.IN Join >> https://www.facebook.com/Sarkaripost.in/


WWW.SARKARIPOST.IN
634 Quantitative Aptitude

26. (a) Total no. of arrangements of the letters of the word 1


and P (53 Fri and 53 Sat) =
10! 7
UNIVERSITY is . P (53 Fri or Sat) = P (53 Fri) + P (53 Sat)
2!
– P (53 Fri and Sat)
No. of arrangements when both I's are together = 9!
2 2 1 3
So. the no. of ways in which 2 I’s do not together = –
7 7 7 7
10! 32. (d) Clearly, P (A B C = 1)
= 9!
2! P(A) + P(B) + P(C) = 1
Required probability
1 1
P ( A) P ( A) P ( A) 1
10! 2 3
9!
2! 10! 9! 2!
= 11
10! 10! P ( A) 1
6
2!

WWW.SARKARIPOST.IN
6
27. (a) Total no. of players = 15 P (A) =
11
Total no. of batsmen = 8
Total no. of bowlers = 7 33. (d) Now, P(A B ) = P(A B )
Total no. of players in the team = 11 = 1 – P (A B) = 1 – 0.8 = 0.2
and P(A B ) = 1 – P (A B) = 1 – 0.3 = 0.7
No. of ways to choose a team = 15
C11 But P(A B ) = P (A ) + P (B ) – P(A B )
No. of way to choose 6 batsmen and 5 bowler 0.7 = P(A ) + P (B ) – 0.2
P (A ) + P (B ) = 0.9.
= 8 C6
7
C5
34. (c) Let E be the event of total of 12.
8 7
C6 C5 E = (2, 2, 2, 3, 3), (2, 2, 3, 3, 2), (2, 3, 3, 2, 2),
Required Probability = 15
C11 (3, 3, 2, 2, 2), (3, 2, 3, 2, 2), (3, 2, 2, 3, 2),
28. (d) Total number of numbers = 4! = 24 (3, 2, 2, 2, 3), (2, 3, 2, 3, 2), (2, 3, 2, 2, 3),
For odd nos. 1 or 3 has to be at unit's place (2, 2, 3, 2, 3)
If 1 is at unit place, then total number of numbers n (E) = 10
= 3! = 6 Sample sapce contain total possibility = 25 = 32
And if 3 is at units place, then total number of numbers Hence, n(s) = 32
= 3! = 6
Total number of odd number = 6 + 6 = 12 n( E ) 10 5
So, P ( E )
n(S ) 32 16
12 1
Required probability = 35. (c) Since, probabilities of failure for engines A, B and C
24 2
P(A), P(B) and P(C) are 0.03, 0.02 and 0.05
29. (c) Required Probability = P ( X ).P(Y ) P( X ).P (Y ) respectively.
The aircraft will crash only when all the three engine
60 50 40 50 1 fail. So, probability that it crashes = P(A). P(B). P(C)
=
100 100 100 100 2 = 0.03 × 0.02 × 0.05
30. (c) P (3 4) = P (c) + P (d) – P (3 4) = 0.00003
8 6 2 12 Hence, the probability that the aircraft will not crash,
= = 1 – 0.00003
25 25 25 25
= 0.99997
31. (b) In a leap year there are 366 days in which 52 weeks
and two days. The combination of 2 days may be : 36. (a) Total possible outcomes, S ={HHH, HHT, HTH, THT,
Sun-Mon, Mon-Tue, Tue-Wed, Wed-Thu, Thu-Fri, TTH, THH, TTT, HTT} and desired outcomes
Fri-Sat, Sat-Sun. E ={HTH, THT}
n(E) = 2 and n(S) = 8
2 2
P (53 Fri) = ; P (53 Sat) = n( E ) 2 1
7 7 Hence, required probability = P ( E ) = =
n(S ) 8 4

PDF Download FROM >> WWW.SARKARIPOST.IN Join >> https://www.facebook.com/Sarkaripost.in/


WWW.SARKARIPOST.IN
Probability 635

4 there will be no boy and only girls. So, total


37. (a) Probability of passing in mathematics
9 possibility of at least one boy is 16 – 1 = 15
Probability of passing in physics 2
5 15
P (at least one boy) = .
16
2 3
Probability of failure in physics = 1 = 44. (c) Let P (A) be the probability that the race will be won
5 5
Given that both the events are independent. by A and P(B) be the probability that the race will be
won by B.
4 3 4
Required probability
9 5 15 1 1
P ( A) and P(B)
13 1 5 6
38. (c) Probability of getting a diamond, P(D) = Probability that the race will be won by
52 4
1 1 11
4 1 A or B = P(A) + P (B) =
and probability to king, P(K) 5 6 30
52 13

WWW.SARKARIPOST.IN
So, required probability = P(D).P(K) 365 1 1
45. (b) Required probability = .
365 365 365
1 1 1
4 13 52 46. (b) A and B are mutually exclusive and exhaustive events
with
39. (c) A dice has six faces. So, in throwing of two dice, the P(A B) = 0, P(A B) = 1
number of exhaustive events is 6 × 6 = 36. we know that
40. (c) 16 tickets are sold and 4 prizes are awarded. A person P(A B) = P(A) + P(B) – P(A B)
4 1 1 = P(A) + 3P(A)
buys 4 tickets, then required probability =
16 4 1 3
P ( A) P( B)
13 4 4
41. (c) Probability of getting a spade =
52 3 1
Hence, P( B) 1 P( B) 1
4 4
4
Probability of an ace = 47. (c) As we know P (A B) 1
52
P(A) + P(B) – P(A B) 1
1 0.8 + 0.7 – P (A B) 1
and probability of getting a spade ace = P(A B) 1.5 – 1
52
P(A B) 0.5
13 4 1 16 4 Hence, the minimum value of P(A B) is 0.5.
Required probability =
52 52 52 52 13 48. (c) Possible samples are as follows
{HHH , HTH , HHT , THH , TTH , THT , HTT , TTT }
4 9
1 Let A be the event of getting one head.
13 13 9
Odds against his winning = Let B be the event of getting no head.
4 4 4
13 13 Favourable outcome for
A TTH , THT , HTT
1 1
42. (b) If both get one head then it is Favourable outcome for
4 4
1 1 B TTT
and if both get two heads then it is
2 2 Total no. of outcomes = 8
1 1 1 1 3 1
Prob (getting same number of heads) P A ,P B
4 4 2 2 8 8
Required probability = Probability of getting one
1 1 5
head + Probability of getting no head
16 4 16
43. (a) Total possibility of 4 children, either girl or boy is 3 1 4 1
= P(A) + P(B)
24 = 16. Out of these there is one possibility in which 8 8 8 2

PDF Download FROM >> WWW.SARKARIPOST.IN Join >> https://www.facebook.com/Sarkaripost.in/


WWW.SARKARIPOST.IN
636 Quantitative Aptitude

49. (a) No. of blue balls =2 P (B) = Probability of drawing two king cards
No. of red balls = 7 4
C2
Total no. of balls = 9 = 52
[ There are 4 king cards]
C2
Required probability
= P (one ball is blue) + P (both ball is blue) P(A B) = Probability of drawing 2 red king cards
2
2 7 2 1 14 2 16 2 C2
= 52
[ There are just 2 red kings]
9 8 9 8 72 72 72 9 C2
Substituting the values in (a), we get
x 26 4 2
50. (c) Given probability of guessing a correct answer C2 C2 C2 325 6 1
12 P( A B) 52 52 52
C2 C2 C2 1326 1326 1326
2 55
and probability of not guessing the correct answer .
3 221
As we know 56. (d) Probability that only husband is selected

WWW.SARKARIPOST.IN
P (occurence of an event) + P (non-occurence of an 1 1 1 4 4
event) = 1 P ( H ) P(W ) 1
7 5 7 5 35
x 2 x 8 Probability that only wife is selected
1 1 x 12 8 4
12 3 12 1 1 6 1 6
51. (a) Since, A and B are mutually exclusive events. = P ( H ) P (W ) 1
7 5 7 5 35
P (AB) = P (A B) = 0 Probability that only one of them is selected
52. (b) Prob. (certain event) = 1
P (E) = 1 4 6 10 2
53. (a) Total number of attempts = 20 35 35 35 7
Favourable no. of attempts = 5 57. (b) The probability that the person hits the target = 0.3
Required probability (running the program correctly The probability that he does not hit the target in a
5 1 trial = 1 – 0.3 = 0.7
in the third run) The probability that he does not hit the target in any
20 4
of the ten trials = (0.7)10
54. (b) Total possible nine digit number = 9!
Probability that he hits the target
Out of these 9! numbers only those numbers are = Probability that at least one of the trials succeceds
divisible by 4 which have their last digits as even = 1 – (0.7)10.
natural number and the numbers formed by their last 58. (c) If six coins are tossed, then the total no. of outcomes =
two digits are divisible by 4. The possible numbers of (2)6 = 64
last two digits are
1
12, 32, 52, 72, 92, 24, 64, 84, 16, 36, 56, 76, 96, 28, Now, probability of getting no tail
64
48, 68.
Thus there are 16 ways of choosing the last two digits. Probability of getting at least one tail
Corresponding to each of these ways the remaining 7 1 63
1–
digits can be arranged in 7! ways. Therefore, the total 64 64
number of 9 digits numbers divisible by 4 is 16 × 7!. 59. (b) Suppose E1, E2 and E3 are the events of winning the
race by the horses A, B and C respectively
16 7! 2
Hence, required probability = 1 1 1 1
9! 9 P ( E1 ) , P ( E2 )
55. 52
(d) 2 cards can be drawn from the pack in C2 ways. 1 3 4 1 4 5
Let A be the event “ Two cards are red” and B be the
1 1
event “Two cards drawn are kings”. P ( E3 )
1 5 6
The required probability is P(A B).
From addition theorem, we have Probability of winning the race by one of the horses
A, B and C
P ( A B ) P ( A) P ( B ) P ( A B ). ...(1)
Now, P (A) = Probability of drawing two red cards P ( E1 or E2 or E3 ) P ( E1 ) P( E2 ) P ( E3 )
26 1 1 1 37
C2
= [ There are total 26 red cards]
52
C2 4 5 6 60

PDF Download FROM >> WWW.SARKARIPOST.IN Join >> https://www.facebook.com/Sarkaripost.in/


WWW.SARKARIPOST.IN
Probability 637

1 1 1
60. (c) Probability of selecting a month = . 67. (d) P(1) = , P(2) = , A and B are independent events,
12 6 2
13th day of the month is Friday if its first day is Sunday 1 1
.
1
that means P(AB) = P(1) P(2) =
6 2 12
1
and the probability of this = . and P (Ac Bc) = 1 – P (A B)
7
P(Ac Bc) = 1 – P(a) – P(b) + P(AB)
1 1 1 1 1 1 12 2 6 1 5
Required probability = . . =1– =
12 7 84 6 2 12 12 12
61. (d) Total of seven can be obtained in the following ways 68. (b) Since A and B are independent
4! P( A B) P( A).P ( B )
1, 1, 1, 4 in = 4 ways
3!
and P ( A / B ) P ( A)
[there are four objects, three repeated]
Similarly, 1
Thus, P ( A / B )

WWW.SARKARIPOST.IN
4! 2
1, 1,2, 3 in = 12 ways Hence, option (b) is not true.
2! 69. (c) Probability of the card being a spade or an ace
4! 16 4
1, 2,2, 2 in = 4 ways . Hence odds in favour is 4 : 9.
3! 52 13
4 12 4 20 So, the odds against his winning is 9 : 4.
Hence, required probability
64 64 70. (c) The probability that a man will not live 10 more years
[ Exhaustive no. of cases = 6 × 6 × 6 × 6 = 64] = 3/4 and the probability that his wife will not live 10
62. (c) The number of ways of getting the different number more years = 2/3. Then the probability that neither will
1, 2, ....., 6 in six dice = 6 !. be alive in 10 years = 3/4 × 2/3 = 1/2
Total number of ways = 66 71. (b) Total number of possibilities = 25 × 25
6! Favourable cases for their winning = 25
Hence, required probability6 =
6
25 1
1 2 3 4 5 6 5 P (they win a prize)
25 25 25
66 324

63. (d) 1
P (they will not win a prize) 1
25
64. (a) Here the number of favourable cases, consists of
throwing 10,11 or 12 with the two dice. The number 72. (c)
of ways in which a sum of 10 can be thrown are (4,6), 73. (d) Here P (A) = 0.4 and P ( A) 0.6
(5,5), (6,4) i.e. 3 ways. The number of ways in which Probability that A does not happen at all = (0.6)³
a total of 11 can be thrown are (5, 6), (6,5) i.e. 2 ways. Thus required probability = 1 – (0.6)³ = 0.784
The number of ways in which a total of 12 can be 74. (d) As there are four jacks and four aces, the number of
thrown in (6, 6) i.e. 1 way. favourable cases = 8
m = number of favourable cases = 3 + 2 + 1 = 6 8 2
n = Total number of cases = 6 × 6 = 36 The required probability p
52 13
m 6 1 75. (a) The favourable cases are (1, 3), (2, 4), (3,5), (4,6) and
Probability = p (1, 4), (2, 5), (3, 6) and their reversed cases like
n 36 6
(3, 1)......
65. (c) Required probability
Total number of favourable cases = 2 7
13 4 1 14 7
= P (Diamond) . P (King) = . p= =
52 52 52 36 18
66. (a) The cards are of four colours and the number of cards 76. (a) The first card can be one of the 4 colours, the second
of given description is 24. can be one of the three and the third can be one of the
two. The required probability is therefore
24 23 46
The probability = . = . 13 13 13 169
52 51 221
4× ×3× ×2× = .
52 51 50 425

PDF Download FROM >> WWW.SARKARIPOST.IN Join >> https://www.facebook.com/Sarkaripost.in/


WWW.SARKARIPOST.IN
638 Quantitative Aptitude

77. (a) Quadratic equations which area unchanged by squaring Standard Level
their roots = 4
whose roots are (0, 0), (0, 1), (1, 1) and ( , 2) 1. (d) If the number on the dice is 1, then the probability that
If which two equation have equals roots 1 red ball is
6
2 1 C1
Probability = chosen = 10
4 2 C1
78. (c) Total number of ways in which 4 persons can be 6
If number on the dice is 2, then the C2
selected out of 3 + 2 + 4 = 9 persons = 9C4 = 126 10
that two red balls can be choosen C2
Number of ways in which a selection of 4 contains
exactly 2 children = 4C2 × 5C2 = 60 and so on, till the number on the dice is six Now,
60 10 ea ch number on the dice can appear with a
reqd. prob. = probability of 1/6.
126 21
Required probability

WWW.SARKARIPOST.IN
1
79. (b) P(A) = 1/4, P(A/B) = , P(B/A) = 2/3 1 6
C1 6
C2 6
C3 6
C4 6
C5 6
C6
2
6 10 10 10 10 10 10
By conditional probability, C1 C2 C3 C4 C5 C6
P(A B) = P(A) P(B/A) = P(B)P(A/B)
1 6 15 20 15 6 1 1
.
1 2 1 1 6 10 45 120 210 252 210 5
P( B) P( B )
4 3 2 3 2. (b) We know,
1 P( A B) P ( A) P ( B) P ( A B)
80. (c) Probability of getting a head on tossing a coin (P1) = .
2 0.8 + 0.9 – p < 1
1 1.7 – p < 1
Probability of getting a six on rolling a dice (P2) = . 0.7 < p
6
Now, P(A) < P (B)
These two events are independent.
P( A B) P ( A)
So the probability that the coin shows the head and the
p < 0.8
dice shows 6 is given by
Hence, 0.7 < p < 0.8
1 1 1 3. (a) P(A) + P(B) + P(C) = 1 2 P(B)/3 + P(B) + P(B)/2 =
P P1 P2 . 1
2 6 12
13P(B)/6 = 1 P(B) = 6/13. Hence, P(A) = 4/13
Solutions for 81 and 82
4. (c) 52
Total number of balls in urn – I = 1 Black + 1 Green = 2 Balls n( s ) C2 = 1326
Total number of balls in urn – II =1 White + 1 Green = 2 Balls Let A = event of getting both red cards
and B = event of getting both queens
1 1 1 then A B = event of getting two red queens
81. (c) Required prob =
2 2 4
26 4
n( A) C2 = 325, n( B ) C2 = 6
82. (d) Required prob
2
1 1 1 1 1 1 1 1 1 3 n( A B) C2 = 1
= =
2 2 2 2 2 2 4 4 4 4 325 6 1
P(A) = , P(B) =
1326 1326 221
1
P(A B)=
1326
83. (a) P(both red or both queens) = P(A B)
= P(A) + P(B) – P(A B)
19/14 1/8 5/24 325 1 1 55
=
We have: 19/40 + 1/8 + 5/24 = 97/120 1326 221 1326 221

PDF Download FROM >> WWW.SARKARIPOST.IN Join >> https://www.facebook.com/Sarkaripost.in/


WWW.SARKARIPOST.IN
Probability 639

5. (c) 52
n( S ) C4 1 5
2
5
4
= 1 ...
Let E1, E2, E3, E4 be the events of getting all spades, 6 6 6
all clubs, all hearts and all diamonds respectively.
Then 13 1 1 1 36 6
n( E1 ) C4 =
6 2 6 11 11
5
n( E2 ) 13
C4 1
6
13
n( E3 ) C4 6 5
Thus, P (A wins) = and P (B wins) =
13 11 11
n( E4 ) C4
7. (a) The probability that A cannot solve the problem
13 13
C4 C4 2 1
P ( E1 ) , P ( E2 ) , 1
52 52
C4 C4 3 3

WWW.SARKARIPOST.IN
The probability that B cannot solve the problem
13 13
C4 C4
P ( E3 ) , P ( E4 ) , 3 1
52 52 1
C4 C4
4 4
Since E1, E2, E3 and E4 are mutually exclusive events. The probability that both A and B cannot solve the
P (getting all the 4 cards of the same suit)
1 1 1
P(E1 or E2 or E2 or E4) = P(E1) + P(E2) + P(E3) + problem
3 4 12
P(E4)
The probability that at least one of A and B can solve
13
C4 44 1 11
= 4 52 4165
the problem 1
C4 12 12
11
6. (a) Let E = the event that A gets a three The probability that the problem is solved =
12
and F the event that B gets a three 8. (b) A and B will contradict each other if one of the events
1 1 A B ' or A ' B occurs. The probability of this
Then, P(E) = , P (F ) happening is
6 6
P[( A B ') ( A ' B) P( A B ') P( A ' B )
5 5
p(E) , P (F ) P ( A) P ( B ') P( A) P( B),
6 6
Suppose A wins then, he gets a three in 1st or 3rd or because A and B are independent. Therefore, putting
5th... throw etc. P(A) = 0.7 and P(B) = 0.8 the required probability is
(0.7) (0.2) + (0.3) (0.8) = 0.38.
P(A wins) = P[E or ( E F E ) or ( E F E F E ) or... ]
9. (a) Since, A and B are independent events.
= P[E or ( E and F and E) or ( E and F and E and P( A B) P ( A) P ( B )
F and E) or ...] Further since, A C, B C, A B C are subsets
= P(E) + P( E and F and E) + P( E and F and E of C, we have
and P( A C) P(C ) 0
F and E) + ... P( B C) P(C ) 0
= P(E) + P( E ) P( F ) P( E ) P( E ) P( F ) .
and P( A B C) P(C ) 0
P( E ) . P( F ) . P( E ) + ... P( A C) 0 P ( A) P (C )
1 5 5 1 5 5 5 5 1 P( B C) 0 P ( B ) P (C )
= ...
6 6 6 6 6 6 6 6 6
P( A B C) 0 P ( A) P ( B ) P (C ) .
2 4 Clearly A, B, C are pairwise independent as well as
1 1 5 1 5
= ... mutually independent. Thus, A,B,C are independent
6 6 6 6 6
events.

PDF Download FROM >> WWW.SARKARIPOST.IN Join >> https://www.facebook.com/Sarkaripost.in/


WWW.SARKARIPOST.IN
640 Quantitative Aptitude

10. (b) Let A be the event of getting an odd number. 3 1


Probability (the packet is not damaged) 1–
Here, n (S) = 6 and 4 4
n (A) = 3 1 2
Probability (there is no short shipment) 1–
3 1 3 3
Probability of getting an odd number
6 2
Hence, probability of not getting an odd number Required probability 4 1 2 2 8
1 1 5 4 3 15 60
1–
2 2 15. (c) Probability of occurence of head in a toss of a coin is
Required probability of 5 successes 1/2.
Required probability = Prob [Head appears once]
5
6 1 1 3 + Prob. [Head appears thrice] + Prob. [Head appears
C5
2 2 32 five times]
11. (d) Total number of balls = 8. Let the first drawn ball is 5 5 5
1 1 1
5 3 4 2 1 = 5C1 5
C3 5
C5
2 2 2

WWW.SARKARIPOST.IN
white, so required probability = .
8 7 6 5 14
But here we had started with a white ball. When we 5
1 16 1
start with a black ball, the required probability = [5 + 10 + 1] =
2 32 2
3 5 2 4 1
= . 16. (b) Total no. of outcomes when two dice are thrown = n
8 7 6 5 14
Since these two cases are mutually exclusive. (S) = 36 and the possible cases for the event that the
1 1 2 1 sum of numbers on two dice is a prime number, are
Total probability = . (1, 1), (1, 2), (1, 4), (1, 6), (2, 1), (2, 3), (2, 5), (3, 2), (3, 4),
14 14 14 7
12. (b) There are 7 + 5 = 12 balls in the bag and the number of (4, 1), (4, 3), (5, 1), (5, 6), (6, 1), (6, 5)
ways in which 4 balls can be drawn is 12C4 and the Number of outcomes favouring the event = n (A) = 15
number of ways of drawing 4 black balls (out of seven)
is 7C4. n A 15 5
Required probability
Hence, P (4 black balls) n S 36 12
7
C4 7.6.5.4 1.2.3.4 7 17. (d) Out of 36 possible outcomes the ones which are
12 1.2.3.4 12.11.10.9 99
C4 favourable for the event are
Thus the odds against the event ‘all black balls’ are (i) When the numbers are both even and
(ii) When the numbers are both odd. There are six
7 7 92 7 doublets and the pairs. (1, 3), (1, 5), (2, 4), (2, 6) etc.
(1 ) : : i.e., : or 92 : 7 .
99 99 99 99 Which make a total of 6 × 3 = 18. The required
13. (b) The word ‘SOCIETY’ contains seven distinct letters probability is 1/2.
and they can be arranged at random in a row in 7P7 18. (c) There are 5 pairs of shoes and 4 shoes can be picked in
ways, i.e., in 7! = 5040 ways. 10 × 9 × 8 × 7 ways. Number of ways in which 4 shoes
Let us now consider those arrangements in which all can be picked such that no two are alike = 10 × 8 × 6 ×
the three vowels come together. So in this case we have 4.
to arrange four letters. S,C,T,Y and a pack of three
vowels in a row which can be done in 5P5 i.e. 5! = 120 10 8 6 4 13
ways. The required probability = 1 – = .
10 9 8 7 21
Also, the three vowels in their pack can be arranged in
3P i.e. 3! = 6 ways. 19. (c) The total number of ways = 6 6 6 = 216 . Now we
3
Hence, the number of arrangements in which the three count the number of favourable ways. Clearly the
vowels come together is 120 × 6 = 720 second number has to be greater than 1.
The probability that the vowels come together If the second number is i (i > 1)
Thus the number of favourable ways
720 1 5
=
5040 7 (i 1)(6 i ) = 1 × 4 + 2 × 3 + 3 × 2 + 4 × 1 = 20
14. (b) Probability (sending a correct programme) i 1

1 4 20 5
1– Required probability = =
5 5 216 54

PDF Download FROM >> WWW.SARKARIPOST.IN Join >> https://www.facebook.com/Sarkaripost.in/


WWW.SARKARIPOST.IN
Probability 641

20. (b) Out of the 63 possible outcomes 6.5.5 outcomes will 27. (b) The event definition is
have all distinct numbers. A girl is selected from the first group and one boy each
4 are selected from the second and third groups. OR A
6.5.4
The probability = 1 – 3
= . girl is selected from the second group and one boy
6 9
each are selected from the first and third groups. OR
21. (b) Favourable cases for one are there i.e., 2, 4 and 6 and A girl is selected from the third group and one boy
for other are two i.e., 3, 6. each are selected from the first and second groups.
3 2 1 11 28. (c) Total 80, Girls = 25, Boys = 55
Hence required probability = 2 10 R, 70 P, 20 I
36 36 36
1 1 25 5
[As same way happen when dice changes numbers
4 8 80 512
among themselves] 29. (b) Given P( A f ) 0.2 and P( B f ) 0.3
22. (d) If a die is thrown, there are 6 equally likely and mutually Since, A and B are independent events
exclusive cases. Since two dice are thrown, the total
number of ways = 6 × 6 = 36. If a sum of 7 is to be P( A B) P ( A).P ( B )

WWW.SARKARIPOST.IN
obtained from the numbers appearing on the two upper (0.2) (0.3) 0.06
faces, the numbers in the two dice can be (1, 6), (2, 5),
(3, 4), (4, 3), (5, 2), (6,1), which are six in number. Required prob P( A B)
Number of favourable cases = m = 6 P ( A) P( B) P( A B )
Total number of cases = 36
= 0.2 + 0.3 – 0.06 = 0.44
m 6 1 30. (d) Nos. divisible by 6 are 6, 12, 18, ......, 90.
The required probability = p
n 36 6 Nos. divisible by 8 are 8, 16, 24, ......, 88.
23. (c) If the last digit in the product is not 1, 3, 5, 7 or 9, it
Now, total no. divisible by 6 = 15
must be 0, 5, or even.
Hence none of the four numbers must end in 0, 2, 4, 5, and total no. divisible by 8 = 11
6, 8. Now, the no. divisible by both 6 and 8 are 24, 48, 72.
Hence out of ten numbers in which any of the four So, total no. divisible by both 6 and 8 = 3
whole numbers may end, four are favourable to the Probability (number divisible by 6 or 8)
event that none of the four numbers must end in 0, 2,
4, 5, 6, 8. 15 11 3 23
=
Hence the probability that each of the four numbers 90 90
4 2 31. (a) Let E = {(1, 6), (2, 5), (3, 4), (4, 3), (5, 2), (6, 1)}
must not end in any of these = 6 1
10 5 P( E )
4
6 6 6
2 16 So, odds against drawing 7
Required probability = .
5 625 1
1–
24. (a) The ace of hearts can be drawn in only 1 way P( E ) 6 5
( in a pack of cards there is only one ace of heart) P( E ) 1 1
1 6
P (A) = Probability of drawing the ace of hearts = . 32. (a) 6C2 ×[(7/11) × (6/10) ×(5/9) × (4/8) ×(4/7) ×(3/6)] =
52
5/11.
Hence the probability of not drawing an ace of hearts 33. (c) n(S) = 100
1 51 Let A be the event of getting a number divisible by 2
= P ( A) 1 P ( A) 1 and B be the event of getting a number divisible by 3
52 52
25. (a) Let P (A) and P (B) be the probability of the events of and C be the event of getting of number dvisible by 5.
getting 4, 5 or 6 in the first throw and 1, 2, 3 or 4 in the (A B) be the event of getting a number divisible by
second throw respectively, then both 2 and 3.
(B C) be the event of getting a number divisible by
1 2 1 both 3 and 5.
P (A and B) = P(A).P(B) =
2 3 3 (A C) be the event of getting a number dvisible by
both 2 and 5.
1 2 5 3 1
26. (c) Required probability = 1 1 (A B C) be the event of getting a number divisible
6 5 6 5 2 by A, B and C.

PDF Download FROM >> WWW.SARKARIPOST.IN Join >> https://www.facebook.com/Sarkaripost.in/


WWW.SARKARIPOST.IN
642 Quantitative Aptitude

Now, n(A) = 50, n(B) = 33, n(C) = 20, n(A B) = 16, 36. (c) Required probability
n(B C) = 6, n(A C) = 10, n(A B C) = 3 = P (X not defective and Y not defective)
50 1 33
P(A) = , P(B) = , = P( X ) P(Y )
100 2 100
= {1 – P(X)}{1 – P(Y)}
20 1 16
P(C) = , n(A B) = 91 95 8645
100 5 100 = = = 0.8645
100 100 10000
6 10
n(B C) = , n(A C) = ,
100 100
3 37. (b) A(3/7) B(1/2)
n(A B C) =
100
Required probability = P(A B C)
= P(A) + P (B) + P(C) – [P(A B)
+ P(B C ) + P(A C)] + P(A B C) X

WWW.SARKARIPOST.IN
50 33 20 16 6 10 3
= Also P A B 1 P A B
100 100 100 100 100 100 100
(3/7) + (1/2) – x = 13/14 x = 0
74 Thus, there is no interference between A and B as
=
100
P A B = x = 0. Hence, A and B are mutually
34. (b) Out of 30 numbers 2 numbers can be chosen in 30C2
ways. So, exhaustive number of cases = 30C2 = 435 exclusive.
Since a2 – b2 is divisible by 3 if either a and b are 38. (b) If I choose a shot that I will make probability p (where
divisible by numbers, of cases = 10C2 + 20C2 = 235 p is between 0 to 1/3), then Michael Jordan will make
the same shot with probability 3p. Hence, the
235 47 probability that I make a shot that Jordan subsequently
Hence, required probability =
435 87 misses is p (1 – 3p). The graph of this function is a
35. (c) Let A, B, C be the respective events of solving the 1
parabola which equals zero when p = 0 and p .
problem and A, B, C be the respective events of not 3
By symmetry the vertex (maximum) is midway
solving the problem. Then A, B, C are independent
1
events between the two, at p .
6
A, B, C are independent events Hence, the best chance I have of winning the game is
1
1 1 1 .
P(A) = , P(B) = and P(C) = 12
2 3 4 39. (b) Total number of triangles formed = 8C3 = 56
1 2 3 Triangles having two sides common = 8
P ( A) , P( B) and P (C ) Triangles having one side common = 8C1 × 4C1 = 32
2 3 4
Triangle having three sides common = 0
P(none solves the problem)
Triangles having no side common = 56 – 40 = 16
= P(not A) and (not B) and (not C)]
So, probability = 16/56 = 2/7
= P( A B C) 40. (d) Probability that first ball is white and second black
= P( A) P( B ) P (C ) ( A, B and C are independent = (4/6) × (5/8) = 5/12
Probability that first ball is black and second white
1 2 3 1
= = (2/6) × (3/8) = 1/8
2 3 4 4
These are mutually exclusive events hence the required
Hence, P(the problem will be solved)
probability
= 1 – P(none solves the problem)
1 3 5 1 13
P .
=1 12 8 24
4 4

PDF Download FROM >> WWW.SARKARIPOST.IN Join >> https://www.facebook.com/Sarkaripost.in/


WWW.SARKARIPOST.IN
Probability 643

41. (a) The total possible pairs of children (B, B), (B, G), (G, 1
B). Now the one child is boy, is confirmed, but we Now, probability of getting no tail
64
don’t know whether he is youngest or elder one. So
the three ordered pairs could be the one describing the Probability of getting at least one tail
children in this family. So the probability of the younger 1 63
1–
children to be boy = 2/3. 64 64
42. (b) The odd against P solving a problem = 8 : 6.
46. (a) P (A B) = P (A) + P (B) – P (A B);
8 4
Probability of P not solving the problem 3 1
14 7 =1 – P( A ) + P(B) –
4 4
The odds in favour of Q solving problem = 14 : 10
10 5 2 2
Probability of Q not solving the problem 1=1– + P(B) P(B) = ;
24 12 3 3
Hence, the probability of P and Q not solving the
2 1 5

WWW.SARKARIPOST.IN
problem Now, P( A B ) = P(B) – P ( A B)= – = .
3 4 12
4 5 5
47. (c) A number is divisible by 9, if the sum of its digits is
7 12 21
divisible by 9. Here 1 + 2 + 3 + 4 + 5 + 6 + 7 + 8 + 9
Probability of the problem being solved = 45 is divisible by 9.
= 1 – probability of the problem not being solved the two numbers to be removed should be such that
their sum is 9.
5 16
1 . they can be any one of the following pairs
21 21 (1, 8), (2, 7), (3, 6), (4, 5).
43. (a) The probability that A cannot solve the problem Hence the number of favourable cases = 4
2 1 Total number of cases of removing two numbers = 9C2
1
3 3 4 4 1
The probability that B cannot solve the problem Required probability = 9 = = .
C2 36 9
3 1
1 48. (a) 5 Students can be selected from 10 in 10C5 ways.
4 4
The probability that both A and B cannot solve the 10! 10 9 8 7 6
n (S) = 10C5 = = = 252
5!.5! 5 4 3 2
1 1 1
problem Let A be the event that the committee includes exactly
3 4 12
2 girls and 3 boys. The two girls. can be selected in
The probability that at least one of A and B can solve 4C ways and the 3 boys can be selected in 6C ways.
2 3
1 11 n (A) = 4C2 6C3 = 6 20 = 120
the problem 1
12 12 n( A) 120 10
P (A) = = =
11 n(S ) 252 21
The probability that the problem is solved =
12 49. (c) If 9 letters are in right envelope, then the 10th
44. (b) Chandra hits the target 4 times in 4 shots. Hence, he automatically has to go to the right envelope.
hits the target definitely. Hence the probability of such occurrence is 0.
The required probability, therefore, is given by.
50. (d) The probability that none of the digits chosen turn out
P (both Atul and Bhola hit) + P (Atul hits, Bhola does
3
not hit) + P (Atul does not hit, Bhola hits) 6 8
to be a perfect square = = , as 1, 4, 9 are the
9 27
3 2 3 4 3 2
= ones which are perfect squares, the required probability
6 6 6 6 6 6
19
1 1 1 4 2 = .
27
6 3 6 6 3 51. (d) n(S) = 7!, n(E) = (3!) × (4!)
45. (c) If six coins are tossed, then the total no. of outcomes (3!) (4!) 6 1 1
= (2)6 = 64 P(E) = = = =
7! 7 6 5 15 35

PDF Download FROM >> WWW.SARKARIPOST.IN Join >> https://www.facebook.com/Sarkaripost.in/


WWW.SARKARIPOST.IN
644 Quantitative Aptitude

52. (c) Let A be the event of getting first card an ace and B be
B P ( A B)
the event of getting second a coloured one. Since, both P =
the events associated with a random experiment. A B P ( A) P( B ) P ( A B)
(i.e. condition of probability) 0.2 0.2 1
Therefore, the probability of getting first card an ace = 0.25
0.7 0.6 0.5 0.8 4
4 1
P ( A) A P( A B ) P( A B ) 1 P( A B)
52 13 56. (c) P
B P( B ) P( B ) P( B )
and probability of drawing a coloured one in second
draw 57. (d) Given P(A) + P(B) – P(A) P(B) = P(A B)
15 5 Comparing with
P ( B / A)
51 17 P(A) + P(B) – P(A B) = P(A B)
we get P(A B) = P(A).P(B)
(since one card has already been drawn)
A and B independent events.
Hence, by conditional probability,

WWW.SARKARIPOST.IN
58. (d) 972/1972 = 243/493.
P ( A B) 59. (b) The required probability will be given by the
P ( B / A)
P( A) expression:

5 P( A B ) The number of young boys who will die


17 1 The total number of people who will die
13
24 2 6
60. (d) (5! 4! 2! 3!) = = 1/1155.
5 1 5 11! 11 10 9 8 7 6
P( A B)
17 13 221
61. (a)
53. (a) Seven people can seat themselves at a round table in
6! ways. The number of ways in which two Total no. of ways in which two people sit together
distinguished persons will be next to each other = 2 P=
Total no. of ways
(5) !, Hence, the required probability
= (10! × 2!)/11!
2(5) ! 1 62. (a) Try to find the number of ways in which 0 or 1 bomb
6! 3 hits the bridge if n bombs are thrown.
The required value of the number of bombs will be
2
54. (c) Probability of A solving the problem = . such that the probability of 0 or 1 bomb hitting the
5
bridge should be less than 0.1.
2 63. (a) The number of events for the condition that he will
Probability of B solving the problem = sing = 4 [34, 43, 26, 62]
3
As A’s solving the problem and B’s solving the problem The number of events in the sample = 90.
are two independent events, required probability Probability that he will sing at least once
= 1 – Probability that he will not sing.
2 2 2 2 4
= + – × = . 64. (b) At least one means (exactly one + exactly two + exactly
5 3 5 3 5
three)
B P [ B ( A B )] At least two means (exactly two + exactly three)
55. (d) P
A B P ( A B) The problem gives the probabilities for passing in at
least one, at least two and exactly two.
P [ B A) ( B B )] 65. (b) The common side could be horizontal or vertical.
=
P ( A) P( B ) P ( A B ) Accordingly, the number of ways the event can occur
is.
Given
n(E) = 8 × 7 + 8 × 7 = 112
P( A B) 0.5 n(S) = 64C2
P( A) P ( A B) 0.5 2 8 7 2 1
Required probability = =
P( A B) P ( A) P ( A B) 0.7 0.5 0.2 64 63 18

PDF Download FROM >> WWW.SARKARIPOST.IN Join >> https://www.facebook.com/Sarkaripost.in/


WWW.SARKARIPOST.IN
Probability 645

Expert Level All the above cases being mutually exclusive, we have
the required probability
1. (c) Total no. of numbers = 6 positive + 8 negative = 14
2 1 2 1 5
n(S) = 14C4 =
5 5 15 10 6
The product of four numbers could be positive when,
(a) all the four numbers chosen are positive or 5 Red + 4 3 Red + 7
5. (b) Green Black
(b) all the four numbers chosen are negative or
(c) two of the chosen numbers are positive and two The no. of ways in which two different coloured
are negative. balls can be drawn are G + R, R + B, G + B.
6
C4 8
C4 6
C2 8
C2 P (two different balls) = P(1st red and 2 nd black)
Required Prob. = 14 14 14 + P (1 st green and 2 nd red) + P (1 st green and 2 nd
C4 C4 C4
Black)
505
=
1001 5 7 4 3 4 7

WWW.SARKARIPOST.IN
2. (b) With three dice the cases favourable for the event of 9 10 9 10 9 10
getting 10 are
35 12 28 75 5
1 3 6 2 2 6 .
3! 90 90 6
1 4 5 2 4 4
× 3! × Total of 27.
2 3 5 3 3 4 2! 6. (a) Two cases are possible here
(1) The person transferred to Grid-II is a director
27
1 (2) The person transferred to Grid-II is a General
Required probability = 3 =
6 8 Manager.
3. (b) Total number students = Number passing in I paper P (director super annuated) = P (director
only + Number passing II paper only + Number passing transferred) × P (director super annuated) + P
in both + Number failed in both (General Manager transferred) × P (director super
500 = (150 – 50) + (350 – 50) + X annuated)
50 1
X = 50 Required probability = 5 3 1 4 3
500 10
4 56 3 7 1 5 4 3 7 1
4. (a) Let A, B, C be the events that A hits the target, B hits
the target and C hits the target respectively. 5 4 4 3 32
4 3 2 .
P( A) , P( B) , P(C ) 9 11 9 11 99
Then,
5 4 3
7. (b) P( at least once) = 1 – P (none double six), for
1 1 1 one trial
P ( A) , P ( B) , P(C )
5 4 3 1 1 1
Case I. P(A, B, and C, all hit the target) = P(A B P (double six) = .
6 6 36
4 3 2 2
C) = P(A) P(B) P(C) = 1 35
5 4 3 5 P (no double six) = 1
36 36
Case II. P(A and B hit but not C) = P[A B C] n
35
4 3 1 1 P (atleast once) = 1 , for n trials.
= P(A) P(B) P( C ) = 36
5 4 3 5
8. (a) One red card and one queen can be drawn in the
Case III. P(A and C hit but not B) = P[A C B] following mutually exclusive ways;
4 2 1 2 (1) by drawing one red card out of 24 red cards
= P(A) P(C) P( B ) = (excluding 2 red queens) and one red queen out
5 3 4 15
of 2 red queens. Let the event be A.
Case IV. P(B and C hit but not A) = P(B C A) (2) by drawing one red card out of 26 red cards
3 2 1 1 (including 2 red queens and one queen out of 2
= P(B) P(C) P( A ) = black queens. Let this event be B.
4 3 5 10

PDF Download FROM >> WWW.SARKARIPOST.IN Join >> https://www.facebook.com/Sarkaripost.in/


WWW.SARKARIPOST.IN
646 Quantitative Aptitude

(3) by drawing two red queens of red. Let this event P ( A ' B ') 1 P ( A B)
be C
Required probability = P (A B C) = P (A) Now, P ( A ' B ') P ( A ') P( B ') P( A ' B ')
+ P (B) + P(C) P( A ') P( B ') [1 P ( A B)]
24 2 26 2 2
C1 C1 C1 C1 C2 101 P( A ') P( B ') P( A B ) 1.
= 52 52 52
C2 C2 C2 1326 Finally, since
9. (b) Since, they obtain equal number of heads, so number P ( A ' B ') P[( A ') ' B '] P[ A ' ( B ') '] P( A ' B ')
of heads obtained by them must be any of 0, 1, 2 and = P(A B ') P( A ' B ) P ( A ' B ')
3.
No. of heads Out comes Probability P( A B ) P ( A ' B) P ( A B ') P( A B)
0 TTT 1/8 [See the Venn diagram].
1 TTH, HTT, THT 3/8 1
2. THH, HTH, HHT 3/8 12. (a) Let P (A) = a and P(B) = b Then P ( A B)
6

WWW.SARKARIPOST.IN
3. HHH 1/8
1
So, the required probability P ( A) P ( B ) , because A and B are independent.
6
2 2 2 2
1 3 3 1 1
ab .....(1)
8 8 8 8 6
Also P ( A B ) [1 P ( A)][1 P ( B )]; .
1 9 9 1 20 5
.
64 64 64 64 64 16 1 1
[1 a ][1 b] 1 a b ab .....(2)
10. (d) Since, one and only one of the three events E1, E2 and 3 3
E3 can happen, therefore
5
P (E1) + P (E2) + P (E3) = 1 .....(1) From (1) and (2) we have a b .....(3)
Odds against E1 are 7 : 4 6
Solving (1) and (3) we get,
4 4
P ( E1 ) .....(2)
4 7 11 1 1 1
a , b , P( A) .
Odds against E2 are 5 : 3 2 3 2
13. (c) Let W stand for the winning of a game and L for losing
3 3
P( E2 ) .....(3) it. Then there are 4 mutually exclusive possibilities
3 5 8 (i) W, W, W (ii) W, W, L, W
4 3 (iii) W, L, W, W (iv) L, W, W, W.
From (1), (2) and (3), we have, P( E3 ) 1. [Note that case (i) includes both the cases whether he
11 8
losses or wins the fourth game.]
4 3 88 32 33 23 23 By the given conditions of the question, the
i.e. P( E3 ) 1
11 8 88 88 23 65 probabilities for (i), (ii), (iii) and (iv) respectively are
Hence odds against E3 is 65 : 23. 2 2 2 2 2 1 1 2 1 1 2 1 1 2 2
. . ; . . . ; . . . and . . . .
3 3 3 3 3 3 3 3 3 3 3 3 3 3 3
11. (d) A B
U Hence the required probability
8 4 4 4 36 4
= .
27 81 81 81 81 9
[ The probability of winning the game if previous
2 2
game was also won is and the probability
A B' 1 2 3
A B A' B
of winning the game if previous game was a loss is
At most one of two events occurs if the event A ' B' 1 1
]
occurs. 1 2 3

PDF Download FROM >> WWW.SARKARIPOST.IN Join >> https://www.facebook.com/Sarkaripost.in/


WWW.SARKARIPOST.IN
Probability 647

14. (b) The whole event consists of the following mutually The only equilateral triangles possible are A1A3A5 and
exclusive ways. A2A4A6
(1) Selecting the bag A, drawing a red ball from A and
2 2 1
putting it into bag B and then drawing a red ball p 6
C3 20 10
from B.
(2) Selecting the bag A, drawing a white ball from A and 17. (a) S {1, 2,3, 4,5, 6} {1, 2, 3, 4, 5, 6}
putting it into bag B and then drawing a white ball
from B. n( S ) 36
(3) Selecting the bag B, drawing a red ball from B and & Let E1 the event that the sum of the numbers
putting it into A and then drawing a red ball from A. coming up is 9.
(4) Selecting the bag B, drawing a white ball from B and & E2 the event of occurrence of 5 on the first die.
putting it into A and then drawing a white ball from A. E1 {(3, 6), (6, 3), (4, 5), (5, 4)}
The tree diagram of the above processes are shown n( E1 ) 4 and
below, with respective probability of each step E2 {(5,1), (5, 2), (5, 3), (5, 4), (5,5), (5, 6)}

WWW.SARKARIPOST.IN
3/8 Red n( E 2 ) 6
3/5 Red
5/8 White E1 E2 {(5, 4)} n( E1 E2 ) 1
1 Bag A 1/4 Red
2/5 White n( E1 E2 ) 1
2 Now, P ( E1 E2 )
3/4 White n( S ) 36
n( E2 ) 6 1
2/3 Red and P ( E2 )
n(S ) 36 6
1 2/7 Red
1/3 White Required probability
2 Bag B
Red 1
5/7 White 1/2 E P ( E1 E2 ) 36 1
P 1
1/2 White E2 P ( E2 ) 1 6
The required probability is 6

1 3 3 1 2 3 1 2 2 1 5 1 1 4p 1 p 1 2p
+ 18. (a) , , are probabilities of the three
2 5 8 2 5 4 2 7 3 2 7 2 4 2 2
mutually exclusive events, then
9 3 2 5 901
. 1 4p 1 p 1 2p
80 20 21 28 1680 0 1, 0 1, 0 1
4 2 2
15. (c) Numbers divisible by 4 are 104, 108.., 196; 24 in
number. Numbers divisible by 7 are 105, 112, ....196; 1 4p 1 p 1 2p
and 0 1
14 in number. Numbers divisible by both, i.e.divisible 4 2 2
by 28 are 112, 140, 168, 196; 4 in number. Hence,
1 3 1 1 1 5
required probability p , 1 p 1, p , p
4 4 2 2 2 2
24 14 4 34
1 1
99 99 99 99 p
2 2
16. (c) Three vertices can be selected in 6 C3 ways. [The intersection of above four intervals]
1
A5 A4 p
2
19. (b) Exhaustive no. of cases = 63
10 can appear on three dice either as distinct number
A6 A3 as following (1, 3, 6) ; (1, 4, 5); (2, 3, 5) and each can
occur in 3! ways. Or 10 can appear on three dice as
repeated digits as following (2, 2, 6), (2, 4, 4), (3, 3, 4)
3!
A1 A2 and each can occur in ways.
2!

PDF Download FROM >> WWW.SARKARIPOST.IN Join >> https://www.facebook.com/Sarkaripost.in/


WWW.SARKARIPOST.IN
648 Quantitative Aptitude

are 9 in number. Number of favourable cases = m =


3!
No. of favourable cases 3 3! 3 27 Number of numbers which have the digit 5 = 10 + 9 =
2! 19
27 1
Hence, the required probability m 19
6 3 8 Probability = p
n 100
20. (b) Since each ball can be put into any one of the three 25. (a) Let A be the event that the A is selected and B be the
boxes. So, the total number of ways in which 12 balls event that B is selected
can be put into three boxes is 312.
Out of 12 balls, 3 balls can be chosen in 12C3 ways. 1 2
P (A) = and P(B) =
Now, remaining 9 balls can be put in the remaining 2 5 7
boxes in 29 ways. So, the total number or ways in which Let C be the event that both are selected.
3 balls are put in the first box and the remaining in C=A B: P(C) = P(A B)
other two boxes is 12C3 × 29. P(C) = P(A) . P(B) as A and B are independent events
12
C3 .29 1 2 2
Hence, required probability = × =
12

WWW.SARKARIPOST.IN
3 5 7 35
21. (a) In any number the last digit can be one of 0, 1, 2, ...... 26. (c) n (S) = Total number of numbers = 5 × 5C4 × 4! = 5
8, 9. Therefore, the last digit of each number can be (5!)
chosen in 10 ways. Thus, exhausitive number of ways Five digit numbers divisible by 6 are formed by using
= 10n. If the last digit be 1, 3, 7 or 9 none of the numbers the numbers 0, 1, 2, 4 and 5 or 1, 2, 3, 4, and 5.
can be even or end in 0 or 5. Thus, we have a choice of number of such numbers
4 digits viz. 1, 3, 7 or 9 with which each of n number = n (E) = 2 (4)! + 2 × 3 × 3! + 4 × 3! = 108
should end. So, favourable number of ways = 27. (d) Probability of exactly M occurs = P ( M N)
4n . 4n 2
n

n . and probability of exactly N occurs = P ( M N)


5
Hence, the required probability = 10 The probability that exactly one of them occurs is
22. (a) Let E1 be the event that exactly two players scored
P (M N ) P (M N)
1 1 3 9
more than 50 runs then P(E1) = P ( M ) – P( M N ) P( N ) – P( M N)
2 3 4 10

1 2 1 9 1 2 3 1 1 1 1 9 P ( M ) P ( N ) – 2 P( M N )
+ 28. (c) Here the probability that the sun is hidden = 2/3.
2 3 4 10 2 3 4 10 2 3 4 10
The probability that it is out = 1/3.
1 1 3 1 1 2 1 1 65 Now at least 4 days shining 4 day out and 1 day
2 3 4 10 2 3 4 10 240 hidden + 5 days out =
Let E2 be the event that A and B scored more than 50 4 5
2 1 1 5 2 1 11
5 .
1 1 3 9 27 3 3 3 243 243 243
runs, then P ( E1 E2 )
2 3 4 10 240
29. (a) There are 13 cards of spade in a pack of 52 cards.
Desired probability So the chance that any of them will cut a spade is
13/52 = 1/4
P( E1 E2 ) 27
= P ( E2 / E1 ) Probability that it is not a spade
P ( E1 ) 65 = 1 – 1/4 = 3/4.
23. (a) The total number of ways in which 3 balls can be drawn Consider the following mutually exclusive ways in
is 9C3 and the number of ways of drawing 3 black balls which A may win : A wins in the first cut, OR A, B, C
is have failed and then A wins, OR A, B, C, A, B, C have
5C ; therefore the chance of drawing 3 black balls failed and then A wins and so on upto infinity
3
5
C3 Respective chances of these events are
5 4 3 5
= = 1/4, [(3/4) × (3/4) × (3/4) × (1/4)],
C3 9 9 8 7 42 .
[(3/4) × (3/4) × (3/4) × (3/4) × (3/4) × (3/4) × (1/4)] ....
Thus the odds against the event are 37 to 5. to infinity
24. (a) Total number of cases = n = 100
A’s chance of succeeding
All the numbers from 50 to 59 have the digit 5. They
are 10 in number. Besides these numbers the numbers = (1/4) + (3/4)3 × (1/4) + (3/4)6 × 1/4 + ....
5, 15, 25, 35, 45, 65, 75, 85, 95 have the digit 5. These = 1/4 [1/(1 – 27/64)] = 16/37.

PDF Download FROM >> WWW.SARKARIPOST.IN Join >> https://www.facebook.com/Sarkaripost.in/


WWW.SARKARIPOST.IN
Probability 649

30. (c) Consider the probability that, for example, an ace and The identical and one different like 009, 117, 225 and
a king are together. There are 4 aces and 4 kings in a
3!
deck. 441–––– 4× = 12
2!
Hence an ace can be chosen in 4 ways, and when that
is done a king can be chosen in 4 ways. All different digits like
Thus an ace and then a king can be selected in 018, 027, 036, 045, 126, 135, 234 –––– 7 × 3!
4 × 4 = 16 ways. = 42
Similarly, a king and then an ace can be selected in 16 So favourable cases = 55
ways.
55
Then an ace and a king can be together in 2 × 16 = 32 Desired probability =
999
ways.
For every one way the combination (ace, king) occurs, 35. (b) Total number of selections = 11 × 11
the remaining 50 cards and the (ace, king) combination Now |x – y| > 5 clearly x 5

WWW.SARKARIPOST.IN
can be permuted in 51! ways. The number of favourable If x = 0, then y > 5 5 favourable cases
arrangements is thus 32(51!).
If x = 1, then y > 6 4 favourable cases
Since the total number of arrangements of all the cards
If x = 2, then y > 7 3 favourable cases
in the deck is 52!, the required probability is
If x = 3, then y > 8 2 favourable cases
32(51!) 32 8
. If x = 2, then y > 9 1 favourable cases
52! 52 13
Symmetrical cases will be obtained for x = 6, 7, 8,
31. (c) The total number of ways in which 3 integers can be 9,10
chosen from first 20 integers is 20C3. Favourable cases = 30
The product of three integers will be even if at least
one of the integers is even. Therfore, the required 30
Desired probability
probability = 1 – Prob. that none of the three integers 121
is even 36. (c) The condition for real is a2 – 4b 0
10
a = 1 : for no value of b the condition is satisfied
C3 2 17 2 : value of b can be 1
1 20
1 .
C3 19 19 3 : 1, 2
[Three odd integers can be chosen in 10
C3 ways as 4 : 1, 2, 3, 4
5 : 1, 2, 3, 4, 5, 6
there are 10 even and 10 odd integers. 6 : 1, 2, 3, 4, 5, 6
32. (b) The condition implies that the last digit in both the Thus (a, b) can be one of the 19 pairs and the probability
integers should be 0, 1, 5 or 6 and the probability
19
=
2 36
1 4 1
4 37. (a) If the product of the four numbers ends in one of the
10 100 25 digits 1, 3, 7, or 9, each number should have the last
[The squares of numbers ending in 0 or 1 or 5 or digit as one of these 4 digits.
6 also 0 or 1 or 5 or 6 respectively] the number of favourable cases = 44
33. (d) Exhaustive number of cases = 12 Total number of all possible cases = 104

Favourable cases = 12C


2 (26 – 2) 44 24 16
Hence the required probability = 4 = 4 = .
10 5 625
12
C2 (2 6 2) 341
Probabiltity 38. (b) For at least 4 successes, required probability
126 125
4 3 5 2
7C
1 1 7 1 1
34. (a) Total no. of cases = 999 = 4
C5
2 2 2 2
Favourable cases.
6 1
The page number can be with all identical digits, 7 1 1 7 17 1
C6 C7 = .
i.e., 333 –––– 1 2 2 2 2

PDF Download FROM >> WWW.SARKARIPOST.IN Join >> https://www.facebook.com/Sarkaripost.in/


WWW.SARKARIPOST.IN
650 Quantitative Aptitude

39. (c) The word ASSISTANT contains two A’s, one I, one N, Now, |x – y| > 5 x – y < – 5 or x – y > 5
three S’s and two T’s whereas the word STATISTICS There are 30 pairs of values of x and y satisfying these
contains one A, one C, two I’s, three S’s and three T’s. two inequalities, so favourable number of ways = 30
total number of ways of choosing one letter from 30
each word = 9C1 . 10C1 = 90. Hence, required probability =
121
Common letters are A, I, S, T. 45. (c) Let A be the event of getting exactly 3 defectives in
the number of favourable cases the examination of 8 wristwatches.
= 2C1 . 1C1 + 1C1 . 2C1 + 3C1 . 3C1 + 2C1 . 3C1 A and B be the event of getting nineth wristwatch
= 2 + 2 + 9 + 6 = 19 defective
19 Then
Hence the required probability = .
90
B
40. (b) Let A the event that drawing ball is white. Required probability = P ( A B) P ( A) P
A
B the event that drawing ball is red.
There are two mutually exclusive cases of the required 4 11
C3 C5

WWW.SARKARIPOST.IN
event : WR and RR Now, P ( A)
15
C8
R 2 2 6
Now P(WR) = P(W) P = .
W 5 4 20 B
And P = Probability that the nineth examined
A
R 3 1 2
P(RR) = P(R)P = . wristwatch is defective given that there were 3
R 5 4 20
reqd. Prob. = P(WR + RR) = P(WR) + P(RR) 1
defectives in the first 8 pieces examined =
7
6 2 8 2
= 4 11
20 20 20 5 C3 C5 1 8
Hence, required probability = 15 7 195
41. (a) The event definition will be: Event X happens and Y C8
doesn’t happen or Y happens and X does not happen.
42. (a) The probability if he drops 3 bombs will be given by: 46. (b) ASSISTANT AA I N SSS TT
Hit and Hit OR Miss and Hit and Hit OR Hit and Miss STATISTICS A II C SSS TTT
and Miss = 0.9 × 0.9 + 0.1 × 0.9 × 0.9 × 0.9 × 0.1 × 0.9 Hence N and C are not common and same letters can
= 0.81 + 0.081 + 0.081 = 0.972 > 0.97. be A, I, S, T, Therefore
Hence, 3 bombs would give him a probability of higher
2 1
than 97% for the bridge to be destroyed. C1 C1 1
Probability of choosing A = 9 10 45
43. (b) 20 girls can be seated around a round table in 19! ways. C1 C1
So, exhaustive number of cases = 19!
Excluding A and B, out of remaining 18 girls, 4 girls 2
1 C1 1
can be selected 18C4 ways which can be arranged in 4! Probability of choosing I = 9 10 45
C1 C1
ways.
Remaining 20 – (4 + 2) = 14 girls can be arranged in 3 3
C1 C1 1
14! ways. Also A and B mutually can be arranged in 2! Probability of choosing S = 9 10
C1 C1 10
ways.
Required number of arrangedment = 18C4 × 4! 2 3
×2! ×14! C1 C1 1
Probability of choosing T = 9 10 15
= 18! ×2 C1 C1
18! 2 2 Hence, required probability
Required probability =
19! 19
44. (d) Since x and y can take values from 0 to 10. So, the 1 1 1 1 19
=
total number of ways of selecting x and y is 11×11 45 45 10 15 90
= 121

PDF Download FROM >> WWW.SARKARIPOST.IN Join >> https://www.facebook.com/Sarkaripost.in/


WWW.SARKARIPOST.IN
Probability 651

Explanation of
Test Yourself

1. (b) Total no. of students in four schools 5. (a) (2/3) × (3/4) + (1/3) × (2/7) = (1/2) + (2/21) = (25/42)
= 12 + 20 + 13 + 17 = 62. 6. (c) A and B will contradict each other if one speaks truth
Now, one student is selected at random. and other false . So , the
Total outcomes = 62C1
4 3 4 3
Required probability 1 1
Now, no. of students in school B2 = 20. 5 4 5 4
No. of ways to select a student from B2 = 20C1 .
4 1 1 3 7
20C1 20 10 5 4 5 4 20
Required probability = =
62C 1 62 31
7. (a) The three balls that are taken out can be either 3 black

WWW.SARKARIPOST.IN
2. (b) Total no. of days in a leap year = 366 balls or 2 black and 1 red ball or 1 black and 2 red ball
i.e., 52 weeks and 2 days. or 3 red balls.
Thus, leap year has 52 Sunday and remaining two days Each of these will give their own probabilities of
can be : drawing a black ball.
(1) Sun and Mon (2) Mon and Tue 8. (c) Two squares out of 64 can be selected in
(3) Tue and Wed (4) Wed and Thu 64 63
64
(5) Thu and Fri (6) Fri and Sat C2 32 63 ways
2
(7) Sat and Sun
The number of ways of selecting those pairs which
Total no. of events = 7
have a side in common
Let A be the event that leap year has 53 Sundays.
There is two ways that remaining two days contain 1
Sunday is = (4 × 2 + 24 × 3 + 36 × 4) = 112
2
(1) Sun and Mon (2) Sat and Sun.
Favourable no. of cases = 2 [Since each of the corner squares has two neighbours
each of 24 squares in border rows, other than corner
2
P (leap year contain 53 Sunday) = ones has three neighbours and each of the remaining
7
36 squares have four neighours and in this computation,
2
i.e. P (A) = . each pair of squares has been considered twice].
7
3. (b) Total no. of balls = 10 .112 1
Hence required probability = .
3 32 63 18
P (drawing one red ball) = ( red ball = 3) 9. (b) Probabilities of winning the race by three horses are
10
P (drawing second red ball without replacement) 2 1 1 1
9 , and .
Required probability 3 2 1 3 4 5
10 9 15 Hence required probability
4. (a) The probability of the problem being not solved 1 1 1 47
=
1 1 3 4 5 60
by A, (P1) = 1 – 10. (a)
2 2
Similarly, the probability of the problem being not
1 2 2/3 5/9
solved by B, (P2) = 1 –
3 3
The probability that the problem being solved by
1 2 1
neither A nor B, (P3) = P1 × P2 =
2 3 3 X
Hence the required probability
From the venn diagram we get:
1 2
P = 1 – P3 = 1 – . (2/3) + (5/9) – x = 4/5 x = (2/3) + (5/9) – (4/5) = 19/45
3 3

PDF Download FROM >> WWW.SARKARIPOST.IN Join >> https://www.facebook.com/Sarkaripost.in/


WWW.SARKARIPOST.IN
652 Quantitative Aptitude

11. (d) P of heads showing on 50 coins 2 1 5


P ( A) P(B)
= 100C50 × P50(1 – P)50 3 6 6
P of heads showing on 51 coins = 100C51 × P51(1 – P)49 1 5
P ( A) [from (i)]
Both are equal 6 P( A) 6
100C × P50(1 – P)50 = 100C × P51(1 – P)49
50 51 6 [P(A)]2 – 5P (A) + 1 = 0
100 49 ... 52 51 1 1
or (1 P ) (2P (A) – 1) (3P (A) – 1) = 0 P( A) ,
1 2 ... 49 50 2 3
100 99 ... 53 52 14. (b) We do not have to consider any sum other than 5 or 7
= P occurring.
1 2 ... 48 49
A sum of 5 can be obtained by any of
51 [4 + 1, 3 + 2, 2 + 3, 1 + 4]
or (1 P) = P
50 Similarly a sum of 7 can be obtained by any of
or 51 × (1 – P) = 50 P [6 + 1, 5 + 2, 4 + 3, 3 + 4, 2 + 5, 1 + 6]

WWW.SARKARIPOST.IN
or 51 – 51P = 50 P For 5: n(E) = 4, n(S) = 6 + 4
or 51 = 101 P P = 0.4
For 7: n(E) = 6
51
P= n(S) = 6 + 4
101 P = 0.6
12. (a) In the first draw, we have 7 even tickets out of 15 and 15. (b) Total no. of pages = 1000
in the second we have 8 odd tickets out of 15. No. of one digit nos. summing 9 = 1
Thus, (7/15) × (8/15) = 56/225.
No. of two digit nos. summing 9
13. (d) Since A and B are two independent events
(1 & 8, 2 & 7, 3 & 6, 4 & 5, 90) = 2! × 4 + 1 = 9
P( A B) P( A) P ( B ) No. of three digit nos. summing 9 = nos. in which last
digit is 0 (180, 270, 360, 450, 900)
P( A) P ( B ) (given) ...(i) + nos. with all 3 different digits (126, 135, 234)
6
+ nos. with 2 same digit (144, 171, 252)
1 + nos. with all 3 digits (333)
and P (neither of A nor B) = P ( A B)
3 3!
17 3 3! 3 1 45
2!
1 2
P( A B) 1 P( A B) 1 [ each number out of Nos. 180, 270, 360 and 450
3 3 can be arrange in 4 ways.
We know that no. of ways = 4(4) = 16]
P( A B) P( A) P(B) P( A B) Total favourable cases = 1 + 9 + 45 = 55
Therefore, required probability
2 1
P( A) P ( B )
3 6 Total favourable cases 55
=
Total outcomes 1000

PDF Download FROM >> WWW.SARKARIPOST.IN Join >> https://www.facebook.com/Sarkaripost.in/


WWW.SARKARIPOST.IN

Mock Tests

WWW.SARKARIPOST.IN
 Mock Test-1

 Mock Test-2

 Mock Test-3

 Mock Test-4

 Mock Test-5

PDF Download FROM >> WWW.SARKARIPOST.IN Join >> https://www.facebook.com/Sarkaripost.in/

You might also like